Cambio Climático

Mostrando 234 respuestas a los debates
  • Autor
    Mensajes
    • #28164
      zero1
      Participante

      El cambio climático global:

      Dado que existe todavía un cierto número de personas que creen que el cambio climático es un invento conspiranoico de una élite mundial que quiere dominar el mundo, inclusive dentro de la propia comunidad científica, dejo un resumen que encontré sobre el cambio climático. Está en forma de cómic, pero no deja de ser explicativo.

      http://tallguywrites.livejournal.com/154171.html

       

      La NASA preparó una página con los hechos y evidencias del cambio climático global para quien quiera consultarla antes de despotricar (está en inglés).

      http://climate.nasa.gov/evidence/

    • #30480

      Lo había leído. Se va un poco por las ramas (lo de los intereses económicos está de más, a mi gusto) y le me hubiera gustado que tuviera más sobre la evidencia del CC. 

      Igual el pingüino es tierno. 

    • #30481
      N3RI
      Superadministrador

      abriste la caja de pandora :P

    • #30482
      DrGEN
      Miembro

      Tarde o temprano iba a aparecer el tema.

      Es sano que en un sitio sobre escepticismo se trate este tema, obviamente.

      Zero1:

      Dado que existe todavía un cierto número de personas que creen que el cambio climático es un invento conspiranoico de una élite mundial que quiere dominar el mundo, inclusive dentro de la propia comunidad científica

      Yo soy uno de los que no está para nada convencido del verso del cambio climático antropogénico.

      Claro está que si partís de una postura como la tuya, casi descalificando a la misma comunidad científica que alberga las mismas dudas que yo, no sé para qué ponés el tema en este foro de escépticos.

      Es decir, si empezamos diciendo que los que no "creen" (como si en ciencia se tratase de "creer"!!) en eso son conspiranoicos, luego vendrán adjetivos como "negacionistas".

      Saldremos los que pensamos diferente diciendo que los que sí creen (acá me parece más acertado el término) en la religión del Cambio Climático son unos "calentólogos", que son unos "alarmistas" y más…

      De ahí a que en los siguientes comentarios comiencen los ataques ad hominems estamos a un paso… y al diablo el debate.

      O mantenemos la actitud escéptica sobre el tema o sino es al pedo dejar un hilo abierto donde de entrada das por cerrado el tema (encima descalificando).

       

      Para que lo piensen, les dejo la prueba contundente de cambio climático…

      tan contundente que es infalsable ;)

       

      Inviernos en Hemisferio Norte más cálidos por el calentamiento global

      Inviernos en Hemisferio Norte más fríos por el calentamiento global

      Calentamiento global disminuye velocidad de rotación de la Tierra

      Calentamiento global aumenta la velocidad de rotación de Tierra

      Atlántico Norte está menos salado

      Atlántico Norte está más salado

      Las avalanchas aumentarán

      Las avalanchas disminuirán

      Monsones más secos en India

      Monsones más húmedos en India

      El plankton crecerá

      El plankton disminuirá

      Menos nieve en los Grandes Lagos

      Más nieve en los Grandes Lagos

      San Francisco con más niebla

      San Francisco con menos niebla

      Menos nieve en invierno Británico

      Más nieve en invierno Británico

      Africa: menos lluvia

      Africa: más lluvia

      Vientos se aceleran [USA]

      Vientos se desaceleran [USA]

       

      Cuál era la otra "teoría" infalsable muy conocida?

      Ah… el psicoanálisis :P

    • #30483
      DrGEN
      Miembro

      Igual el pingüino es tierno.

      Che… igual… "más perdido que pingüino en el ártico" :P

    • #30484
      zero1
      Participante

      Buenas DrGEN:

      Yo también considero importante que estos temas controvertidos se discutan. Lo de "conspiranoicos" se aplica a cualquiera que cree que detrás de todo hay un plan orquestado para dominar al mundo, como ocurre con la vacunación, el sida, robo de organos, etc. No iba dedicado a nadie en especial. Tampoco conocía su posición, por lo tanto no lo tome como personal. 

      Durante algún tiempo yo también tuve mis dudas sobre si esto del cambio climático es algo real o un gran invento. Por eso empecé a leer un poco más y no me quedé con las teorías negacionistas solamente. Conocía algo sobre la influencia que las teorías de Milankovitch, bamboleo de Charles, ciclos solares,  los rayos cósmicos, etc tienen sobre el clima. Lo cierto es que los estudios que se han realizado sobre éstas apuntan a descartarlas como "la causante" del cambio climático. Ninguna logra explicar las temperaturas actuales ni la concentración de CO2 atmosférico (la más alta de la historia).

      Además hay que comprender la diferencia entre clima y el tiempo. El tiempo se refiere a las condiciones actuales, qué temperatura hace hoy, la humedad, presión, velocidad del viento, etc. Mientras que el clima refiere los valores estadísticos de esos elementos del tiempo, medidos durante un gran período.

      http://www.youtube.com/watch?v=ocOMA_1vtp4

       

      De todos modos, me parece correcto que plantees tus dudas (o mejor dicho tus convicciones) para que se pueda debatir sobre cada argumento con datos concretos. 

       

      Saludos.

       

    • #30485
      DrGEN
      Miembro

      Zero1:

      No hay problema, no lo tomo personal. De hecho en tu comentario inicial caen como "conspiranoicos" todos los científicos climatólogos que no soportan las afirmaciones del IPCC.

      Esto, no solo no es cierto sino que estás sobreestimando a personas que estudiaron y entienden de clima seguramente muchísimo más que vos (y seguro más que yo).

      O sea, vamos, pensás que todos esos científicos hablan de una conspiración? No, no es así, muestran que la ciencia climática está muy lejos de un consenso, sin entrar a hablar de intereses ni nada por el estilo. Simplemente en base a datos y observaciones… ciencia en fin y no política.

      Por otro lado, es incorrecto decir que la concentración de CO2 actual es "la más alta de la historia". Excepto en los tiempos de glaciaciones, hubo casi siempre concentraciones más grandes que las actuales.

      En el jurásico se calcula que el CO2 tenía una concentración de 2000ppm, unas 5 veces más que la actual! GEOCARB III (y los dinosaurios y toda la vida de enonces la pasaban de 10!). E incluso huvo períodos (Cámbrico) con concentraciones de hasta 8000ppm, 20 veces más que la actual!

      Conozco perfectamente la diferencia entre "tiempo" y "clima", así que no te preocupes que no caigo en ese error.

      Por último, no son mis convicciones lo que planteo, sino cuestiono ese supuesto consenso que no es tal, basado en pruebas "inequívocas" que no son tales. De existir esas pruebas, sería el primero en aceptarlas, soy escéptico y no negacionista.

      Saludos!

      PD: Fijate que en tu comentario, volvés a decir "negacionistas"….

    • #30486
    • #30487
      zero1
      Participante

      Los niveles de CO2 fueron superiores en el pasado:

      "La prueba de que el CO2 no conduce climático se encuentra durante el Ordovícico-Silúrico y los períodos Jurásico-Cretácico, cuando los niveles de CO2 eran mayores a 4000ppmv  y 2000ppmv (partes por millón por volumen), respectivamente. Si la teoría del IPCC  es correcta debería haber habido un de efecto invernadero desbocado y calentamiento global inducido durante estos períodos, en lugar de una glaciación."

       

      Cuando los niveles de CO2 eran más altos en el pasado, los niveles solares también fueron menores. El efecto combinado del sol y el CO2 se adapta muy bien con el clima.

      A lo largo de la historia de la Tierra, hubieron  momentos en donde el CO2 atmosférico tenía nivel mayor que los niveles actuales. Curiosamente, el planeta experimentó regiones generalizadas de glaciación durante algunos de esos períodos. ¿Contradice esto el efecto de calentamiento del CO2?

      No, por una sencilla razón. CO2 no es el único motor del cambio climático . Para entender el clima del pasado, tenemos que incluir otros forzamientos climáticos. Para ello, hay un estudio de 490 registros proxy para la reconstrucción de los niveles de CO2 en los últimos 540 millones de años ( Royer, 2006 ). Este período es conocido como la era Fanerozoico.

       

       

      Figura 1: El CO2 atmosférico a través del Fanerozoico. La línea de puntos muestra las predicciones del modelo de ciclo de carbono GEOCARB con sombreado que representa la escala de incertidumbre. La línea sólida muestra alisada representación del registro del proxy ( Royer, 2006 ).

       

      Los niveles atmosféricos de CO2  han alcanzado los valores espectaculares en el  pasado, posiblemente superando más de 5000 ppm en el Ordovícico tardío, hace unos 440 millones años. Sin embargo, la actividad solar también disminuye a medida que uno retrocede. A principios de los Fanerozoico, la radiación solar era cerca de 4% menor que los niveles actuales. El efecto neto combinado de las variaciones de CO2 y la energía solar se muestra en la Figura 2. Los períodos de hielo geográficamente extendidos se indican las áreas sombreadas.

      Figura 2: forzamiento radiativo combinado de CO2 y el sol a través del Fanerozoico. Los valores se expresan en relación con las condiciones pre-industrial (CO2 = 280 ppm; luminosidad solar = 342 W / m 2). Las bandas sombreadas oscuras corresponden a los períodos con fuerte evidencia de hielo extendida geográficamente.

       

      Los períodos de bajas emisiones de CO2 coinciden con los períodos de la extensión geográfica de hielo (con una notable excepción, más adelante). Esto lleva al concepto del umbral de CO2-hielo – el nivel de CO2 necesaria para iniciar una glaciación. Cuando el sol es menos activo, el umbral de CO2-hielo es mucho mayor. Por ejemplo, mientras que el umbral de CO2 de hielo de la Tierra hoy en día se estima en 500 ppm, el umbral equivalente durante el Ordovícico (hace 450 millones de años) es de 3000 ppm.

      Sin embargo, hasta hace poco, los niveles de CO2 durante el Ordovícico tardío se creían  mucho mayores de 3000 ppm, que era problemático, ya que la Tierra experimentó condiciones glaciales en este momento. Los datos de CO2 en el Ordovícico tardío son escasos, con un punto de datos en el registro de cerca de proxy de CO2 a este período – que tiene un valor de 5.600 ppm. Dado que la radiación solar fue de alrededor de 4% inferiores a los niveles actuales de CO2 tendría que caer a 3.000 ppm para permitir condiciones  glaciales. ¿Podrían los niveles de CO2 haber caído tanto? Dada la baja resolución temporal del registro de CO2, los datos no fueron concluyentes.

      Una Investigación que examina los isótopos de estroncio en el registro sedimentario arrojar más luz sobre esta cuestión ( 2009 Young ). La erosión de las rocas elimina el CO2 de la atmósfera. El proceso también produce un isótopo particular de estroncio, regado hacia los océanos a través de los ríos. La proporción de isótopos de estroncio en las capas de sedimentos se pueden utilizar para construir un registro proxy de la actividad de la erosión continental. El registro de estroncio muestra que alrededor del Ordovícico Medio, la  mayor erosión que conduce a un aumento en el consumo de CO2. Sin embargo, esto se compensó con mayor emisión de gases volcánicos agregando CO2 a la atmósfera. Hace alrededor de 446 millones de años, la actividad volcánica se redujo, mientras que la meteorización se mantuvo alta. Est hizo caer el nivel de CO2  por debajo de 3000 ppm, iniciando el enfriamiento. Resulta que el descenso del nivel de CO2 fue la causa de la glaciación del Ordovícico tardío.

      Así vemos que para  la comparación del clima de hoy con los períodos de 500 millones de años atrás hay que tener en cuenta que el sol fue menos activo que en la actualidad. ¿Cuántas veces más cerca que hoy? En el período más reciente, cuando los niveles de CO2 eran tan altos como hoy en día (alrededor de 400 ppm) fue hace unos 15 millones de años, durante el Mioceno Medio. ¿Cuál fue el clima en ese momento? Las temperaturas globales fueron de 5 a 10 grados Fahrenheit mayores que hoy en día. El nivel del mar era aproximadamente de 75 a 120 pies más alto. No había ninguna capa de hielo permanente en el Ártico y el hielo muy poco sobre la Antártida y Groenlandia. El estrecho acoplamiento entre el CO2 y el clima llevó al autor a concluir que "las observaciones geológicas que ahora tenemos en los últimos 20 millones años dan un fuerte apoyo a la idea de que el dióxido de carbono es un agente importante para impulsar el cambio climático a lo largo de la historia de la Tierra". ( Tripati 2009 ).

      Si los científicos del clima estaban reclamando que CO2 fue el único motor del clima, luego la gran cantidad de CO2 durante los períodos glaciales sería problemática. Sin embargo, cualquier científico del clima le dirá de CO2 no es el único piloto del cambio climático. 

      Refutación escrita por John Cook. Última actualización el 9 de julio de 2010.

      http://www.skepticalscience.com/co2-higher-in-past-intermediate.htm

      http://www.skepticalscience.com/co2-higher-in-past-basic.htm

    • #30488
      arielbenz
      Miembro

      Quiero dejar un informe que el otro día hojeando unas revista de Discover en Español encontre y me pareció bastante interesante para el tema que se debate, el informe trata sobre John Christy que es uno de los que está en contra de la versión oficial del cambio climático. Son 6 páginas y corresponden a la edición de marzo de 2001, a pesar que tiene diez años, tiene información interesante de como John llegó a sus conclusiones y las respalda con datos tomados por su equipo.

    • #30489
      sebas
      Miembro

      Hola. Saludos a todos, en mi primera intervención en el foro.

      Veamos. Cosas en las que creo:

      · El hombre llegó a la Luna en 1969.

      · El Gobierno de USA nada tuvo que ver en la perpretración de los atentados a las Torres Gemelas.

      · A Kennedy lo mató Lee Harvey Oswald, un loquito, por las suyas, y de loco que estaba, nomás.

      · A Lee Harvey Oswald lo mató Jack Ruby, otro loquito, por las suyas, y de loco que estaba, también.

      · Ningún gobierno del mundo tiene cadáveres de extraterrestres ni restos de ovnis ni nada.

      · Etc.

      Así, me resisto a ser considerado un conspiranoico ni mucho menos, aun cuando no crea en El Cambio Climático Producido Por El Hombre Que Nos Va A Llevar Al Desastre Si No Hacemos Algo Pronto Para Evitarlo.

      ¿Por qué no creo? ¿Soy meteorólogo acaso, o siquiera científico? Ni a palos. Como cualquiera que no sea experto, me informé hasta donde pude, me basé en mi intuición y en mi sentido común (y en mis prejuicios), tomé partido, y después le creo más a los expertos que dicen que no que a los que dicen que sí. No me voy a poner a pelar acá argumentos copipasteados; a quien le plazca, están a un tiro de google. Me limitaré a aportar (?) mis propias consideraciones, cualquier cosa menos científicas, pero al menos de cosecha personal.

      Cuando yo era chico, el flagelo, el inminente peligro, la catástrofe en ciernes, era: la polución. Si seguíamos así, en pocas décadas todo el aire del mundo se iba a volver irrespirable. Había consenso: lo decían los expertos, los ecologistas, los periodistas especializados; y quien se atreviese a dudarlo era un crédulo optimista, o peor: un cínico a sueldo de las multinacionales.

      Desde ya que el smog era (es) un problema real, en algunas ciudades grandes y/o muy industrializadas. Se aplicaron un par de leyes, disminuyeron (más por los naturales avances tecnológicos que por la legislación) las emisiones tóxicas de motores y fábricas y adiós, el problema prácticamente desapareció (no sin antes haberle dado de comer a unos cuantos por bastante tiempo), por lo menos a esos niveles de dramaticidad.

      Pero antes de eso a mí, niño que era, ya me sonaba medio raro. Cuando me fui enterando del asunto, quedé pasmado. ¡¡¡Nos vamos a quedar sin aire!!! ¡¡¡Qué mierda voy a respirar cuando sea grande!!! Sin embargo, poco después, supongo que tras haber viajado por primera vez en avión o algo, me empecé a hacer preguntas bastante obvias. Como ser: ¡Eh! Si uno sale de la ciudad donde vivimos apiñados, ve que entre ciudad y ciudad hay kilómetros, y kilómetros, y kilómetros de campo o desierto o montaña o lo que sea, y eso sin contar los océanos. ¿Las ciudades no son una superficie insignificante, a escala planetaria? Y si el smog está en las ciudades, ¿el aire no tendría que ser ya un veneno mal acá en Avellaneda, como para que el poco tiempo no se pueda respirar ni en altamar? Y yo respiro lo más bien todavía…

      Antes de que pudiera develar la incógnita, el tema pasó a quinto plano, y recién me acordé de aquellas precoces dudas mías cuando empezó a saltar el Agujero de Ozono, perdón, digo: el Calentamiento Global, perdón, digo: el Cambio Climático Antropogénico.

      Una vez hice un cálculo. La especie humana, si amontonada, tipo mitín (4 personas por m2, según el cálculo estándar) cabe en la superficie de la Provincia de Tucumán. Sí, y lo que es más: cabe trece veces. Y todavía sobra un cacho, eh. Haced el cálculo vosotros mismos, hijos míos, si de mí dudáis.

      No tenemos conciencia, porque vivimos un poquito como amuchados, pero a escala planetaria no somos nada. Un musguito. Menos. Apenas sí existimos. Me resisto a creer que podamos tener la menor influencia sobre el clima a nivel global. Por más actividades que emprendamos. Simplemente no-damos-la-talla.

      Desde ya que la podemos cagar. A nivel local. Podemos estropear nuestro entorno inmediato; y vaya si lo hemos hecho. Podemos, si le ponemos ganas, lograr que el Riachuelo sea un verdadero asquete. Y hasta que el Río de la Plata dé un poco de cosa. Pero ya el Atlántico, sepámoslo, se nos caga bien de la risa, me temo.

      ¿Y por qué tanto ruido con el CC, entonces? ¿Sospecho de una cospiración internacional? Nah. Ni hace falta. Alcanza con que un tema tenga prensa y se ponga de moda. Lo vemos mil veces. La gente le tiene miedo al terrorismo, a los pibes chorros, a la gripe A, y la gente es la que consume noticieros, y la gente es la que vota, entonces el periodismo se ocupa de ello, y los políticos se ven forzados a ocuparse, y las empresas que quieren mantener su imagen, y los científicos que reciben fondos del Estado y de las empresas; y de mil cuestiones mucho más decisivas pero sin prensa (como el Chagas, o ¡las bañeras, gran flagelo que se cobra miles víctimas que se resbalan cada año!) se ocupa Magoya. Un círculo que se retroalimenta. Y bué.

      Toda la puta cuestión del CC, si llegáremos a tener razón los negacionistas, sería sumamente dañina. Se dedican inapreciables recursos que podrían destinarse a problemas reales (como ser en nuestro caso, ya que lo mencioné, el Riachuelo). Pone trabas al desarrollo donde más necesario es (andá a recibir un crédito del Banco Mundial para cualquier cosa que emita un poquito de carbono -o sea, para cualquier desarrollo industrial o energético real-, hoy por hoy). Genera unos negocios más turbios que (ya que lo mencioné) el agua del Riachuelo… ¡los bonos del carbono, los dioses sean loados! Y unos largos etcéteras.

      Antes de que alguien me crea pago (¡ojalá alguien me pagara por estas boludeces que escribo por estar de vacaciones y al pedo!) por las corporaciones, declaro solemnemente que si por mí fuera, expropiaría a todas las petroleras ya mismo y que vuelva la YPF estatal. Así de derechoso soy.

      A mí me resulta más bien derechosa toda esta cosa climática. Porque entonces la culpa, en mayor o menor medida, la tenemos todos. Hay sólo culpables: grandes, sí, y pequeños culpables, pero culpables todos al fin. Cada vez que me tiro un pedo o respiro estoy contribuyendo. Cada vez que prendo la luz o tiro la cadena. No una cofradía de grandísimos hijos de puta: es la especie humana in toto la responsable de todos los males. Pero que se vayan a cagar.

      Y por cierto: si éstos son la izquierda según la historieta esa, yo soy Mao-tsé Tung, mirá. ¡Aguante Dee Snider!

      Otrosí: los (perdón) alarmistas usan algunos métodos que me resultan bastante sospechosos. ¿En qué se basan para predecir que si seguimos así en tantos años vamos a estar no sé cómo? ¿En modelos y simuladores informáticos? Ah, ¿sí? ¿Alguna vez alguno de ustedes vio uno? A mí me parecen herramientas muy útiles para comprobar si en teoría, planteando una versión matemática ultra mega giga simplificada de un escenario tal, podría ser que la cosa funcionara de tal o cual modo. Punto. Cómo evolucionarían los seres vivos si… si los seres vivos fueran unos triangulitos; pero cargale toda la información genética y todas la variables efectivas de un ecosistema real, y no habrá la recontrasúper computadora que te pueda arrojar un resultado prelim inar en menos de 500 milenios. No jodamos. El clima es algo terriblemente complejo que recién empezamos a comprender; y tocando cualquier variable, aun a nivel infinitesimal, un simulador va a dar lo que se te cante. Las simulaciones servirán para poner a prueba consistencias internas de una teoría; pero para predecir hechos del mundo real, pindonga. Y cuando me quieren correr con eso, con titulares catastróficos basados en algo tan endeble, ya empezamos mal; ya, como decía El Carpo (Q.E.P.D.): desconfío.

      Los gritos de condena hacia la especia humana y sus espantosos e irredimibles actos que desencadenarán el Apocalipsis siempre me suenan más a Antiguo (y/o Nuevo) Testamento que a ciencia de la científica.

      Bueno, ya tecleé más que demasiado. Hacía mucho que necesitaba hacer catarsis con esto, se ve; y la ligaron acá. Perdón. Abrazos.

    • #30490
      Malena
      Participante

      Debo decir Sebas que yo sí creo en el cambio climático (aunque con mis momentos de agnosticismo). Aunque ésta no es ni a palos la época más cálida del planeta, ni lo va a ser ni aunque aumentaramos 10 grados globales (y eso es un montón aunque no lo parezca), la velocidad del cambio es lo que asombra a los científicos.

      Aunque los seres humanos entremos 13 veces en tucumán, no somos pocos, estamos muy dispersos, y podemos hacer mucho daño. Un solo cástor puede cambiar todo un ecosistema, que no pueden hacer 6000 millones de primates con fuego.

      Pero aún así:

      a) no sé hasta que punto es revertible.

      b) no sé hasta que punto convendría reventirlo.

      c) no estoy segura que sea algo malo (los árboles del mundo están bastante contentos).

      d) a lo sumo a los únicos que afectará realmente van a ser: las especies que de todas formas se iban a extinguir. Otras especies floreceran y nuevas especies nacerán (como siempre que aparecen nuevos nichos). Ni siquiera va a afectarnos mucho a nosotros, nos adaptaremos, como hacemos siempre. Catastrofes naturales hay con o sin cambio climático. Y las provocadas por el hombre también, la desertificación es independiente al cambio climático.

      e) reciclar, reusar y toda la bola, consumir menos luz, agua, etc., es algo que deberíamos hacer de todos modos con o sin cambio, por la economía y por nuestra salud. Mientras menos basureros haya, mejor.

    • #30491
      zero1
      Participante

      Me parece que no se trata de "creer" o "no creer" en el cambio climático cual religión, sino de entender que hay sobrada evidencia que apunta a que esta teoría del calentamiento global antropogénico es la más correcta. Se han propuesto diversas otras causas, como ya sabrán, y no alcanzan para explicarlo. Hay consenso científico. El hecho que existan personas que niegan el asunto (aun siendo palabra autorizada en el tema) no sirve para tirar abajo la teoría, evidencia, modelos, etc. (argumentum ad verecundiam,  ej.: si Montagnier niega las causas establecidas del sida, siendo él quien descubrió el hiv, entonces el resto de la comunidad científica miente para ganar prestigio y fondos). Es cierto, todavía hay algunos puntos que se deben investigar mejor, pero no por ello la teoría es refutada (como en el caso de la teoría de evolución que si bien contiene puntos que faltan entender mejor, la misma no pierde inmediatamente su validez como pretenden algunos creacionistas).

      Ojo, no confío mucho en los alarmistas que predican el apocalipsis para el futuro inmediato. No sé hasta qué punto las predicciones sobre catástrofes que van a ocurrir llegan al sensacionalimo. También tengo mis dudas en los fundamentalistas ecológicos que quieren reciclar hasta sus propias heces para contribuir con el medioambiente y hacerlo "eco-amigable". Pero no por eso debemos restale importancia.

      Es cierto que el tema se ha politizado mucho, la película de Al Gore en lugar de mostrar evidencia concreta y llamar a expertos a discutirlo, se trató de su persona, su carrera política, su familia y de cómo él "heroicamente" nos ayudaría a solucionar el problema para las generacioenes futuras y bla, bla, bla…

      También, por supuesto, hay quienes detrás de esto hicieron grandes negocios como con los cupos de emisiones de co2 para las industrias. Ud. puede seguir emitiendo gases más allas del límite impuesto si le compra el excedente a otras empresas.

    • #30492

      Toda la puta cuestión del CC, si llegáremos a tener razón los negacionistas, sería sumamente dañina. Se dedican inapreciables recursos que podrían destinarse a problemas reales (como ser en nuestro caso, ya que lo mencioné, el Riachuelo). Pone trabas al desarrollo donde más necesario es (andá a recibir un crédito del Banco Mundial para cualquier cosa que emita un poquito de carbono -o sea, para cualquier desarrollo industrial o energético real-, hoy por hoy). Genera unos negocios más turbios que (ya que lo mencioné) el agua del Riachuelo… ¡los bonos del carbono, los dioses sean loados! Y unos largos etcéteras.

      En realidad, el costo de adaptarse no es tan grande como lo hacés ver. 

       

      fuente.

    • #30493
      Kenshin
      Miembro

      Hace tiempo en la epoca en que me dedicaba a ponerle palos en la rueda al movimiento Zeitgeist, encontre entre los archivos de su Foro la siguiente pagina.

      http://www.mitosyfraudes.org/Calen7/Solar23.html

      A pesar del formato ultranacionalista y conspiranoico de la misma; el texto resulta ser muy interesante.

      Al parecer el cambio climatico es mas bueno que malo. ¿Como? Segun esta pagina, el sobrante de CO2 en la atmosfera esta impulsando al agro mundial, esto se debe a que dicho gas constituye la principal fuente de alimento de las plantas (eso se ve en primaria, las plantas no sacan su alimento del suelo sino del aire y lo sintetizan usando luz solar). Y de esta forma las plantas se encuentran menos estresadas y crecen mas, y mas rapido.

      Tambien da otros datos interesantes.

    • #30494

      Claro. Si no contamos el aumento del nivel del mar, la desertificación de grandes partes de la Tierra (y la des-desertificación de otras), los eventos climáticos más extremos, el derretimiento de las capas polares y los glaciares…. algunas consecuencias del cambio climático van a ser buenas.

    • #30495
      sebas
      Miembro

      De acuerdo: no se trata de creer o no en el Cambio Climático. Se trata, como dije, de creer o no en el Cambio Climático Producido Por El Hombre Que Nos Va A Llevar Al Desastre Si No Hacemos Algo Pronto Para Evitarlo, lo que ya es otra cosa. Es un paquete que contiene 5 ó 6 afirmaciones juntas; y conque cualquiera de ellas, una, fuera errónea, se cae todo el paquete –así que, Malena, si no una negacionista, sos por lo menos una "dudacionista" (?). Y el paquete que se viene en banda se llama Protocolo de Kyoto, que es lo que al cabo importa.

      Siempre se dice que el tema se desvirtuó, se puso "muy politizado". A mi humilde entender, es un tema político desde el vamos. Pone en juego intereses, asignación de recursos, definición de políticas públicas, económicas, productivas, etc etc; si éste no es per se un tema político, no sé qué cosa lo sería.

      Si se terminase imponiendo el dichoso protocolo, en mayor o menor medida, pero seguro, aplicaría un impulso recesivo en la economía mundial (que ya viene de por sí bastannnte jodidita). Para gente como ustedes y yo, verdaderos privilegiados, unos puntos de recesión podrían implicar, qué sé yo, que el viejo monitor de la PC deberá tirar un par de años más de lo planeado, y cosas por el estilo. Pero para los cientos de millones de humanos que hoy apenas sobreviven al día, podría significar la debacle. Y si fuera por una falsa alarma… Son cosas con las que no se jode, y que trascienden por mucho las meras ganas de ganar una discusión en un foro –¡como si eso hubiese suucedido alguna vez! :)

      Con franqueza admito que mis conocimientos ni mi capacidad dan para que no me pierda en la maraña de gráficos que tiran de uno y otro lado. ¡Vean, vean como la temperatura va siguiendo al CO2! ¡Vean, vean cómo el CO2 va siguiendo a la temperatura! (y siempre es el mismo gráfico). Tengo dos posibilidades: me meto en la carrera de meteorología y en unos años les cuento, o sigo el debate entre expertos hasta donde me dé la mente, y tomo partido por la postura en la que identifico mi visión del mundo. Qué remedio.

      Hace bastante tiempo, vienen chocando dos concepciones opuestas sobre la naturaleza. La visión que llamaré grimpís, que considera la naturaleza como un jarrón de fina porcelana china, que en cualquier momento los humanos, con nuestras zarpas embrutecidas por la codicia, vamos a estrolar el precioso tesoro y hacerlo añicos. No sé por qué; por reminiscencias bíblicas potenciadas por Hiroshima, tal vez, o por lo que fuere, es hoy la visión dominante. Yo pertenezco al bando minoritario: siento un profundo respeto, cabal, por la natura; pero respeto en serio; para mí no es una criaturita agonizante por nuestras negligencias, sino un mostro descomunal, ciego, sordo, imparable, mil millones de veces más poderoso que yo, al que si un día se le da por deshacerse de nosotros lo haría con tanta facilidad como quien se sacude la caspa del hombro.

      ¡Planeta, voy a salvarte! –y el planeta se recaga de la risa un buen rato y me manda a mudar de un pestañazo.

      La idea de que seamos cualquier tipo de amenaza para la vida me resulta totalmente fuera de escala, de una arrogancia antropodelirante llevada al infinito. Insisto: podemos cagar nuestro entorno inmediato, y hasta podemos joderle la vida a unas pocas especies, si ya venían medio debiluchas y nos ensañamos particularmente con éllas. Pero… ¡Para la vida, para toda ella! Caramba, yo no logro ser una amenaza seria para unas hormigas que me aparecieron en casa, ni para unas plantitas empecinadas en germinar en las grietas de mi medianera…

      Antes venía un tsunami y resultaba que era el castigo de los dioses por nuestros aberrantes pecados (garchar, etc). Hoy, el tsunami es el castigo por nuestros aberrantes pecados de vivir, desarrollarnos, producir; por atrevernos a andar en nuestros autitos, prender luces, criar ganado pedorrero: ¡condenados! ¡Os lo dije, impíos: pretendisteis dominar la naturaleza, y mirad lo que habéis logrado! –grita el Moisés lilito-ecologista desde la montaña.

      Bien: esa visión apocalíptica del hombre, que sigue presente, que cala hasta en lo mejorcito del ámbito científico –vean a Sagan cuando mete la paradoja de Fermi en la ecuación de Drake, calculando que (no me acuerdo bien la relación, pero pongamos) 3 de cada 4 especies inteligente terminan autodestruyéndose en una guerra nuclear; o Hawking, que dijo hace poco que a la humanidad le queda ¡¡¡un siglo de vida!!!— es el substrato filosófico en el cual el CCAC (Cambio Climático Antropogénico Catastrófico, de ahora en más) puede prender con facilidad, en un campo de la ciencia que está en pañales (hasta no hace mucho, era común decir que si el servicio meteorológico predijo día soleado, mejor llevá el paraguas; recién en los últimos años la empezaron a pegar más o menos bien… de acá a tres días, no pidamos más).

      Por supuesto que un castor puede alterar un ecosistema. El de la Laguna de Cochicó, pongamos. Ok. De ahí a afirmar que esa alteración va a repercutir en el ecosistema del delta del Meh Kong… eso ya me resulta bastante más difícil de comprar. Ahora: cuando encima aparece un tipo que dice que puede predecir esa repercusión, munido de un software simulador, ya me toman para el churrete.

      6000 millones de primates con fuego (6700, para ser más exactos) desparramados es una cifra que impresiona, cómo que no. Ahora: cuando uno va comparando con la cantidad de peces, de gusanos, de insectos, de ácaros, de bacetrias, de vegetales, la cifra pierde toda espectacularidad. Y cuando se tiene en cuenta la superficie terrestre, el volumen de la atmósfera, el volumen del mar, de la biomasa misma… terminás comprendiendo que este mundo posee apenas una dosis homeopática (hehe) de humanidad.

      Admito no cazar una de meteorología, ni de química, ni de biología, ni de estadísticas, no sé otrogarle validez o ver los puntos flacos de un gráfico o de una serie de ecuaciones; y albergo la sospecha de que todo material que se blande es al menos en parte operación; hay una pugna y cada bando, como siempre fue y será en el campo que sea, opera. ¿Alguna vez siguieron un polémica entre economistas? Jamás se entiende nada, todos dicen algo opuesto al otro, todos operan flagrantemente, y, por cierto: jamás se cumple ni la menor predicción de ninguno de ellos. Y vale para todos: tanto para los que ideológicamente me gustan como para los que no. Por eso, no voy a ponerme acá a pelar argumentos ajenos que apenas entiendo. Pero cuando hablan los escépticos del CCAC, oh, oigo mi música; la reconozco al dedillo. Rocanrol, nena. Cuando hablan sus afirmadores… oigo canto gregoriano, death metal, no sé qué. Por principo, tiendo a confiar más en Penn & Teller y sus invitados que en Al Gore y Grimpís.

      Insisto: si nosotros, los descreídos, tuviéramos razón… Europa, Japón, etc. podrán darse, o creen poder (o creían hasta hace un par de años, muehehe) darse el lujo de retrotraer el desarrollo y la actividad económica, just in case. Allá ellos. Acá, en el tercer mundo, definitivamente no.

      Lo que el mundo necesita es más reactores nucleares, y no pedorros molinillos ni pedorros  paneles solares. Hace falta mucho más desarrollo, y –sobre todo– que sea muchísimo más equitativo que hasta ahora. Hace falta sacar de la miseria a un cuarto o un tercio o la mitad de la población mundial. Ésa es la prioridad, para mí (y no save the planet, que no necesita que nadie lo salve y menos que menos yo dejando de usar bolsitas de nylon o desenchufando lo aparatos para que no gaste electricidad el led que queda siempre encendido –juro haber visto una nota que recomendaba expresamente hacer eso, to save the planet, mon dieu). Y el Protocolo de Kyoto no sería precisamente lo que se dice una ayuda contra la miseria, sino una carga más. Y la puta que lo parió. Saludos para todos.

    • #30496
      Malena
      Participante

      Daneel, la desertificación es producida principalmente por malas políticas agrarias. La peor sequía y desertificación en Estados Unidos fue en la década del treinta. Acá nos veníamos salvando porque tenemos pastos adaptados a enormes mamíferos sudamericanos, que ya no están, y un horizonte fértil de más un metro, lo cual es una rareza. Pero mucho de ese horizonte era producto de los incendios generalizados, y ya no los permitimos.

      Sebas, si querés que leamos bien tus mensajes, resumí, o cortalos, son muy largos.

      Con "creo", no me refiero a "yo tengo fé", me refiero a "esa es mi conclusión después de haberme informado sobre el tema pero podría haber algo que no sé que me hiciera cambiar de opinión". No me acuerdo si es Sam Harris o Dennet el que dice que todos creemos, la diferencia está en cómo.

    • #30497

      sebas, como dice Malena, tratá de organizar tus argumentos de una manera más o menos sintética y coherente. en lo que leo hay mucho de argumento por incredulidad personal ("La idea de que seamos cualquier tipo de amenaza para la vida me resulta totalmente fuera de escala"), cuestinoes de preferencias personales ("tomo partido por la postura en la que identifico mi visión del mundo"), falso balance (la proporción de científicos que niega el CC es minúscula), confundir tiempo con clima ("hasta no hace mucho, era común decir que si el servicio meteorológico predijo día soleado, mejor llevá el paraguas") y argumentos por las consecuencias ("Y el paquete que se viene en banda se llama Protocolo de Kyoto, que es lo que al cabo importa").

      Vos mismo decís que sos un lego en la materia y que no sabés ni J sobre climatología. La enorme mayoría de nosotros estamos en la misma. Es por eso que veo como completamente ilegítimo negar el consenso sólido de una comunidad científica madura y tomar partido por una posicion tremendamente minoritaria entre los expertos en la materia (menos del 3% de los climatólogos activos). Para ponerlo en otro contexto, sería como ir a 100 mecánicos con un desperfecto técnico y decir "no se absolutamente nada de mecánica, pero a pesar de que 97 de ellos me dijeron que es el cigüeñal, voy a creerle a esos 3 que me dijeron que es el espejo retrovisor". 

    • #30498
      1nfest
      Miembro

      Opino igual que Daneel.

      Yo se de quimica, y pensandolo desde esa optica me parece razonable el calentamiento global producto de la influencia humana, evidencias hay ha montones.

      Quizas la linea de razonamiento mas asequiable para el que no sabe nada de ciencia sea esta:

       

      Hay tres hechos científicos sobre los que no cabe duda:

      1. Sin la presencia natural de los gases invernadero que atrapan el calor en la atmósfera, la Tierra sería un planeta gélido incapaz de sostener la vida tal como la conocemos.

      2. Pero los gases invernadero se están acumulando en la atmósfera más allá de los niveles naturales, y ciertas actividades humanas, sobre todo la quema de combustibles fósiles, son la causa.

      3. La cantidad de dos de los gases invernadero más importantes presentes en la atmósfera, dióxido de carbono y metano, ha aumentado mucho más que en cualquier época en por lo menos 220.000 años.

      Que resultado deberiamos esperar si nos basamos en esos 3 hechos?

    • #30499
      zero1
      Participante

      Correcto 1nfest. Me tomo el atrevimiento de agregar unos puntos más que también me parecen importantes:

       

      4.    La temperatura aumenta debido a la mayor concentración de CO2 atmosférico.

      5.    La temperatura de los océanos aumenta también, esto disminuye la concentración de CO2 disuelta en el agua (recordar que la solubilidad de un gas en un líquido disminuye con el aumento de temperatura).

      6.    El CO2 que estaba en los océanos va a para a la atmósfera, lo cual aumenta la concentración de CO2 en la misma.

      7.    Se da un efecto de realimentación positiva que empeora aún más la situación (volvemos al punto 4. pero con más CO2).

    • #30500
      1nfest
      Miembro

      Es cierto y aunque la solubilidad cambia muy poco con la temperatura, el volumen de los oceanos es tan grande que hace un diferencia importante. Amen de las 2 toneladas de CO2 que produce cada persona anualmente.

    • #30501
      DrGEN
      Miembro

      A TODOS:

      Sinceramente esperaba encontrar una actitud más escéptica sobre el tema en este foro.

      Sin meternos en el tema científico de la climatología, para lo cual la gran mayoría no damos la talla, me llama la atención que los así llamados escépticos (en otros temas) se coman el verso oficial sin siquiera cuestionar cosas muy básicas. Cosas que si vinieran de la homeopatía causarían carcajadas y después un asalto feroz hacia esos "argumentos".

      En primer lugar, el "consenso". Daneel no se cansa de ponerse el cassette y repetir: "97% de los climatólogos blablabla"…

      Desde cuando la ciencia se maneja por consensos?!?!

      La ciencia no es democrática! Basta una prueba y el 99.999999 de "consenso" se tira por la borda.

      La ciencia se maneja con pruebas, la política con consensos.

      Se imaginan que a uds les digan: "Sí, el 97% de los químicos está de acuerdo con que el agua está formada por H2O"

      Consenso?!?! No pedirían pruebas?!

      Ya lo dije en otro lugar y lo repito acá, me llama poderosamente la atención que gente que está acostumbrada a leer papers se coma el verso ridículo del "consenso".

      Por cierto… curiosamente ninguno hizo ni siquiera referencia a la infalsabilidad del "cambio climático"…

       

      Daneel:

      http://climate.nasa.gov/evidence/

      Evidencias de "cambio climático"… sí, tan obvio como que te muestro un reloj y lo llamo evidencia de que el tiempo pasa.

      Vamos! Ya sabemos que el clima cambia! Siempre cambió!

      El punto es otro: es ese cambio antropogénico? y debido al CO2 del mismo origen?

      Para eso, NINGUNA de las cosas mostradas en esa página (ni en ninguna otra) son EVIDENCIAS. (ni hablar que como sabrán, evidencia no es lo mismo que prueba) ;)

      Por cierto, varias de las cosas citadas en esa página son completas falacias (ej: el hielo del ártico está bajando – NO, no está bajando: http://notrickszone.com/2011/01/31/arctic-adds-2000-cubic-kilometers-of-ice-despite-reports-of-accelerating-ice-melt/).

       

      Zero1:

      Te recuerdo que fuiste vos (y no yo) quien dijo:

      Ninguna logra explicar las temperaturas actuales ni la concentración de CO2 atmosférico (la más alta de la historia).

      Lo que yo te mostré en mi anterior comentario es que tu afirmación era falaz.

      Por supuesto que eso no contradice el efecto del CO2 en el calentamiento, pero tampoco lo prueba. De hecho… el CO2 no explica ni el calentamiento visto a principios del siglo XX ni el enfriamiento entre los años 40 y 70 del mismo.

      Por supuesto también que el CO2 no es el único motor del clima. Es más… ni siquiera es el más importante! ni el más influyente!

       

      Sebas:

      Total, pero totalmente de acuerdo con vos!

      Y, ya que estamos, no te limites a "resumir" tus comentarios. A los que les interese (mi caso) que lo lean completo, y punto.

       

      1nfest:

      Hablás de "tres hechos científicos sobre los que no cabe duda".

      El 1º es correcto.

      El 2º hablás de "gases invernadero" pero estás refiriéndote sólo al carbono y al metano (supongo). No generalices sino el punto 2 NO es cierto.

      El 3º:

      3. La cantidad de dos de los gases invernadero más importantes presentes en la atmósfera, dióxido de carbono y metano, ha aumentado mucho más que en cualquier época en por lo menos 220.000 años.

      Esto definitivamente es una falacia: el gás de efecto invernadero MAS IMPORTANTE es el VAPOR DE AGUA. El vapor de agua es un gas de invernadero más eficiente que el CO2, es el que más contribuye al efecto invernadero debido a la absorción de los rayos infrarrojos. Su contribución se calcula entre el 36 y 66% (cielo claro) y 66 y 85% (cielo nublado) del total del efecto invernadero.

      https://secure.wikimedia.org/wikipedia/en/wiki/Greenhouse_gas

      Hablaste del mismo?

      Está suficientemente estudiada la influencia de dicho gas en la dinámica climatológica?

      Está tenido en cuenta en los modelos del IPCC el gás invernadero más importante?

      Decís:

      Que resultado deberiamos esperar si nos basamos en esos 3 hechos?

      Bueno… no son 3 "hechos", por lo que… a hacer de nuevo la tarea.

    • #30502
      zero1
      Participante

      Me refería a la más alta en la historia de la humanidad. De hecho, es la más alta en los últimos 650.000 años.

    • #30503

      De hecho, la ciencia siempre se manejó por concenso. Si bien es cierto que para cuestiones más bien puntuales el concenso es poco signficativo, éste sí es importante cuando se trata de cosas más complejas. El fisiólogo que estudia un animal tiene que confiar en las teorías más aceptadas por los químicos si no tiene la formación necesaria para analizar independientemente la evidencia, las controvercias, etc… Lo mismo para el físico que utiliza teoremas matemáticos, el psicólogo que utiliza la evolución, etc… Pensar que la ciencia se realiza un vacío es una visión reduccionista al extremo de dejar de ser útil. La ciencia es una empresa comunitaria y si hay un concenso en una disciplina, hay que tener muy buenas razones para desestimarlo. 

      Obviamente que haya un concenso no quiere decir que sea verdad. Pero si la comunidad de climatólogos que conocen el tema mucho más que cualquiera de nosotros acuerda en una serie de concluisones básicas, entonces la mejor que puede hacer el lego es apostar por el concenso. Y si alguien quiere mostrar que el concenso está equivocado, entonces no lo hace desde blogs, libros o artículos, sino por el mismo sistema que construyó el concenso: revistas científicas de revisión por pares. 

      Claro que muchos responderían que eso no se puede porque hay una conspiración… y bueno, eso es una afirmación extraordinaria (que toda una disciplina científica se conspire para falsear resultados y censurar opiniones contrarias) que requiere evidencias extraordinarias (climategate, por ejemplo, NO es una evidencia extraordinaria). 

      Decís que el link de la NASA no significa nada poruqe "el clima siempre cambia". Eso es medio falso e irrelevante. Para empezar, no es tan cierto que el clima cambie constantemente. En general los procesos de retroalimentación hacen que esté estable a menos que algo lo fuerce. Pero más importante es que eso importa poco y nada. Lo que muestran las evidencias de ese sitio es que el clima actualmente está cambiando en cierta dirección y que el CO2 antropogénico es el principal responsable. Desestimar esos datos diciendo que "el clima siempre cambia" es como desestimar la enfermedad de un paciente diciendo "la gente siempre se enferema".

      El link que ponés sobre el Ártico me resulta incomprensible. ¿Cómo alguien que critica los gráficos de temperatura por no presentar suficientes años cree también que un aumento en la cantidad de hielo desde enero de 2009 (2 años) es suficiente para determinar una tendencia?

      Esto definitivamente es una falacia: el gás de efecto invernadero MAS IMPORTANTE es el vapor de agua

      Sí, el vapor de agua aumenta más la temperatura. El vapor de agua que aumenta en la atmósfera POR EL EFECTO DEL CO2. Es decir, el vapor de agua aumenta el efecto del CO2 y otros gases invernaderos. Pero el vapor de agua dura unos 10 días en la atmósfera (mientras que el CO2 está varias décadas) y no estamos emitiendo grandes cantidades de vapor de agua. En otras palabras, el vapor de agua tiene un efecto de retroalimentación positiva, no de forzamiento. 

      Está tenido en cuenta en los modelos del IPCC el gás invernadero más importante?

      Sí:

      3.4.2.1 Surface and Lower-Tropospheric Water Vapour – AR4 WGI Chapter 3: Observations: Surface and Atmospheric Climate Change

      2.3.7 Stratospheric Water Vapour – AR4 WGI Chapter 2: Changes in Atmospheric Constituents and in Radiative Forcing

      TS.3.1.3 Changes in the Water Cycle: Water Vapour, Clouds, Precipitation and Tropical Storms – AR4 WGI Technical Summary

      8.6.3.1 Water Vapour and Lapse Rate – AR4 WGI Chapter 8: Climate Models and their Evaluation

      Búsqueda "water vapour" en la web del IPCC

    • #30504
      N3RI
      Superadministrador

      admirable respuesta de Daneel.

       

      DrGen:

       

      "Sinceramente esperaba encontrar una actitud más escéptica sobre el tema en este foro"

       

      Pedís escepticismo, pero vos ya decidiste que es un "verso". No deberías ser escéptico vos también?

      Además, a qué debemos aplicar el escepticismo según vos? A la teoría científica con mayores evidencias y la que la enorme mayoría de la comunidad científica afirma o a la que un 2-3% (también de científicos) propone? Generalmente el escepticismo se aplica a la segunda.

       

      La ciencia sí se maneja por "consenso". En el contexto científico, esa palabra no significa que "el 97% de los científicos CREE (porque le sale de los cojones) que…", significa "el 97% de los expertos en el tema hicieron mediciones, experimentos y análisis y llegaron al mismo resultado". Basta una prueba (y que se pueda replicar) y el consenso no se tira, se cambia y se tiene un nuevo consenso (pero esa prueba debe ser replicable por ese 97%, sino dicha prueba no se acepta). O sea, sigue habiendo consenso. Lamentablemente, no ha habido dicha prueba.

      Vos cuando hablás de "consenso" lo querés hacer ver como que los científicos opinan lo que les sale del upite, cuando en realidad, los científicos opinan lo que obtienen como resultado de sus investigaciones. Nadie acá acepta el consenso según lo ves vos, acá aceptamos el consenso según te lo acabo de explicar. Hacés con la palabra "consenso" lo mismo que los creacionistas hacen con la palabra "teoría", exactamente lo mismo.

      La ciencia se maneja por consenso, y el consenso está basado en las pruebas.

       

      Y espero que con esto se acabe de una vez por todas el debate ridículo y semántico sobre la palabra "consenso". Empezá a reemplazarla por "resultados de los análisis científicos realizados".

       

      Otro problema que tenés es con el "cambio climático" y que "el clima siempre está cambiando".

      Para empezar, esa afirmación es irrelevante. Acá lo que importa es 

      1. la velocidad con la que está cambiando

      2. hacia dónde está cambiando

      3. cómo nos afecta a nosotros y a la vida en el planeta ese cambio.

       

      1. no importa si a lo largo de la historia del planeta hubo momentos más cálidos o con más CO2, lo que importa es que ahora el cambio es abrupto y acelerado (empezó en la industrialización, pero se acentuó en los últimos 50 años y los 50 que vienen). En esos otros momentos de la historia-prehistoria los cambios fueron graduales, duraron cientos, miles, millones de años. La diferencia es que las especies tuvieron tiempo de adaptarse (o se extinguieron, o sobrevivieron unos pocos). 

      2. el calentamiento producirá un montón de consecuencias: inundaciones, clima más violento, extinciones de especies enteras que no llegarán a adaptarse.

      3. si hoy mismo hubiera un "día después de mañana" (o la temperatura subiera a 50º) seguramente la raza humana no se extinguiría, pero de los miles de millones quedarían unos pocos miles. Y no queremos eso. Además, las bases del ecosistema mundial son muy sensibles a cambios climáticos, el plancton, el krill, los arrecifes, las abejas… con que una de esas especies se vea afectada, chau vida en la Tierra (o casi, pero que habría extinciones masivas, te lo garantizo)

       

      Y espero que con esto quede aclarado el tema de "el clima siempre cambia y eso no importa" y cómo nos puede afectar.

       

      El hielo en el ártico sí está bajando. Hermano, UNA fuente contra miles que dicen que sí? Y no sólo tenés gráficos estadísticos que lo demuestran, también tenés un montón de putas fotos satelitales!! Qué más necesitás para aceptarlo?

       

      El efecto del CO2 en el calentamiento global está totalmente probado, científicamente, químicamente, experimentalmente, en el laboratorio! Además, es absurdo resumir todo el problema a "el CO2", es uno de los principales factores, pero no es el único.

       

      Lo del vapor de agua ya te lo explicó bien arriba. Lo importante no es estudiar el efecto invernadero existente, sino el que se está produciendo EXTRA, como te dijeron, lo que tenés que considerar no es la retroalimentación, sino el forzamiento.

       

      Y eso es en lo último que me quiero centrar. No sé si lo ves completamente, pero el tema acá es ese forzamiento. El clima "no está cambiando todo el tiempo", al contrario, el clima es un sistema que se autoregula muy bien, tiene infinidad de mecanismos para regularse. Y las especies dependen de (y colaboran con) esa regulación. Y ese es el problema, si el cambio climático es muy abrupto (una o dos generaciones) podría causar (mejor dicho, causará) extinciones masivas. No es lo mismo que la temperatura aumente 3 grados en 20.0000 años o más, que en 75 años. Ni el Hombre es capaz de adaptarse tan rápido. El clima es un sistema que se auto equllibra y nosotros estamos rompiendo ese equilibrio. Y como si de una torre de cartas se tratara, el desequilibrio podría causar que todo el sistema colapse. 

       

      <meta content=»text/html; charset=utf-8″ http-equiv=»content-type» />

    • #30505

      Agrego algunos datos sobre el hielo en el Ártico:

       

      "Arctic sea ice extent averaged over January 2011 was 13.55 million square kilometers (5.23 million square miles). This was the lowest January ice extent recorded since satellite records began in 1979. It was 50,000 square kilometers (19,300 square miles) below the record low of 13.60 million square kilometers (5.25 million square miles), set in 2006, and 1.27 million square kilometers (490,000 square miles) below the 1979 to 2000 average."

      (fuente: Arctic Sea Ice News & Analysis- National Snow and Ice Data Center)

    • #30506
      Kenshin
      Miembro

      Me paso al Bando de los Negacionistas.

      Antes de seguir leyendo veanse este genial video de George Carlin (el de "holy Shith" de Zeitgeist 1)

      http://www.youtube.com/watch?v=P2nBU9UcRgM&feature=related

      "Un 97% de los cientificos afirman que el cambio climatico super-mortal que se biene es real".

      A- ¿Cuantos son esos "97%"?, es decir, dudo que un Quimico, un Climatologo y un Astrofisico, entiendan en lo mas minimo la ciencia de sus pares y aun mas dudo que haya 30.000 cientificos que se dediquen durante decadas (eso dura uno de estos estudios) a estudiar el clima; sin mencionar lo dificil que les resultaria conseguir una beca por parte del gobierno. Seamos realistas, a nadie le interesa estudiar el clima y por ende debe de ser una de las carreras menos seguidas y menos subencionadas. Asi que no me vengan con que el "97% de los cientificos estan de acuerdo" cuando no hay mas que 20 o 30 expertos en el tema en todo el mundo. De hecho hay un solo estudio metodico, y que durante 30 años que hizo un yanqui (ese que aparece en la pelicula de Al Gore, sorry no me acuerdo como se llama). Tambien estaba ese otro estudio que hizo Carl Sagan cuenta… si tomamos en cuenta que el mismo es de Venus donde la atmosfera es 90 veces mas densa y esta hecha de acido sulfurico… la verdad no se parece mucho a la nuestra.

       

      En fin, no soy un "experto" pero solo quiero mostrar que los dichosos estudios no son TAN concluyentes y absolutos.

      PD: y no hay 30.000 expertos climatologos que abalen dicho cambio. Porque no hay 30.000 climatologos, ni 3.000 ni 300… ya les digo, a nadie le interesa tanto el tema como para clavarse 30 años midiendo el CO2

       

      En cuanto a las mediciones… no son tan cientificas como parecen. Pongamos por ejemplo que durante 30 años se toman muestras del aire en el mismo lugar… ¿y si la actividad industrial se desarrolla fuertemente en las cercanias? ¿eso no influiria? ¿y si durante un par de años hay seguia e incendios forestales o por el contrario abundantes lluvias (las cuales contrarestan el CO2, por eso es tan limpio el aire despues de una tormenta)? ¿Los distintos tipos de cultivos influyen en la muestra?… y un millon de variables mas…

      PD2: cuando nuestro amigo Carlitos Sagan hizo Cosmos existia el riesgo real de que nos aniquilaramos con armas nucleares; de hecho "el cambio climatico" era solo una supocicion de la epoca que poca importancia tenia ¿que importa que en 100 años nos tape el mar si en 10 años podemos dejar de existir?. Ademas el se referia al impacto que los incendios masivos producto del MAD (mutual asegurated destruction) tendrian sobre la atmosfera, riesgo muy real con 60.000 armas atomicas en el mundo.

      PD3: a finales de los 80 mientras caia la URSS Carlitos Sagan predijo como el mejor de los astrologos que el incendio de campos petroliferos en Kuwait crearia un "invierno nuclear" sobre la India matando a 1.000 millones de personas de hambre y creando una nueva guerra mundial… le erro como los mejores astrologos. Los campos ardieron y aunque huvo mucha contaminacion… ni cerca estubieron del Badabum final. Despues de eso se dedico a escribir libros pues su carrera politica se habia terminado.

      Hay tienen su "cambio climatico super-mortal"

      PD4: Ya hubieron muchos cambios climaticos repentinos en apenas los ultimos 2.000 años; como el enfriamiento global de 450-480 que termino con los Romanos y obligo a los Galos invadir la peninsula italiana porque se morian de hambre y frio en el norte. El verano medieval entre 900 y 1350 aprox seguido de otro periodo inusualmente frio para finalmente entrar en otro periodo de calor en 1850 aprox.

    • #30507

      A- ¿Cuantos son esos "97%"?, es decir, dudo que un Quimico, un Climatologo y un Astrofisico, entiendan en lo mas minimo la ciencia de sus pares y aun mas dudo que haya 30.000 cientificos que se dediquen durante decadas (eso dura uno de estos estudios) a estudiar el clima; sin mencionar lo dificil que les resultaria conseguir una beca por parte del gobierno

      La figura del 97% es de los climatólogos activamente publicando en revistas con revisión por pares. Podés ver este estudio o este otro [pdf]. Y sí, los que se encargan de la climatología están toda su carrera esutiando eso. Igual que los físicos, biólogos, etc… 

      ¿y si la actividad industrial se desarrolla fuertemente en las cercanias? ¿eso no influiria? ¿y si durante un par de años hay seguia e incendios forestales o por el contrario abundantes lluvias (las cuales contrarestan el CO2, por eso es tan limpio el aire despues de una tormenta)? ¿Los distintos tipos de cultivos influyen en la muestra?… y un millon de variables mas…

      Seguro, y los cientícicos controlan los datos para eliminar el efecto de esas variables… igual que se hace en todas las demás disciplinas. 

       

      Estás muy poco informado sobre el tema y cuando decís que "no sos un experto" no sabés cuanta razón tenés. No estoy haciendo un ad hominem ya que no digo que lo que decís sea falso por tu falta de calificaciones; lo que estoy diciendo es que te falta muchísima información y desestimar un concenso cientíico (tanto de personas como de evidencia) en esas condiciones es, en mi opinión, una muestra de la más grande de las arrogancias. Ponés objeciones que francamente son ridículas (la lluvia no quita el CO2 de la atmósfera, por ejemplo) y cuestiones brutalmente obvias (como el tema de la isla de calor, que es conocido por todos los científicos en el medio). En vez de tirar esas preguntas retóricas al aire, te recomiendo investigar si ya fueron respondidas o no. Un buen lugar para empezar es el informe de síntesis del IPCC, o esta lista de argumentos

    • #30508
      1nfest
      Miembro


      @DrGEN

      2. Me referia a los gases invernaderos que la humanidad produce, principalmente Metano Y Dioxido de carbono. 

      El vapor de agua en la atmosfera esta en equilibrio con el agua liquida de los oceanos y otras fuentes y su concentracion depende de la temperatura principalmente. 

       El Dioxido de carbono esta en equilibrio solo con el dioxido de carbono solubilizado en los oceanos, actualmente el oceano ya no tiene capacidad de regular las cantidades ingentes de dioxido de carbono que se liberan a la atmosfera.

      La concentracion de metano practicamente no esta regulada por nada y se queda en la atmosfera hasta que se degrada a dioxido de carbono.

      3. Nuevamente, ya que estamos hablando de el calentamiento global antropocentrico hacia referencia a los gases que nosotros liberamos mas activamente.

      En ninguna parte negue o afirme que el vapor de agua contribuyera al efecto invernadero. Por otro lado lo de la variacion de los niveles de dioxido de carbono y metano salio de aca. Nature 364 407-412 (29 July 1993)

    • #30509
      Kenshin
      Miembro

      Es posible que tengas razon, pero la realidad es que todos los estudios que eh leido, comienzan "buscando las causas del calentamiento global" es decir, ninguno se plantea si verdaderamente "existe" dicho calentamiento y no es en cambio un proceso natural de "rebote" de la temperatura en el cual la misma occila constantemente.

      Por otro lado existen otras causas que no estan directamente relacionada con la atmosfera y que son mucho mas abarcativas que un poco de humo. Me refiero a la siembra masiva. En los ultimos 200 años pasamos de 700 millones de habitantes a 7000 lo cual significa que las superficies cultivables tambien se multiplicaron por 10. El color negro absorme mucha mas luz y genera mucho mas calor que por ejemplo el verde. Esto podria sugerir que las millones de hectareas deforestadas y listas para la siembra actuaran como "islas de calor" gigantes aumentando ligeramente la temperatura mundial. Ningun experimento climatico toma en cuenta este factor. Por otro lado ¿Como podria ser medido? Tambien esta el oscurecimiento de los mares y rios causado por la sobresiembra; la irrigacion y la deforestacion añaden tierra y cenizas al agua de rios que terminan en el mar oscureciendolos ¿otra posible causa no medible?; ademas incluso si lo midieramos, terminar con la industria del carbon es mucho mas facil que terminar con la industria alimenticia… Y la poblacion sigue creciendo.

       

      No en serio no busco ser arrogante… solo esceptico. Ademas segun los propios estudios ya mencionados:

      A_ "Con las mejores intenciones de los paises industriales" las cuales ovbiamente no tienen, les llevaria al menos 40 o 50 años reemplazar el sistema electrico actual por uno mas eficiente y "verde"; eso sin mencionar un crecimento CERO en los proximos 40 o 50 años y todos sabemos que el pueblo sigue creciendo.

      B_No existen alternativas viables al Carbon. El Uranio se va a agotar en 30 o 40 años, los paneles solares son inutiles en Finlandia, Noruega, Canada y el resto de los paises que se ubiquen cerca o por encima del circulo polar, Las turbinas eolicas necesitan muchisimo terreno para ser hubicadas y encima con tantas partes moviles, su mantenimiento es costosisimo y ademas requiere una industria pesada descomunal (la cual, usaria carbon para fabricar "la solucion verde").

      C_El Carbon seguira siendo "lo mas barato" durante muchos siglos mas (al menos hasta que se resuelvan los inconvenientes generados por las perforaciones geoterminas, como los terremotos y la inprevisibilidad de la presion). En cuanto a la hipotetica "energia de fusion" ¿cuantos paises podran costear una de esas usinas? y ademas ¿si fallara la usina no quedaria todo el sistema sin energia? recordemos que estas no pueden producir poca energia, sino muchisima.

      D_ Y la mas importante de todas; a la tierra le llevara al menos 500 años volver a los niveles "normales" de CO2; Si la produccion fuera CERO durante todo ese tiempo.

    • #30510

      Es posible que tengas razon, pero la realidad es que todos los estudios que eh leido, comienzan "buscando las causas del calentamiento global" es decir, ninguno se plantea si verdaderamente "existe" dicho calentamiento y no es en cambio un proceso natural de "rebote" de la temperatura en el cual la misma occila constantemente.

      Aún si fuera un "rebote", habría que buscar las causas. De todas formas no tiene mucha relevancia cómo empiecen los estudios. Hay múltiples lineas de evidencia que muestran que la temperatura global está aumentando y múltiples líneas de evidencia que muestran que se trata de un efecto invernadero antropogénico. El tema del efecto albedo (hacer más negra la superficie) se conoce y se mide; de hecho, hay personas que proponen pintar los techos de blanco en vez de colores obscuros (total, hay que pintarlos igual) para aumentar el albedo y disminuir el calentamiento. Pero las características del calentamiento señalan sin duda a un efecto invernadero. Por ejemplo, las noches se están calentando más que los días. Esto tendría que ser exactamente al revés si se tratara de las consecuencias del menor albedo. 

      Además, cualquier explicación alternativa tiene, sí o sí, que explicar por qué las cantidades masivas de CO2 antropogénicas no están teniendo un efecto invernadero. Eso es ciencia básica. 

       

      No en serio no busco ser arrogante… solo esceptico. Ademas segun los propios estudios ya mencionados:

      Lo sé, pero como dije anteriormente, "no dudés si no sabés dudar". El escepticismo no puede ser a priori y sin tener en cuenta lo que una comunidad de expertos dice sobre el tema. El escéptico debe ser informado por las mejores conclusiones científicas y a falta de ser un experto, el único recurso es confiar en el concenso robusto. Para llegar a una conclusión sólida hay que investigar y conocer muchísima información. La persona promedio y aún el aficionado es muy dificil que lo logre. Por suerte hay gente que sí hace eso y en este caso se llaman climatólogos. Ignorar lo que te están diciendo y basarse en el conocimiento admitidamente parcial, no me parece una buena forma de tomar decisiones.  Sería como si alguien que en su vida vió un motor a gasolina decidiera que, a pesar de lo que dicen todos los expertos, en realidad funciona con agua. 

      También es importante separar las consecuencias y políticas de la ciencia del cambio climático. Alternativas climáticas hay (hace poco salió un paper que calculó que en 50 años, si nos ponemos las pilas y con la tecnología actual, podríamos tener una economía mundial basada en energía solar y eólica) pero eso es irrelevante a la cuestión si el CC existe y si tiene orígen antropogénico.  

    • #30511
      DrGEN
      Miembro

      Daneel:

      Parece que estás confundiendo lo que es consenso con lo que es un paradigma.

      Estás tratando de ridiculizar una postura pero acabás ridiculizando la tuya.

      De ninguna manera (ni aun vos queriendo hacerlo ver de esa forma) estoy diciendo que haya que empezar a estudiar las ramas de la ciencia de cero.

      El fisiólogo no debe «confiar», simplemente parte de cosas probadas (con lo perfectible que luego pueda resultar eso).

      Decís:

      Quote:
      «Pensar que la ciencia se realiza un vacío es una visión reduccionista al extremo de dejar de ser útil.»

      Esa no fue mi postura en ningún momento. Si tratás de hacer ver eso, o no estás entendiendo o estás siendo poco leal en la discución. Si querés que mantengamos las posturas te pido que no vuelvas a poner en mi boca cosas que aparecen en tu cabeza.

      Decís:

      Quote:
      «si la comunidad de climatólogos que conocen el tema mucho más que cualquiera de nosotros acuerda en una serie de concluisones básicas, entonces la mejor que puede hacer el lego es apostar por el concenso»

      Volvés a equivocarte, no es la comunidad de climatólogos la que establece el consenso. Es el IPCC, formado por 2500 tipos, de los cuales sólo el 20% (dicho por el número 2 del IPCC) son climatólogos!!!

      El propio Pachauri (número 1 del IPCC) es cualquier cosa menos climatólogo.

      El consenso, reitero por enésima vez, es político.

      Entre los climatólogo, o para decirlo bien clarito, en la ciencia climatológica NO HAY UN CONSENSO sobre el origen y sobre el futuro de los cambios climáticos.

      Ese número que tantas veces repetis del «97%» también es una falacia, tan grande como eso otro que te gusta citar sobre la «falta de papers» con posturas diferentes a la oficial. Habría que hablar largo y tendido sobre cómo se logra ese «consenso» en las publicaciones, pero basta por ahora recordar que una encuesta entre climatólogos jóvenes (con una publicación), reveló que quienes tenían posturas contrarias a la «oficial» se limitaban a publicar por miedo a ser estigmatizados.

      Daneel, eso no es ciencia.

      Dije que el clima siempre cambió a lo que decís:

      Quote:
      «Eso es medio falso e irrelevante.»

      Ni es medio falso ni irrelevante. El clima simepre cambió y sería una gran noticia que el clima sea estable, nunca lo fue. Y si de esa afirmación depende la economía mundial, dificlimente sea «irrelevante».

      Decís: «En general los procesos de retroalimentación hacen que esté estable a menos que algo lo fuerce.»

      Esa es una afirmación extraordinaria, de la cual por lo menos esperaría una prueba del mismo calibre.

      No Daneel, justamente eso es lo que sostienen algunos de ese 3% de climatólogos escépticos: los cambios en el clima pueden explicarse perfectamente en base a los drivers internos del sistema (sin necesidad de recurrir al aporte antropogénico).

      Es bueno leer los papers «oficiales», pero leé de los otros que no muerden.

      Decís:

      Quote:
      «Lo que muestran las evidencias de ese sitio es que el clima actualmente está cambiando en cierta dirección y que el CO2 antropogénico es el principal responsable»

      La primera parte es cierta, la segunda carece de pruebas y no por mucho repetirla vas a validarla…

      Decís:

      Quote:
      «¿Cómo alguien que critica los gráficos de temperatura por no presentar suficientes años cree también que un aumento en la cantidad de hielo desde enero de 2009 (2 años) es suficiente para determinar una tendencia?»

      Desde enero de 2009, 2 años, se sigue diciendo que el hielo en el ártico es cada vez menor, y es mentira.

      Reitero: se sigue diciendo en los medios. Es parte de la propaganda oficial, que tan efectiva demuestra ser… incluso entre quienes «leen papers».

      En tu comentario hacia Kenshin decís:

      Quote:
      «Además, cualquier explicación alternativa tiene, sí o sí, que explicar por qué las cantidades masivas de CO2 antropogénicas no están teniendo un efecto invernadero. Eso es ciencia básica. «

      Parece que faltaste a la clase de ciencia básica…

      No Daneel, lo que afirman los escépticos es que el cambio se puede explicar de otras maneras y que justamente el CO2 NO explica el cambio.

      En ningún momento dicen que el CO2 no tenga efectos (que es lo que parece que creés vos por tu comentario) sino que ese efecto no alcanza para explicar lo observado.

      N3RI:

      Decís:

      Quote:
      «Pedís escepticismo, pero vos ya decidiste que es un «verso». No deberías ser escéptico vos también?»

      Che, pasate por la página del CEA (invitalo a Daneel de paso), donde dice «Preguntas Frecuentes»… leélo, mirá:

      Quote:
      «¿Qué es el escepticismo?

      El escepticismo consiste en no aceptar una afirmación hasta que tengamos las suficientes pruebas o evidencias para hacerlo.»

      Yo digo que es «verso» y es una opinión.

      Mi postura en cambio es escéptica con respecto a la ciencia climatológica y a sus afirmaciones, estoy abierto a que me prueben lo que dicen y listo.

      Acepto pruebas.

      Eso es escepticismo N3RI… a ver si lo aprendés: el escepticismo no es como vos creés, que «Generalmente el escepticismo se aplica a la segunda.» sino a cualquier afirmación que se haga sin pruebas.

      Decís:

      Quote:
      «La ciencia se maneja por consenso, y el consenso está basado en las pruebas.»

      Totalmente equivocado! La ciencia recurre al consenso por FALTA de pruebas! Una sola prueba y no se necesita «consenso».

      Es algo básico…

      Decís:

      Quote:
      «Otro problema que tenés es con el «cambio climático» y que «el clima siempre está cambiando».»

      Parece que los que tienen problemas con eso son uds.

      Para mi es lo más natural (y obvio).

      Decís:

      Quote:
      «El hielo en el ártico sí está bajando. Hermano, UNA fuente contra miles que dicen que sí? Y no sólo tenés gráficos estadísticos que lo demuestran, también tenés un montón de putas fotos satelitales!! Qué más necesitás para aceptarlo?»

      Bajá el tonito… después llorás.

      En cuanto a eso, ni siquiera Daneel cuestionó los datos porque la fuente es confiable.

      Decís:

      Quote:
      «El efecto del CO2 en el calentamiento global está totalmente probado»

      Dale, andá y ganale la apuesta a Spencer! :P (leer más abajo)

      Decís:

      Quote:
      «No es lo mismo que la temperatura aumente 3 grados en 20.0000 años o más, que en 75 años. Ni el Hombre es capaz de adaptarse tan rápido. El clima es un sistema que se auto equllibra y nosotros estamos rompiendo ese equilibrio.»

      Primero, en los últimos 75 años no subió 3º sino sólo 0.6ºC (en el último siglo).

      Segundo: el hombre no es capaz de adaptarse? podés probarlo?

      Tercero: nosotros estamos rompiendo el equilibiro? podés probarlo?

      1nfest:

      En tu primer comentario no aclaraste que eran los «gases invernaderos que la humanidad produce».

      Por eso mi comentario, sin la aclaración tu afirmación es falaz.

      Kenshin:

      No te gastes, Daneel no se va a cansar de repetir la perorata del «97%» (sin hacer caso a cómo se logra ese 97%)… es más cómodo.

      Daneel y N3RI:

      Ambos usaron el tema de la retroalimentación del clima pero lo hicieron de una manera harto simplista.

      Es cierto que existen mútilples retroalimentaciones, pero eso no hace que el clima sea «estable» (sé que no piensan eso, pero leído por distraídos así pareciera).

      Las oscilaciones dentro del clima y dentro de esas retroalimentaciones llevan a que tengamos variaciones enormes dentro del sistema.

      Acaso no escucharon hablar de las glaciaciones? y los períodos cálidos entre las mismas?

      Sí, hay oscilaciones y el clima se mantiene generalmente entre límites pero dentro de esos límites (tan amplios!) SIEMPRE cambia y ese cambio es el resultado de cientos (o miles) de variables.

      La ciencia recurre a los consensos cuando sólo se tienen evidencias y faltan pruebas.

      Una sola prueba de lo que afirma el consenso político sobre le cambio climático y el asunto estaría sanjado.

      No habría discución y la postura de ese 3% de climatólogos escépticos dejaría de ser un grano en el culo, de hecho, como ellos hacen ciencia, pasarían a estar con el mainstream.

      Pero… ese es el punto: no hay prueba de lo que afirman.

      Se la pasan hablando de que la «evidencia es irrefutable» pero a la hora de probarlo no pueden hacerlo.

      Roy Spencer es climatólogo respetadísimo a nivel mundial (no me vengan con que apelo a la autoridad, entiendan que ES una autoridad mucho más que cualquiera de nosotros para hablar de clima).

      Spencer lanzó una simple apuesta a ese 97% de climatólogos del mundo.

      Al igual que Randy con los homeópatas, el bueno de Roy dijo que él le pagaría X cantidad de dinero a cualquiera que le muestre un solo paper que demuestre que los cambios que se ven en el clima no son debidos a todos los drivers naturales del sistema.

      El que sepa un poco de ciencia, sabrá que no se está pidiendo la demostración de la «no existencia de algo» sino que lo único que pide Spencer es que se descarte la hipótesis nula a la hipótesis del cambio climático antropogénico.

      Quote:

      Hipótesis Nula:

      Indica que H_0, representa la hipótesis que mantendremos a no ser que los datos indiquen su falsedad

      Y el que sabe de ciencia, sabe también que sin descartar la hipótesis nula, no puedo afirmar que mi hipótesis sea «sin lugar a dudas la correcta».

      Saben quién cobró el dinero?

      Saben quién le tapó la boca a Spencer?

      Nadie…

      Así que no me jodan con eso de la «evidencia incontrastable», porque a la hora de probar algo no probaron nada.

      PD: Parece que en Argentina somos afortunados, un chabón me dijo que él era capaz de probarle a Randy que la homeopatía funciona… y acá tenemos a N3RI que le va a ganar a Spencer :P

    • #30512
      1nfest
      Miembro

      Algún día me voy a poner a leer estas 4 páginas. Algún día me voy a decidir en que lado estoy. Mientras tanto hoy leí esto y estoy del lado de DrGen, pero siempre me parece que la información que tengo es insuficiente.

    • #30513
      DrGEN
      Miembro

      Tarea para el hogar:

      Sería bueno que busquen la diferencia entre evidencia y prueba en ciencia…

      PS: El 97% de la humanidad cree en dios y seguramente tiene evidencias de su existencia… espero hagan la tarea sino van a empezar a hablar por acá de dios :P

    • #30514
      N3RI
      Superadministrador

      DrGen me gusta cómo dejás de lado y hacés oídos sordos a los argumentos más fuertes, y te concentrás en pequeñeces y definiciones (o interpretaciones) de dos o tres palabras.

      Ah, DrGen, dícese de una persona que cree sostiene a muerte que la NASA no fue parte de una conspiración que inventó el viaje del hombre a la luna (cosa a la que se dedican) pero que, en cambio cree sostiene la NASA es parte de una conspiración que inventó el cambio climático (cosa a la que no se dedican directamente, sino que sólo colaboran con otros investigadores aportándoles datos). Las fotos de la NASA del alunizaje no son falsas, pero los gráficos de la NASA sobre el cambio climático, sí son falsas. ¡Tiene mucho sentido!

      Yo ya estoy un poco perdido, he de confesar, con cuál es tu argumento. En algunas partes hablás como si creyeras que no hay cambio climático en lo absoluto, pero en otras parece que aceptás que sí hay cambio climático, pero no que sea en sentido hacia el calentamiento sino hacia «otra cosa» y finalmente, en otras partes pareciera que aceptás que existe un cambio climático significativo en los últimos años, pero que no aceptás que sea antropocéntrico (causado por nuestros pedos industriales).

      ¿Al final, en qué quedamos?

      ¿Me aclararías cuál es tu postura?

      1. no hay cambio climático (acelerado en menos de 100 años)

      2. hay cambio climático acelerado, pero no es calentamiento global

      3. hay cambio climático, es acelerado, pero no es causado por el hombre, sino por otra cosa (aclarar cuál sería esa otra cosa)

      4. hay cambio climático pero tardará miles o millones de años (este punto sería casi igual al punto 1)

      así sabemos qué parte te tenemos que aclarar :P (espero que sepas tolerar un poco de humor ad hominem, ad sense, ad nominis patris, filli, spiritus santus, admen)

    • #30515

      DrGEN

      Decís:

      Quote:
      «si la comunidad de climatólogos que conocen el tema mucho más que cualquiera de nosotros acuerda en una serie de concluisones básicas, entonces la mejor que puede hacer el lego es apostar por el concenso»

      Volvés a equivocarte, no es la comunidad de climatólogos la que establece el consenso. Es el IPCC, formado por 2500 tipos, de los cuales sólo el 20% (dicho por el número 2 del IPCC) son climatólogos!!!

      El propio Pachauri (número 1 del IPCC) es cualquier cosa menos climatólogo.

      El consenso, reitero por enésima vez, es político.

      El IPCC es sólo una organización que sistematiza los papers que andan dando vuelta y escriben un informe de síntesis. Podrías pensarlo como un grupo que realiza una revisión sistemática de la literatura. Además tiene 3 partes, la primera es la única que trata de las báses científicas, los que no son científicos son los que participan de los otros 2 reportes (que hablan de los impactos sociales y políticos, y de las formas de mitigación).

      Podemos ver que hay un consenso robusto porque más del 97% de los climatólgos activos están de acuerdo en las cosas básicas y las investigaciones se centran en otros aspectos de la disciplina. También podemos saberlo porque virtualmente la totalidad de los papers publicados lo apoyan implícita o explíticamente. Esto muestra que tu siguiente afirmación es simplemente falsa. (También vale aclarar que la totalidad de las organizaciones científicas están de acuerdo con los puntos básicos del consenso).

      Que quede bien claro: no existe una controversia sobre los puntos más básicos detrás del cambio climático. Los investigadores ya dejaron de discutir sobre eso hace rato.

      Quote:
      Ese número que tantas veces repetis del «97%» también es una falacia, tan grande como eso otro que te gusta citar sobre la «falta de papers» con posturas diferentes a la oficial. Habría que hablar largo y tendido sobre cómo se logra ese «consenso» en las publicaciones, pero basta por ahora recordar que una encuesta entre climatólogos jóvenes (con una publicación), reveló que quienes tenían posturas contrarias a la «oficial» se limitaban a publicar por miedo a ser estigmatizados.

      Supongo que debe ser parecido a lo que le pasaría a un biólogo que trata de publicar un paper apoyando el diseño inteligente, ¿no?

      Quote:
      «Eso es medio falso e irrelevante.»Ni es medio falso ni irrelevante. El clima simepre cambió y sería una gran noticia que el clima sea estable, nunca lo fue. Y si de esa afirmación depende la economía mundial, dificlimente sea «irrelevante».

      Nada depende de esa afirmación ya que, como dije, aún si fuera verdad o mentira, las consecuencias son las mismas: actualmente estamos en un proceso de calentamiento acelerado producido por el hombre.

      ¿Cuál es tu argument? El clima siempre cambió.. ¿y entonces? ¿qué?

      Quote:
      Decís: «En general los procesos de retroalimentación hacen que esté estable a menos que algo lo fuerce.»

      Esa es una afirmación extraordinaria, de la cual por lo menos esperaría una prueba del mismo calibre.

      No Daneel, justamente eso es lo que sostienen algunos de ese 3% de climatólogos escépticos: los cambios en el clima pueden explicarse perfectamente en base a los drivers internos del sistema (sin necesidad de recurrir al aporte antropogénico).

      Es bueno leer los papers «oficiales», pero leé de los otros que no muerden.

      Como dije, si tanta razón tienen, entonces cambiará el consenso entre los climatólogos. Pero no, quienes más saben sobre el tema no creen que los argumentos tengan mérito. Por otro lado, yo tampoco leí ninguno que lo tenga.

      Quote:
      Decís:

      «¿Cómo alguien que critica los gráficos de temperatura por no presentar suficientes años cree también que un aumento en la cantidad de hielo desde enero de 2009 (2 años) es suficiente para determinar una tendencia?»

      Desde enero de 2009, 2 años, se sigue diciendo que el hielo en el ártico es cada vez menor, y es mentira.

      Reitero: se sigue diciendo en los medios. Es parte de la propaganda oficial, que tan efectiva demuestra ser… incluso entre quienes «leen papers».

      No es mentira, la tendencia sigue ahí. Tanto la cobertura como el volumen de hielo siguen descendiendo. Es inevitable que haya años en los que se recupera un poco, pero eso no marca una tendencia.

      Si un paciente tiene una enfermedad terminal, el hecho que esté muriendo no es refutado porque durante una semana muestre signos de mejora.

      Quote:
      En tu comentario hacia Kenshin decís:

      Quote:
      «Además, cualquier explicación alternativa tiene, sí o sí, que explicar por qué las cantidades masivas de CO2 antropogénicas no están teniendo un efecto invernadero. Eso es ciencia básica. «

      Parece que faltaste a la clase de ciencia básica…

      No Daneel, lo que afirman los escépticos es que el cambio se puede explicar de otras maneras y que justamente el CO2 NO explica el cambio.

      En ningún momento dicen que el CO2 no tenga efectos (que es lo que parece que creés vos por tu comentario) sino que ese efecto no alcanza para explicar lo observado.

      Ajá. «Tenemos influencia pero no la principal». Pues entonces tienen que probarlo (en revistas con revisión por pares, no es sitios web, libros o documentales). Todo lo que sabemos indica que el ser humano es el principal responsable y todos los mecanismos alternativos que leí son o mínimos (influencia solar) o incompatibles con las observaciones (reducción del albedo).

      Y no sólo las emisiones antropogénicas bastan para explicar el aumento del CO2, sino que el aumento observado sólo explica un 55% de las emisiones. Es decir, estamos tirando mucho más CO2 del que observamos. Esto se explica por los procesos de absorción de los océanos que, como consecuencia, muestran una mayor acidificación (y los corales, sufren)

      Quote:
      Decís:

      «La ciencia se maneja por consenso, y el consenso está basado en las pruebas.»

      Totalmente equivocado! La ciencia recurre al consenso por FALTA de pruebas! Una sola prueba y no se necesita «consenso».

      Es algo básico…

      Claro, osea que casi la totalidad de los papres publicados apoyan la posición de consenso, casi la totalidad de los climatólogos aceptan los puntos básicos del consenso, la totalidad de las agrupaciones científicas de nivel nacional y para arriba aceptan los puntos básicos del consenso… Afirmás que no es porque toda (o la gran mayoría) la evidencia apunte hacia una misma respuesta, ¿entonces por qué es?

      Quote:
      Decís:

      «No es lo mismo que la temperatura aumente 3 grados en 20.0000 años o más, que en 75 años. Ni el Hombre es capaz de adaptarse tan rápido. El clima es un sistema que se auto equllibra y nosotros estamos rompiendo ese equilibrio.»

      Primero, en los últimos 75 años no subió 3º sino sólo 0.6ºC (en el último siglo).

      Segundo: el hombre no es capaz de adaptarse? podés probarlo?

      Tercero: nosotros estamos rompiendo el equilibiro? podés probarlo?

      Si sigue el negacionismo el hombre, más allá de si sea capaz o no de adaptarse, directamente no va a hacerlo. De todas formas, creo que él se refería a la adaptación por evolución biológica.

      Quote:
      Kenshin:

      No te gastes, Daneel no se va a cansar de repetir la perorata del «97%» (sin hacer caso a cómo se logra ese 97%)… es más cómodo.

      Decinos, entonces. ¿Cómo es que se logra ese 97%? ¿Intimidan becarios? ¿Rompen piernas? ¿Sobornan gente?

      Lo que uds. están diciendo acá es que toda una disciplina científica está corrompida. ESA es una afirmación extraordinaria que debe ser demostrada con evidencias extraordinarias. Además, es una afirmación que vuelve tu posición inmune a la evidencia. Cualquier paper, investigación o dato puede ser desestimado de esa forma.

      Quote:
      Daneel y N3RI:

      Ambos usaron el tema de la retroalimentación del clima pero lo hicieron de una manera harto simplista.

      Es cierto que existen mútilples retroalimentaciones, pero eso no hace que el clima sea «estable» (sé que no piensan eso, pero leído por distraídos así pareciera).

      Las oscilaciones dentro del clima y dentro de esas retroalimentaciones llevan a que tengamos variaciones enormes dentro del sistema.

      Acaso no escucharon hablar de las glaciaciones? y los períodos cálidos entre las mismas?

      Sí, hay oscilaciones y el clima se mantiene generalmente entre límites pero dentro de esos límites (tan amplios!) SIEMPRE cambia y ese cambio es el resultado de cientos (o miles) de variables.

      Las glaciaciones son fruto de otros forzamientos. Actividad solar, cambios en la órbita terrestre, volcanismo que emite CO2… hay mil maneras por las que se puede afectar el sistema.

      De todas formas, no me voy a gastar en ese punto porque, como dije, es irrelevante a la cuestión de fondo.

      Quote:
      La ciencia recurre a los consensos cuando sólo se tienen evidencias y faltan pruebas.

      Una sola prueba de lo que afirma el consenso político sobre le cambio climático y el asunto estaría sanjado.

      El consenso no es político, es científico. No tiene que ver con votaciones, sino el arribo a un conjunto de conclusiones básicas que explican las observaciones de la mejor manera.

      Quote:
      Se la pasan hablando de que la «evidencia es irrefutable» pero a la hora de probarlo no pueden hacerlo.

      Me lo pruebas y me lo sostienes! Las evidencias son amplísimas, desde la caracterísicas del calentamiento que señalan indudablemente a un efecto invernadero a la relación isotópica de 12C y 13C que apunta a la quema de combustibles fósiles como la fuente del carbono extra observado en la atmósfera. El cambio climático es innegable y también lo es la huella humana en el mismo.

      Quote:
      Roy Spencer es climatólogo respetadísimo a nivel mundial (no me vengan con que apelo a la autoridad, entiendan que ES una autoridad mucho más que cualquiera de nosotros para hablar de clima).

      Spencer lanzó una simple apuesta a ese 97% de climatólogos del mundo.

      Al igual que Randy con los homeópatas, el bueno de Roy dijo que él le pagaría X cantidad de dinero a cualquiera que le muestre un solo paper que demuestre que los cambios que se ven en el clima no son debidos a todos los drivers naturales del sistema.

      Eso es una falacia. NADA en la ciencia se demuestra mediante un solo paper. Cada trabajo va eliminando hipótesis alternativas y dando más credibilidad a una conclusión. Ningún paper es perfecto o está libre de crítica, pero en su conjunto señalan una explicación como la única razonable.

      Por otro lado, cualquiera con dos dedos de racionalización puede inventar hipótesis ad hoc para evitar «descartar la hipótesis nula».

      Abby dijo:

      Algún día me voy a poner a leer estas 4 páginas. Algún día me voy a decidir en que lado estoy. Mientras tanto hoy leí esto y estoy del lado de DrGen, pero siempre me parece que la información que tengo es insuficiente.

      ¿En serio me decís? Justo hoy voy a publicar en mi blog una crítica a ese artículo. Leelo de nuevo y escribite en un papel qué evidencias y argumentos utiliza para refutar los puntos que pone al principio. Vas a ver que brillan por su ausencia y lo único que hay son argumentos políticos y críticas a Al Gore… También fijate sus fuentes (listadas como pies de página) y anotate cuántos papers cita.

      EDIT: Acá está.

    • #30516
      Ernesto Garcia
      Participante

      Soy nuevo en este debate. Por años he debatido y refutado negacionistas de toda clase (de la evolución, del Holocausto, de la relación causal VIH-SIDA, del alunizaje del Apolo XI y un largo etcétera) pero hasta ahora nunca le había prestado demasiada atención a la controversia sobre el cambio climático. Provisionalmente y debido a los argumentos que encontré en este sitio he adoptado el consenso científico actual de que efectivamente existe un cambio climático y se debe a la actividad de nuestra especie. Mantengo sin embargo una mente abierta para considerar evidencia contraria, que en mi humilde opinión todavía no ha sido presentada.

    • #30517
      DrGEN
      Miembro

      N3RI:

      ¿Cuáles son los «argumentos más fuertes»? o.O

      Daneel:

      Leí tu ladrillo y no encontré nada interesante para rebatir… de hecho, marcame alguno de tus argumentos porque no los pude encontrar.

      PS1: Leyeron mi comentario?

      PS2: Ah… «El cambio climático es innegable y también lo es la huella humana en el mismo.» y lo «probás» con un par de post en tu blog… contundente!

    • #30518
      DrGEN
      Miembro

      N3RI y Daneel:

      En serio… lean la diferencia entre evidencia y prueba en ciencia.

      Luego de hacer eso, y sólo luego de eso, pónganse a pensar si la postura oficial del IPCC merece la pena ser analizada escépticamente (o sea, faltan pruebas).

      Después vengan y hablamos…

    • #30519

      Te lo resumo:

      1. El IPCC no es el que ‘dicatamina’ el consenso. Sabemos que hay un consenso porque virtualmente todos los expertos en una disciplina están de acuerdo en un conjunto de puntos básicos. También porque virtualmente todos los papers publicados en revistas con revisión por pares apoyan explícita o implícitamente el consenso. El consenso no es por votación. Es falso que no haya «consenso sobre el origen y sobre el futuro de los cambios climáticos»
      2. Si tal abrumador consenso no es por la evidencia, ¿por qué es? ¿Cómo se llega a ese 97%? ¿Rompiendo piernas? ¿Amenazando becarios?
      3. Tu posición está completamente aislada de la realidad. Cualquier dato que la contradiga puede ser negado en base a la supuesta corrupción de toda una disciplina científica y, no olvidemos, la totalidad de las academias de ciencias.
      4. Si los ‘disidentes’ quieren cambiar el consenso, deben hacerlo probando su punto en publicaciones y papers en revistas de revisión por pares, no en documentales o en the internets.
      5. No basta 1 (una) sola pieza de evidencia para refutar el conocimiento sólido de décadas de estudio. Para rechazar una teoría o modelo la evidencia en su contra tiene que ser, como mínimo, tan fuerte como la evidencia a su favor.
      6. Es irrelevante que el clima haya cambiado o no. ¿Cuál es tu argumento? El clima siempre cambió… ¿Y entonces?
      7. El ‘desafío’ ese es una falacia ya que nada en ciencia de demuestra con un sólo paper. Por otro lado, cualquiera con una mediana capacidad de racionalización puede inventar excusas ad hoc para negar los resultados de cualquier investigación.
      8. La evidencia que te presento son mis posts, sino que en ese artículo está la evidencia (con las fuentes correspondientes). Por otro lado vos símplemente afirmás que no haya pruebas…
      9. Como colorario: el aumento de la concentración de CO2 atmosférico no sólo es explicada en su totalidad por las emisiones antropogénicas, sino que éste sólo explica el 55%. Es decir, un 45% de nuestras emisiones son absorbidas por procesos naturales. Y eso se puede ver, por ejemplo, en la acidificación de los océanos.

    • #30520
      DrGEN
      Miembro
      Quote:
      La evidencia que te presento son mis posts, sino que en ese artículo está la evidencia (con las fuentes correspondientes). Por otro lado vos símplemente afirmás que no haya pruebas…

      Seguís si hacer la tarea… «busque la diferencia entre evidencia y prueba en ciencia».

      Después, sólo después, si aun no entendés la postura escéptica de quienes dudan del «consenso», paso a contestarte esos 9 puntos…

    • #30521

      La «prueba» sólo existe en matemática. En ciencia hay que conformarse con evidencias.

    • #30522
      DrGEN
      Miembro

      Daneel:

      ¿Querés decís que no hay forma de probar que dos cuerpos con diferente peso caigan a la misma velocidad?

      ¿Que no hay forma de probar que la molécula de agua esté formada por dos átomos de hidrógeno y uno de oxígeno? jajaja…

      Ahora queda todo más claro.

      No más preguntas su señoría…

    • #30523
      Julian R
      Miembro

      No, no lo podes probar, tenes evidencia que sugiere que es asi; del mismo modo el dia de mañana podes tener evidencia que sugiera que sea de otro modo.

    • #30524

      Julian R dijo:

      No, no lo podes probar, tenes evidencia que sugiere que es asi; del mismo modo el dia de mañana podes tener evidencia que sugiera que sea de otro modo.

      +1

      Claro que la evidencia empírica y teórica es tan basta para los dos ejemplos que pusiste, que sería absurdo que fuera de otra manera. La cantidad de evidencia y datos contradictorios que debería haber para concluir que la molécula de agua no está formada por dos átomos de hidrógeno y uno de oxígeno debería ser al menos tan grande como la evidencia a su favor (virtualmente toda la química, la física, la biología, etc…)

    • #30525
      DrGEN
      Miembro

      Julian y Daneel:

      Me temo que están ambos equivocados.

      Claro que se puede probar.

      De hecho Galileo probó lo primero en la Torre de Pisa (y se puede probar la velocidad de la luz y miles de cosas más).

      Es curioso, yo díría hasta irónico, el hecho de que adhieran al escepticismo radical extremo en el caso de hablar de pruebas en ciencia (no animándose ni siquiera a usar el término) pero cuando se trata del «calentamiento global antropogénico» suspenden el juicio y adhieren cual fieles religiosos…

      En mi barrio, que es un barrio muy culto, a eso le llaman doble standard.

      En el barrio vecino (que es más culto) le llaman disonancia congnitiva…

    • #30526
      N3RI
      Superadministrador

      DrGen, frenaste total y abruptamente el debate con este caprichito de la definición de dos palabras, por qué no das la definición que querés que te digan y así se puede continuar con el debate.

      Y ojalá la diferencia que querés resaltar esté grabada en piedra en la biblia científica, porque sino no entiendo que le des tanta transcendencia como para frenar el debate por eso.

      Ah, y aclarame mi duda de qué tipo de negacionista sos

    • #30527
      DrGEN
      Miembro

      N3RI:

      Aclarame así podemos seguir:

      ¿Qué tipo de alarmista sos?

      a.- Ignorante

      b.- Crédulo

      c.- Idiota

      d.- Troll

      e.- Todas las anteriores

    • #30528
      DrGEN
      Miembro

      Por si no entendieron a qué vino el anterior comentario, copio lo que dije al inicio de este hilo:

      Quote:
      Es decir, si empezamos diciendo que los que no «creen» (como si en ciencia se tratase de «creer»!!) en eso son conspiranoicos, luego vendrán adjetivos como «negacionistas».

      Saldremos los que pensamos diferente diciendo que los que sí creen (acá me parece más acertado el término) en la religión del Cambio Climático son unos «calentólogos», que son unos «alarmistas» y más..

      N3RI, como nos tiene acostumbrados, ataca diréctamente a sus rivales en los debates y haciendo caso omiso a lo citado, reiteró su pregunta usando «alarmistas» (y obviamente lo hiso concientemente).

      Por eso mi anterior respuesta.

    • #30529

      Ah, buenísimo. Quienes estén de acuerdo con el consenso sobre cambio climático son idiotas, crédulos, ignorantes o trolls y además lo creen de forma religiosa. Así sí que se puede tener una discusión racional.

      EDIT: no había leido el siguiente post (ninja), pero no podés criticar a N3RI por atacar a sus rivales e inmediatamente bajar a su nivel.

      Mientas tanto, el único dato sólido ofrecido para tratar de contradecir la existencia del cambio climático antropogénico fue que afirmar que las capas polares se están recuperando (que es falso). Como dice N3RI, pusiste un parate al debate con un tema de definiciones al que no le veo la relevancia y en vez de aclarar por tu cuenta a qué te referís con «prueba» y «evidencia», estás dando más vueltas que un perro antes de irse a dormir.

    • #30530
      Julian R
      Miembro

      DrGEN dijo:

      Julian y Daneel:

      Me temo que están ambos equivocados.

      Claro que se puede probar.

      De hecho Galileo probó lo primero en la Torre de Pisa (y se puede probar la velocidad de la luz y miles de cosas más).

      Es curioso, yo díría hasta irónico, el hecho de que adhieran al escepticismo radical extremo en el caso de hablar de pruebas en ciencia (no animándose ni siquiera a usar el término) pero cuando se trata del «calentamiento global antropogénico» suspenden el juicio y adhieren cual fieles religiosos…

      En mi barrio, que es un barrio muy culto, a eso le llaman doble standard.

      En el barrio vecino (que es más culto) le llaman disonancia congnitiva…

      Y que Galileo lo haya «probado», ¿descarta total y absolutamente la posibilidad, por remota que sea, de que el dia de mañana alguien haga lo mismo y los objetos caigan a velocidades diferentes o se queden flotando en el lugar? Si dicha situacion, por remotamente improbable que sea, ocurre, ¿donde nos metemos la «prueba» de Galileo? No se si me explico bien, a lo que voy es que ninguna «prueba» es definitoria y contundente, y pretender que algo esta «probado» como si no se pudiera decir nada mas al respecto es ir totalmente en contra, no solo del escepticisimo que (se supone) nos agrupa, sino tambien ir en contra de todo lo que se supone que es la ciencia.

      No tiene nada que ver con el «escepticisimo radical», la premisa fundamental de la ciencia, lo que la diferencia de la religión o las pseudociencias, es que todo conocimiento es provisorio y tentativo, siempre suceptible de cambiar o ser refutado. Pretender que algo queda «probado» desde hoy y para siempre solo porque tenga toneladas de evidencia a su favor es mezclar los tantos terriblemente.

      Y, de onda, baja un cambio. Que te pongas a revolear bardeadas y agites a diestra y siniestra no aporta nada. Y, ademas, estas metiendome en una bolsa a la cual no sabes si pertenezco o no. Eso, en mi barrio, le dicen estar razonando por fuera del recipiente.

    • #30531
      N3RI
      Superadministrador

      DrGEN dijo:

      N3RI:

      Aclarame así podemos seguir:

      ¿Qué tipo de alarmista sos?

      a.- Ignorante

      b.- Crédulo

      c.- Idiota

      d.- Troll

      e.- Todas las anteriores

      bueno, me parece que tu respuesta es un poco subida de tono como para ser la respuesta a la palabra «negacionista». No tenía idea que te afectara tanto como para desbocarte así. Nosotros no le damos tanto poder a esas palabras, ni vamos a llorar porque digas «alarmista».

      No me voy a poner a tu nivel y devolverte los insultos, sólo te voy a recordar dos cosas de la vida:

      1. los insultos dicen más de la persona que los utiliza que de aquel al que van dirigidos

      2. «no me insultes, pelotudo de mierda, forro hijo de puta, mogólico infradotado»… me hacés acordar a mi hermanita, ponete de acuerdo, o pedís que no te insulten o insultás vos… pero las dos cosas no. Si no querés que te digan negacionista, no les digas a los demás alarmistas. No te parece?

      3. en serio deberías replantearte tu forma de dirigirte a los demás si sos tan sensible con respecto a cómo los demás se dirigen a vos. Porque es cierto que yo te estoy jodiendo y chicaneando x diversión, pero los demás te trataron con un respeto absoluto y vos no les contestás de la misma forma.

      Tu reacción me parece exagerada, y es la segunda vez que respondo a tus insultos con una respuesta calmada y explicada, la próxima vez no será así. Comportate.

    • #30532
      N3RI
      Superadministrador

      volviendo al debate, contanos con qué parte de la teoría del cambio climático antropocéntrico no estás de acuerdo, así podemos saber en qué dirección seguir el debate, porque no es lo mismo que no creas que sea generado por el hombre que que directamente no creas que exista el calentamiento global.

    • #30533
      DrGEN
      Miembro

      Daneel:

      Quote:
      pero no podés criticar a N3RI por atacar a sus rivales e inmediatamente bajar a su nivel.

      Tenés razón, lamento haberme puesto a su nivel.

      Espero que ustedes se lo hagan notar así no tengo que volver a hacerlo.

      Quote:
      Mientas tanto, el único dato sólido ofrecido para tratar de contradecir la existencia del cambio climático antropogénico fue que afirmar que las capas polares se están recuperando

      No, volvés a equivocarte. Eso no fue el dato sólido para contradecir el AGW. Eso fue sólo para mostrar cómo se maneja la propaganda en el tema.

      Julián:

      Que Galileo lo haya probado no garantiza que siempre sea así, el conocimiento científico avanza mejorando los conocimientos previos. Es perfectible.

      Eso no quita que una prueba, provisoriamente, sea una prueba (y es diferente a evidencia).

      De onda, fijate quién comienza a «revolear bardeadas».

      Pedí de entrada (mirá mi primer comentario) que mantengan la postura si quieren debatir el tema.

      Que les haya hecho notar un punto incómodo no es mi problema. Si no estás en el grupo de los disonantes cognitivos, te excluyo.

      Por cierto de qué barrio sos?

      N3RI:

      Copio acá tu twit:

      Quote:
      «los ad hominems no son para ganar nada, sino para trollearte y hacerte saltar la térmica y que quedes tan mal parado como quedaste :P«

      Ese es tu modus operandi, siempre.

      Sería bueno que quienes no te conocen lo sepan, de tu propia boca.

    • #30534

      En vez de simplemente afirmar las cosas, podrías hacer avanzar el debate. ¿Qué datos sólidos presentaste para contradecir el AWG? ¿Cuál es, según vos, la diferencia entre prueba y evidencia?

      Así, en vez de seguir dando vueltas, podemos llegar a algún lado.

    • #30535
      N3RI
      Superadministrador

      DrGEN dijo:

      Si no estás en el grupo de los disonantes cognitivos, te excluyo.

      ¿Ves a lo que me refiero? Paraste todo el debate para quejarte de que te hayan llamado «negacionista» pero no tenés problema en llamar «alarmistas», «disonantes cognitivos» entre otras linduras, a los demás. Si sos tan sensible en cómo los demás se dirigen a vos… controlá más cómo te dirigís vos a los demás. Si vas a seguir bardeando a todos, después no te quejes, si te vas a quejar, entonces comportate.

      Y por favor, podés dejar de frenar el debate?

    • #30536

      Lástima que no siguió el debate. En cualquier caso, dejo este artículo:

      Reproducing the atmospheres of yesterday, today and tomorrow (in terms of oxygen, nitrogen and carbon dioxide levels), this tool was used to measure the effects of an oxidative stress (hydrogen peroxide, H2O2) on E. coli at different levels of atmospheric CO2 (from 40 to 1,000 ppm, the current atmospheric concentration being 389 ppm). The results show that an increase in CO2 is accompanied by an increase in various parameters, including cell death, DNA mutation frequency and the number of DNA lesions.

      Fuente: Predicted increase in atmospheric CO2 will directly affect living organisms: Carbon dioxide exacerbates oxygen toxicity

      Corto y en castellano: altas concentraciones de CO<sub>2</sub> parecen incrementar la toxicidad del oxígeno y producir daños en el ADN y otros problemas.

    • #30537
      leox
      Miembro

      niñas niñas, esperen a que llene la pelopincho de barro!

    • #30538
      N3RI
      Superadministrador

      La NASA advierte de que los Polos se deshacen más rápido de lo previsto

      http://www.elmundo.es/elmundo/2011/03/09/ciencia/1299634169.html

      http://www.nasa.gov/topics/earth/features/earth20110308.html

      lo dejo x acá, me voy a seguir estudiando, si alguien quiere pegarle un análisis al artículo (disculpen que sea de elmundo, pero lo vi en meneame) es todo suyo

    • #30539

      Lo había leído en SciAm, creo. Estos del IPCC se la pasan equivocándose…

    • #30540
      DrGEN
      Miembro

      N3RI:

      No más comida para vos…

      dont-feed-troll%203.jpg

      Don’t feed the troll!

      Daneel:

      Quote:
      Lástima que no siguió el debate.

      Yo diría que acá nunca hubo debate.

      El mismo podría darse en dos niveles: el académico, para el cual ninguno de los

      acá presentes está suficientemente calificado (corríjanme si hay algún climatólogo en la sala); y el «lego» por llamarlo de alguna manera.

      El primero de ellos, a pesar del supuesto consenso, existe y se mantiene casi constantemente entre los climatólogos.

      El segundo, el nuestro, acá no se pudo dar. De hecho, pareciera que vos Daneel no creés que existan escépticos en el tema: para vos son pseudoescépticos (usaste las comillas en tu último post, los llamaste «escépticos») o negacionistas.

      Si de entrada te parás creyendo que tenés la verdad, de manera casi dogmática,

      y negás la existencia genuina de los escépticos en el tema, difícilmente puedas participar de un debate.

      Al margen de todo esto, creo que el nivel del que podría darse acá, sería

      interesantísimo si analizamos no sólo la cuestión científica del tema (que tan

      bien lo hacen algunos científicos de ambos lados) sino las cuestiones políticas, económicas, las implicancias sociales, la cuestión mediática (y propagandística), el proceso de revisión por pares dentro de la climatología, los fraudes, etc, etc, etc.

      Aramis Latchinian (autor del libro «Globotomía») dijo:

      Quote:
      «Cuando surge una voz disidente, la primera reacción es aislarla, execrarla,

      cortarle el financiamiento. Hay montones de casos así con científicos de primer

      nivel en el mundo. Te pongo un ejemplo: el calentamiento global. Todo el mundo

      conoce una versión del calentamiento global. Hay otra, con fuertes fundamentos

      científicos, diciendo “señores, el calentamiento global por supuesto que

      existe, pero tiene un origen natural, lo muestran los ciclos de calentamiento y

      enfriamiento”.»

      Mi postura es la de genuino escepticismo en el tema, y ante las pruebas me rindo.

      Como dijo Keynes:

      Quote:
      “When the facts change, I change my mind. What do you do, sir?”

    • #30541
      DrGEN
      Miembro

      Daneel:

      Publicaste un artículo sobre los efectos del CO2 en la toxicidad celular por oxígeno.

      Interesantísimo artículo desde el punto de vista científico…

      Pero…

      ¿qué tiene que ver con el cambio climático (o el calentamiento global)?

      Te cuento, a mayor cantidad de radiación solar mayor es el daño del ADN y mayor incidencia de cancer de piel.

      Espero que el IPCC no lea esto… van a querer limitar el sol.

      Esto da pie al debate que te comentaba.

      Eso es un claro ejemplo de cómo se influencia la opinión pública. Se toma cualquier cosa que muestre algo negativo sobre el CO2 y se lo mezcla subrepticiamente con el cambio climático.

      Demonizar al CO2 nunca está de más.

      Dijo el Dr.Roy Spencer:

      Quote:
      «Carbon dioxide is necessary for life on Earth. That it has been so successfully demonized with so little hard evidence is truly a testament to the scientific illiteracy of modern society

      Se toma la ola de calor en Rusia se habla de los efectos catastróficos del

      cambio climático… no importa que luego tenga que salir el propio NOAA a decir

      que eso es variabilidad natural.

      Fuente:

      http://www.noaanews.noaa.gov/stories2011/20110309_russianheatwave.html

      Eso mismo que vos hechas en cara a los «escépticos» (no saber la diferencia

      entre clima y tiempo) es exactamente lo que pasa, pero del otro lado: cualquier

      evento meteorológico es tomado como «prueba» del cambio climático.

      Más adelante decís irónicamente:

      Quote:
      Estos del IPCC se la pasan equivocándose…

      Veamos…

      Los modelos utilizados por el IPCC toman una sensibilidad climática (aumento de

      temperatura al duplicar la concentración de CO2) de entre 3º y 4ºC.

      La sensibilidad climática establecida mediante observaciones por la NASA es de 1.6ºC!

      Y según algunos cálculos puede llegar a ser de sólo 0.45ºC.

      Paper: Lindzen and Choi

      http://www.drroyspencer.com/Lindzen-and-Choi-GRL-2009.pdf

      Sí, el IPCC la pasa equivocándose.

      Eso que parece una nimiedad es lo que hace que en los modelos del IPCC se predigan los catastróficos aumentos de temperatura, y en la realidad se vea que sólo fue de 0.6ºC en un siglo. Y de hecho un descenso en la temp. de 0.1ºC en los último 10 años.

      Piensen un poco eso: la sensibilidad climática según la NASA es de 1.6ºC. Esto

      significa que si duplicamos la concentración atmosférica de CO2 (de 390ppm

      actuales a 780ppm) sólo subiría 1.6ºC la temperatura global. Entiéndanlo, eso

      es duplicando! y para subir otro 1.6ºC tendríamos que llegar a 1560ppm!

      Esperemos que haya suficiente petróleo :P

      Entonces cabe aquí la pregunta: vale la pena limitar las emisiones de CO2 gastando trillones y trillones de dólareS y frenando el desarrollo económico del mundo?

      Se podrá invertir en mejores cosas?

      Por lo pronto (y hablando de la parte política), el Senado de EEUU votó que se deje de financiar al IPCC.

      El Dr Robert Austin de Princeton, testigo del Senado explicó que:

      Quote:
      El IPCC se había vuelto una institución políticamente controlada y no confiable.

      Fuente: http://www.climatesciencewatch.org/2011/02/19/house-votes-244-179-to-kill-u-s-funding-of-ipcc/

      Ahora hablando un poco de las cuestiones que rondan la investigación climatológica, en palabras de Dr.Roy Spencer:

      Quote:
      «The perpetual supply of climate change research money also biases them. Everyone in my business knows that as long as manmade climate change remains a serious threat, the money will continue to flow, and climate programs will continue to grow.»

      Le preguntaron a Gavin Schmidt (Climatólogo y creador de modelos para el GISS-NASA):

      Quote:
      -If 1998 is not exceeded in all global temperature indices by 2013, you’ll be worried about state of understanding?

      -Yes.

      Fuente: http://www.realclimate.org/index.php/archives/2007/12/a-barrier-to-understanding/comment-page-2/#comment-78146

      Estamos en 2011 y aparentemente tampoco este será el año… sólo le quedan 2

      años, ya veremos :P

      A otro climatólogo (Eduardo Zorita) se le preguntó:

      – ¿Cuantos años más de una tendencia cerca de plana más pueden pasar para invalidar al IPCC?

      Y contestó que a ojo, sin calcularlo, pensaba que con diez años bastaban. Y ya los tenemos…

      Pasemos a ciencia:

      El SOL:

      Para el IPCC y muchos climatólogo (incluidos algunos escépticos) la actividad solar (medida por el número de manchas solares) no tendría una influencia directa, pero…

      Un paper de Friis-Christensen and Lassen (1991, que ha sido citado más de 580 veces!) muestra cómo la temperatura del hemisferio norte se correlaciona estrechamente con los ciclos solares (11 años de duración) por los últimos 130 años.

      Fuente: http://www.sciencemag.org/content/254/5032/698.abstract

      También un paper con más de 600 referencias, concluye que sólo el forzamiento solar es responsable de la mitad del calentamiento desde 1860! El resto (0,27ºC) pudo ser causado por varios factores.

      Fuente: http://www.geo.umass.edu/faculty/bradley/lean1995.pdf

      Los Modelos:

      Un paper de David Douglass, John Christy, Benjamin Pearson y Fred Singer

      llamado: «A comparison of tropical temperature trends with model predictions»

      analiza las predicciones de huellas realizadas por 22 modelos dominados por gases de efecto invernadero y muestran cómo no concuerdan con los datos observados.

      Figura:

      http://2.bp.blogspot.com/_4ruQ7t4zrFA/R14vtJ7162I/AAAAAAAAAOc/-V_JV7Bpv8E/s1600-h/douglass-singer.jpg

      douglass-singer.jpg

      Como dijeron por ahí: los modelos le dijeron chau a la realidad ;)

      Paper: http://www.uah.edu/News/pdf/climatemodel.pdf

      Los HIELOS:

      Según los mapas PIPS2, el hielo grueso en el ártico se duplicó y el volúmen

      total de hielo muestra un aumento del 26% en los últimos 3 años.

      Reitero: Es mentira que siguen desapareciendo.

      Fuente: Polar Ice Prediction System

      http://www7320.nrlssc.navy.mil/pips2/index.html

      También, el hielo báltico es el mayor de los últimos 25 años.

      Fuente: Swedish Meteorological and Hydrological Institute

      http://www.thelocal.se/32262/20110225/

      Y la temperatura?

      La temperatura, en febrero por debajo de la media de los último 30 años:

      Anomalía de Temperatura Febrero-2011: -0.018ºC

      Esto es ligeramente inferior (aunque estadísticamente insignificante) que la media del período comprendido entre 1980 y 2010.

      Fuente: UAH AMSU

      http://vortex.nsstc.uah.edu/data/msu/t2lt/uahncdc.lt

      Imagen:

      UAH_LT_1979_thru_Feb_2011.gif

      *- Nuevo estudio de Naciones Unidas dice que la mitad del calentamiento observado en el ártico sería causado por el hollín y no por el CO2 ;)

      Fuente: http://notrickszone.com/2011/03/05/new-earth-moving-un-study-says-half-of-arctic-warming-caused-by-soot-and-not-co2/

      Paper: http://www.duh.de/uploads/media/black-carbon_220409.pdf

      Sensibilidad Climática: (Este es el gran punto!)

      Una nueva predicción calcula un efecto de calentamiento del CO2 siete veces inferior a la media del IPCC: 0,45ºC por doblar la cantidad de CO2, en vez de 3,2ºC.

      http://meetingorganizer.copernicus.org/EGU2011/EGU2011-4505-1.pdf

      Los modelos están equivocados:

      Contrariamente a los modelos del IPCC, se demostró que los hielos también crecen desde abajo.

      El científico Robin Bell de la Universidad de Columbia dijo que los modelos de capas de hielos (ice sheets) -que son relevantes para el comportamiento de Groenlandia y Antártida que subsecuentemente influyen en el nivel del mar, la retroalimentación del albedo por hielo y otras cosas- «están tan mal como modelos de autos sin ruedas» :P

      Fuente: http://www.sciencemag.org/content/early/2011/03/02/science.1200109

      James Hansen, Director del GISS (NASA) y considerado por muchos como el climatólogo más importante del mundo, en 1988 había predicho frente al Congreso de EEUU que la temperatura global aumentaría 2-4ºF por década (20-40ºF o 11-22ºC por siglo). De hecho, en el siglo XX sólo aumentó 0.6ºC.

      Bastante errado estaba el climatólogo más capo y sus modelos.

      Mencioné que la tendencia de la última década fue negativa?

      Fuente:

      https://stevengoddard.wordpress.com/2011/03/09/hansen-1986-2-to-4-degrees-warming-from-2001-2010/

      Algo interesante que leí (el artículo es extenso y desmonta uno a uno los puntos de alguien que trató de responder al desafío propuesto por el Dr.Roy Spencer):

      «We don’t understand the climate perfectly and changes of the temperature by

      +-0.5 °C per century remain beyond our precision and abilities.»

      Fuente: http://motls.blogspot.com/2011/02/why-dana1981-hasnt-proved-climate.html

      Para pensar:

      Si el aumento fue de sólo 0.6ºC en el siglo XX, pero nuestra precisión y

      habilidad para medir la temperatura es de +-0.5ºC.

      Entonces puede ser que ese aumento fuera de «tanto» como 1.1º o de tan sólo 0.1ºC

      Interesante:

      Aumento del CO2 Atmosférico, natural o antropogénico?

      Fuente: http://www.drroyspencer.com/2009/01/increasing-atmospheric-co2-manmade%E2%80%A6or-natural/

      Interesante 2:

      Entrevista a Freeman Dyson:

      http://www.independent.co.uk/environment/climate-change/letters-to-a-heretic-an-email-conversation-with-climate-change-sceptic-professor-freeman-dyson-2224912.html

      Correlación entre CO2 y temperatura?

      Veamos:

      CO2_450mm_yrs.jpg

      Afirmación: Los niveles del mar están aumentando… realmente?

      Fuente: http://notrickszone.com/2011/02/16/a-level-look-at-sea-levels/

      Y para finalizar…

      El CLIMAGATE:

      Daneel escribió sobre el tema, también lo hice yo (Climagate) (obviamente desde distintos

      lugares).

      Ahora bien, el post de Daneel se centraba en una serie de revisiones independientes que habían establecido que no existió fraude en el Climagate…

      Malas noticias: parece que esas revisiones fueron tan independientes como el Papa a la hora de criticar al Cristianismo.

      Quote:
      «A federal government inspector general has revealed prima facie proof that the so-called independent inquiries widely if implausibly described as clearing the ClimateGate principals of wrongdoing were, in fact, whitewashes. This has been confirmed to Senate offices. It will not be released to the public for some time because the investigation is ongoing.»

      Fuente: http://thegwpf.org/science-news/2604-penn-state-university-whitewashed-climategate.html

      Volviendo a lo que dijo Daneel:

      Quote:
      «Estos del IPCC se la pasan equivocándose… «

      Sí, y sino cometiendo fraudes ;)

      Que empiece el debate…

    • #30542

      DrGEN dijo:

      Daneel:

      Quote:
      Lástima que no siguió el debate.

      Yo diría que acá nunca hubo debate.

      El mismo podría darse en dos niveles: el académico, para el cual ninguno de los

      acá presentes está suficientemente calificado (corríjanme si hay algún climatólogo en la sala); y el «lego» por llamarlo de alguna manera.

      El primero de ellos, a pesar del supuesto consenso, existe y se mantiene casi constantemente entre los climatólogos.

      Seguís diciendo eso pero no presentás ni una razón para creerlo. A pesar de tus acusaciones de fraude y manipulación, la comunidad de climatólogos ya no discute sobre las cosas básicas. De la misma forma que la comunidad de biólogos no discute sobre las cuestiones básicas de la evolución.

      No reconocer esto es un fallo enorme ya que impide entender el contexto en el que se está discutiendo.

      El segundo, el nuestro, acá no se pudo dar. De hecho, pareciera que vos Daneel no creés que existan escépticos en el tema: para vos son pseudoescépticos (usaste las comillas en tu último post, los llamaste «escépticos») o negacionistas.

      Si de entrada te parás creyendo que tenés la verdad, de manera casi dogmática,

      y negás la existencia genuina de los escépticos en el tema, difícilmente puedas participar de un debate.

      Al contrario. Al creer que el consenso sobre el CCA se mantiene mediante fraude, intimidación y quién sabe qué otras artimañas, cualquier evidencia presentada puede ser desestimada.

      DrGEN dijo:

      Daneel:

      Publicaste un artículo sobre los efectos del CO2 en la toxicidad celular por oxígeno.

      Interesantísimo artículo desde el punto de vista científico…

      Pero…

      ¿qué tiene que ver con el cambio climático (o el calentamiento global)?

      Nada específico. Sólo sirve para entender un poco más las consecuencias del aumento de CO2. No tiene nada que ver con «demonizarlo» ni con la opinión pública. Y creo que la motivación para esa investigación es clara: hay un aumento del CO2, tenemos que saber qué consecuencias va a tener.

      Se toma la ola de calor en Rusia se habla de los efectos catastróficos del

      cambio climático… no importa que luego tenga que salir el propio NOAA a decir

      que eso es variabilidad natural.

      Fuente:

      http://www.noaanews.noaa.gov/stories2011/20110309_russianheatwave.html

      Ajá. Si el propio NOAA sale a decir que se trata de una variabilidad natural (y todos los climatólogos entienden esto perfectamente), ¿dónde está el catastrofismo? Sólo en los medios que, como sabemos, hacen lo mismo con toda noticia que encuentren.

      Más adelante decís irónicamente:

      Quote:
      Estos del IPCC se la pasan equivocándose…

      Veamos…

      Los modelos utilizados por el IPCC toman una sensibilidad climática (aumento de

      temperatura al duplicar la concentración de CO2) de entre 3º y 4ºC.

      La sensibilidad climática establecida mediante observaciones por la NASA es de 1.6ºC!

      Y según algunos cálculos puede llegar a ser de sólo 0.45ºC.

      Paper: Lindzen and Choi

      http://www.drroyspencer.com/Lindzen-and-Choi-GRL-2009.pdf

      Sí, el IPCC la pasa equivocándose.

      Pues Knutti y Hegerl (2008) hacen una revisión de varias líneas de evidencia y llegan a conclusiones simliares al IPCC (más o menos entre 2º y 4º)

      Climate_Sensitivity_500.jpg

      Claro que, de nuevo, esto es sabido por los climatólogos y el IPCC, que en tiene toda una sección destinada al tema de la sensibilidad climática. En su AR4 dice:

      Quote:
      The current generation of GCMs[5] covers a range of equilibrium climate sensitivity from 2.1°C to 4.4°C (with a mean value of 3.2°C; see Table 8.2 and Box 10.2

      (fuente)

      Quote:
      Piensen un poco eso: la sensibilidad climática según la NASA es de 1.6ºC. Esto

      significa que si duplicamos la concentración atmosférica de CO2 (de 390ppm

      actuales a 780ppm) sólo subiría 1.6ºC la temperatura global. Entiéndanlo, eso

      es duplicando! y para subir otro 1.6ºC tendríamos que llegar a 1560ppm!

      Esperemos que haya suficiente petróleo :P

      Entonces cabe aquí la pregunta: vale la pena limitar las emisiones de CO2 gastando trillones y trillones de dólareS y frenando el desarrollo económico del mundo?

      Se podrá invertir en mejores cosas?

      No pude encontrar la fuente original de lo que decís de la NASA así que mucho no puedo decir precisamente sobre eso. Lo que sí puedo señalar es, como ya hice, el consenso de la evidencia, y notar que resulta curioso que ahora sí confíes en la NASA.

      También que hables de los trillones de dólares necesarios sin referencia alguna cuando varios modelos económicos muestran que el coste de disminuir las emisiones es mínimo y en algunos casos, incluso es negativo.

      Quote:
      Por lo pronto (y hablando de la parte política), el Senado de EEUU votó que se deje de financiar al IPCC.

      El Dr Robert Austin de Princeton, testigo del Senado explicó que:

      Quote:
      El IPCC se había vuelto una institución políticamente controlada y no confiable.

      Fuente: http://www.climatesciencewatch.org/2011/02/19/house-votes-244-179-to-kill-u-s-funding-of-ipcc/

      Nada sorprendente, considerando que el senado yanki está controlado por los republicanos.

      Quote:
      Pasemos a ciencia:

      El SOL:

      La actividad solar no es suficiente para explicar el calentamiento observado, como vos bien citaste. Y como bien dice la NASA:

      Quote:
      Longer-term estimates of solar irradiance have been made using sunspot records and other so-called “proxy indicators,” such as the amount of carbon in tree rings. The most recent analyses of these proxies indicate that solar irradiance changes cannot plausibly account for more than 10 percent of the 20th century’s warming.1

      Quote:
      Los Modelos:

      Un paper de David Douglass, John Christy, Benjamin Pearson y Fred Singer

      llamado: «A comparison of tropical temperature trends with model predictions»

      analiza las predicciones de huellas realizadas por 22 modelos dominados por gases de efecto invernadero y muestran cómo no concuerdan con los datos observados.

      Figura:

      http://2.bp.blogspot.com/_4ruQ7t4zrFA/R14vtJ7162I/AAAAAAAAAOc/-V_JV7Bpv8E/s1600-h/douglass-singer.jpg

      douglass-singer.jpg

      Como dijeron por ahí: los modelos le dijeron chau a la realidad ;)

      Paper: http://www.uah.edu/News/pdf/climatemodel.pdf

      Obviamente una imagen así es imposible de interpretar sin un contexto adecuado y menos para alguien que no es experto. Sí puedo decir que las «huellas» del cambio climático son enteramente consistentes con un efecto invernadero. Y por si no confiás en mi artículo (que, en realidad, está basado en datos del NOAA), la NASA también

      Quote:
      Los HIELOS:

      Según los mapas PIPS2, el hielo grueso en el ártico se duplicó y el volúmen

      total de hielo muestra un aumento del 26% en los últimos 3 años.

      Reitero: Es mentira que siguen desapareciendo.

      Fuente: Polar Ice Prediction System

      http://www7320.nrlssc.navy.mil/pips2/index.html

      ¿3 años? ¿en serio? De nuevo, según la NASA:

      [img]i.imgur.com/AZDUy.gif[/img]

      Quote:
      Y la temperatura?

      La temperatura, en febrero por debajo de la media de los último 30 años:

      Anomalía de Temperatura Febrero-2011: -0.018ºC

      Esto es ligeramente inferior (aunque estadísticamente insignificante) que la media del período comprendido entre 1980 y 2010.

      Fuente: UAH AMSU

      http://vortex.nsstc.uah.edu/data/msu/t2lt/uahncdc.lt

      Imagen:

      UAH_LT_1979_thru_Feb_2011.gif

      Anteriormente te quejabas de quienes toman eventos aislados como evidencia de CG y ahora vos hacés precisamente eso. Por otro lado, decidite. ¿Hay calentamiento o no? Arriba culpás al Sol, pero acá negás directamente que haya calentamiento.

      [img]i.imgur.com/fcfZh.gif[/img]

      Nuevamente la fuente es la NASA

      Quote:
      Sensibilidad Climática: (Este es el gran punto!)

      Una nueva predicción calcula un efecto de calentamiento del CO2 siete veces inferior a la media del IPCC: 0,45ºC por doblar la cantidad de CO2, en vez de 3,2ºC.

      http://meetingorganizer.copernicus.org/EGU2011/EGU2011-4505-1.pdf

      Ehm… ¿cómo podés presentar esto como evidencia (un modelo) y en la siguiente oración afirmar que «los modelos están equivocados»? De nuevo, obviamente no tengo capacidad para analizar ese paper, pero sí puedo decir que se trataría de una excepción. Los demás modelos son consistentes en sus proyecciones y congruentes con las observaciones (de hecho, la temperatura está en los límites más altos del IPCC)

      Quote:
      Los modelos están equivocados:

      Contrariamente a los modelos del IPCC, se demostró que los hielos también crecen desde abajo.

      El científico Robin Bell de la Universidad de Columbia dijo que los modelos de capas de hielos (ice sheets) -que son relevantes para el comportamiento de Groenlandia y Antártida que subsecuentemente influyen en el nivel del mar, la retroalimentación del albedo por hielo y otras cosas- «están tan mal como modelos de autos sin ruedas» :P

      Fuente: http://www.sciencemag.org/content/early/2011/03/02/science.1200109

      El derretimiento de los hielos es una gran incertidumbre y es por eso que no fueron incluidos en las proyecciones para el aumento en el nivel del mar. Pero, más allá de cualquier crítica… hace unos días salía un estudio que confirmaba que los hielos están retrocediendo más rápido de lo proyectado.

      Quote:
      James Hansen, Director del GISS (NASA) y considerado por muchos como el climatólogo más importante del mundo, en 1988 había predicho frente al Congreso de EEUU que la temperatura global aumentaría 2-4ºF por década (20-40ºF o 11-22ºC por siglo). De hecho, en el siglo XX sólo aumentó 0.6ºC.

      Bastante errado estaba el climatólogo más capo y sus modelos.

      Mencioné que la tendencia de la última década fue negativa?

      Fuente:

      https://stevengoddard.wordpress.com/2011/03/09/hansen-1986-2-to-4-degrees-warming-from-2001-2010/

      Eso es porque hay procesos que «ocultan» la acumulación de energía. Por ejemplo, el océano está aumentando su temperatura (los 0,6ºC es la temperatura de la superficie) y acumulando calor.

      ….

      En fin. Son las 3 de la mañana y no pienso seguir con tu gish gallup. Confío en que las inconsistencias internas que señalé y los puntos que refuté con relativa facilidad basten. Ahora, si querés concentrarte en uno o dos puntos y, antes que nada, establecer tu posición con claridad (como ya mostré, primero decís que el sol causa el calentamiento y luego directamente lo negás).

    • #30543
      N3RI
      Superadministrador

      si, es importante que aclares tu posición drgen, para poder seguir con el debate.

    • #30544
      DrGEN
      Miembro

      Daneel:

      Quote:
      A pesar de tus acusaciones de fraude y manipulación, la comunidad de climatólogos ya no discute sobre las cosas básicas.

      No son mías las acusaciones. Te pasé la fuente que ignoraste… como todo lo que no conviene a tu postura.

      Quote:
      De la misma forma que la comunidad de biólogos no discute sobre las cuestiones básicas de la evolución.

      Comparar la climatología con la biología, o peor aun, las evidencias del cambio climático antropogénico con las evidencias de la teoría evolutiva simplemente deja en claro tu conocimiento en esos campos… o simplemente (que es lo que creo) lo hacés para justificar el «consenso». Pero vamos, ni vos te creés que se puedan comparar!

      Quote:
      resulta curioso que ahora sí confíes en la NASA.

      Jajaja, no es necesario «confiar», simplemente te muestro un dato de la NASA, si te molesta podés seguir ignorándolo…

      Quote:
      De nuevo, obviamente no tengo capacidad para analizar ese paper, pero sí puedo decir que se trataría de una excepción. Los demás modelos son consistentes en sus proyecciones y congruentes con las observaciones.

      Es bueno que lo reconozcas, sería bueno que también actúes en consecuencia.

      Hay gente que sí tiene la capacidad.

      Quote:
      En fin. Son las 3 de la mañana y no pienso seguir con tu gish gallup.

      Espero que sólo sea por la hora.

      Es tragicómico, te la pasás pidiendo pruebas (que obviamente el AGW no tiene) pero cuando alguien te los tira en la cara salís gritando «gish gallup! gish gallup!»…

      Quote:
      Confío en que las inconsistencias internas que señalé y los puntos que refuté con relativa facilidad basten.

      Jajaja, me sacaste una carcajada!

      Quote:
      Ahora, si querés concentrarte en uno o dos puntos y, antes que nada, establecer tu posición con claridad (como ya mostré, primero decís que el sol causa el calentamiento y luego directamente lo negás).

      Te volvés a equivocar, te mostré que el calentamiento no es como lo pronosticó el IPCC (pero no importa, ignorá esto también) y que gran parte de lo observado puede ser debido al Sol.

      No lo digo yo, lo dice gente que (a diferencia nuestra) sí tiene la capacidad.

      También está el modelo del Dr.Roy Spencer, quien propone un modelo basado en la formación y dinámica de nubes. Ya que «te gustan las nubes», tratá de leerla, es interesante.

      Este modelo sí explica lo observado y no contiene las contradicciones con los datos observados que sí tiene el modelo del CO2.

      Efectivamente, lo que está lleno de inconsistencias es el AGW.

      Por ejemplo, pronostica que el calentamiento de la atmósfera superior (por el efecto invernadero) debe ser mayor que el de la temperatura superficial de la tierra, pero la realidad muestra justamente lo opuesto!

      Los datos muestran que el aumento de la temperatura es mayor durante la noche que durante el día… cuando el modelo basado en CO2 pronosticaba justo lo opuesto. Por nombrar unas.

      El gran Rinze dijo sobre el confirmation bias:

      Quote:
      Recuerden los aciertos, olviden los fallos, aférrense a cualquier experiencia que confirme sus erróneas preconcepciones y recuerden que de una cantidad lo suficientemente grande de datos puede extraerse prácticamente cualquier patrón que nos convenza. Aquí lo tenemos: un método de razonamiento destinado a hacerles sentir en posesión de la razón absoluta pase lo que pase. Solamente hay que escoger las pruebas más favorables a nuestra postura y olvidar el resto.

      ¿qué más agregar? ;)

    • #30545
      DrGEN
      Miembro

      Por cierto, acá está la fuente: Sensibilidad climática de 1.64ºC según la NASA

      http://www.theregister.co.uk/2010/12/08/new_model_doubled_co2_sub_2_degrees_warming/

    • #30546

      DrGEN dijo:

      Daneel:

      Quote:
      A pesar de tus acusaciones de fraude y manipulación, la comunidad de climatólogos ya no discute sobre las cosas básicas.

      No son mías las acusaciones. Te pasé la fuente que ignoraste… como todo lo que no conviene a tu postura.

      ¿Qué fuente? ¿Que en un blog dice que que las revisiones independientes no eran realmente independientes? ¿Las 6 ó 7 (ya no me acuerdo exactamente cuantas fueron)? ¿Y eso es evidencia de una gran conspiración de climatólogos? (Porque, claro está, la mejor forma de hacerse millonario es siendo un investigación en climatología, no siendo dueño de una petrolera.. )

      Además, como vos bien aceptaste en un comentario en mi blog, aún cuando las acusaciones del climategate fueran ciertas, no harían mella alguna en nuestro entendimiento del CC. Éste se basa en los datos y conclusiones de una comunidad entera de expertos, no en los resultados de un laboratorio particular en Gran Bretaña.

      Quote:
      De la misma forma que la comunidad de biólogos no discute sobre las cuestiones básicas de la evolución.

      Comparar la climatología con la biología, o peor aun, las evidencias del cambio climático antropogénico con las evidencias de la teoría evolutiva simplemente deja en claro tu conocimiento en esos campos… o simplemente (que es lo que creo) lo hacés para justificar el «consenso». Pero vamos, ni vos te creés que se puedan comparar!

      Me gusta tu argumento circular: no hay consenso porque no hay pruebas, y no hay pruebas porque no hay consenso.

      Quote:
      resulta curioso que ahora sí confíes en la NASA.

      Jajaja, no es necesario «confiar», simplemente te muestro un dato de la NASA, si te molesta podés seguir ignorándolo…

      No lo ignoré sino que presenté el resto de las líneas de evidencia que muestran una sensibilidad climática de entre 2 y 4 ºC.

      Quote:
      De nuevo, obviamente no tengo capacidad para analizar ese paper, pero sí puedo decir que se trataría de una excepción. Los demás modelos son consistentes en sus proyecciones y congruentes con las observaciones.

      Es bueno que lo reconozcas, sería bueno que también actúes en consecuencia.

      Hay gente que sí tiene la capacidad.

      ¡Sí! Se llaman climatólogos y yo comparto lo que concluyó más del 97% de ellos (aunque sigas renegando, ese número sigue en pie). Es decir, yo sí actúo en consecuencia y confío en las personas que, en palabras de Doran et al, «entienden las sutilezas y bases científicas de las tendencias climáticas a largo plazo».

      Quote:
      En fin. Son las 3 de la mañana y no pienso seguir con tu gish gallup.

      Espero que sólo sea por la hora.

      Es tragicómico, te la pasás pidiendo pruebas (que obviamente el AGW no tiene) pero cuando alguien te los tira en la cara salís gritando «gish gallup! gish gallup!»…

      Es que vos parece que no conocés puntos medios. O cerrás todo debate con una discusión sobre definiciones (que, dicho sea de paso, todavía no aclaraste) o tirás 2 mil datos sólo medianamente relevantes dedicados a los detalles. Ninguna de las dos cosas puede conducir a una verdadera conversación. Para colmo, todavía no decís cuál es exactamente tu posición, lo único que hacés es tirar todos los palos que puedas, sin importar si se contradicen entre ellos.

      Quote:
      Confío en que las inconsistencias internas que señalé y los puntos que refuté con relativa facilidad basten.

      Jajaja, me sacaste una carcajada!

      Baja sensibilidad climática =/= el clima siempre cambió.

      Los modelos no sirven =/= Los modelos desmienten el CCA

      Usar variabilidad natural como evidencia a favor del CCA (eventos climáticos extremos) =/= Usar variabilidad natural como evidencia contra el CCA (1998 fue muy cálido, este febrero es más frío de lo normal, los últimos 3 años las capas polares ganaron hielo)

      Los climatólogos utilizan eventos climáticos extremos como evidencia a favor del CCA =/= Los climatólogos dicen que los eventos climáticos extremos no son evidencia a favor del CCA

      Hay un fraude global =/= Los datos de la NASA y las NU desmienten el CCA

      Es el Sol =/= El Sol sólo fue responsable de la mitad del calentamiento

      Es el Sol =/= No hay calentamiento

      Es el Sol =/= Hay más calentamiento durante la noche que durante día

      La mitad del calentamiento sería producido por hollín =/= No hay calentamiento antropogénico.

      Quote:
      Ahora, si querés concentrarte en uno o dos puntos y, antes que nada, establecer tu posición con claridad (como ya mostré, primero decís que el sol causa el calentamiento y luego directamente lo negás).

      Te volvés a equivocar, te mostré que el calentamiento no es como lo pronosticó el IPCC (pero no importa, ignorá esto también) y que gran parte de lo observado puede ser debido al Sol.

      No lo digo yo, lo dice gente que (a diferencia nuestra) sí tiene la capacidad.

      ipcc_2001_temp.gif

      Arctic_models_obs.gif

      También está el modelo del Dr.Roy Spencer, quien propone un modelo basado en la formación y dinámica de nubes. Ya que «te gustan las nubes», tratá de leerla, es interesante.

      Este modelo sí explica lo observado y no contiene las contradicciones con los datos observados que sí tiene el modelo del CO2.

      Las nubes no se sabe exactamente si son un feedback neto positivo o negativo, aunque varios estudios sugieren que es positivo (aumento en las nubes de baja altura)

      Efectivamente, lo que está lleno de inconsistencias es el AGW.

      Por ejemplo, pronostica que el calentamiento de la atmósfera superior (por el efecto invernadero) debe ser mayor que el de la temperatura superficial de la tierra, pero la realidad muestra justamente lo opuesto!

      Lo que se pronostica es que la tropósfera se va a calentar mientras que la estratósfera va a enfriarse… y eso es justamente lo observado. Observación, dicho sea de paso, completamente inconsistente con la idea de que el causante es el Sol. Por otro lado, hay más de 10 indicadores climáticos que demuestran que el calentamiento es producto de un efecto invernadero, incluyendo los inviernos calentándose más que los veranos, observaciones satelitales que muestran menos calor escapándose al espacio, observaciones terrestres de más calor volviendo a la superficie, la tropopausa bajando… (todo eso se puede encontrar en el State of the Climate 2009 del NOAA.

      Ah, hablabas de 1998.. NOAA – 2010 Tied with 1998 as Warmest Global Temperature on Record. Claro que, como gente ilustrada que somos, entendemos que los datos de un solo año no sirven para evaluar tendencias a largo plazo.

      Quote:
      Los datos muestran que el aumento de la temperatura es mayor durante la noche que durante el día… cuando el modelo basado en CO2 pronosticaba justo lo opuesto. Por nombrar unas.

      Mayor calentamiento durante la noche que el día no sólo es perfectamente consistente con un efecto invernadero, ¡sino que es inconsistente con la idea de que el Sol es la causa!

      Explico en palabra simples: si el aumento en la actividad solar fuera el responsable del aumento de la temperatura, entonces este aumento necesariamente debería darse durante el día. ¿Cómo hace el Sol para calentar durante la noche? Si, por el contrario, se trata de un efecto invernadero, entonces es durante la noche cuando más influiría el efecto de «frazada», impidiendo que el calor escape al espacio.

      El gran Rinze dijo sobre el confirmation bias:

      Quote:
      Recuerden los aciertos, olviden los fallos, aférrense a cualquier experiencia que confirme sus erróneas preconcepciones y recuerden que de una cantidad lo suficientemente grande de datos puede extraerse prácticamente cualquier patrón que nos convenza. Aquí lo tenemos: un método de razonamiento destinado a hacerles sentir en posesión de la razón absoluta pase lo que pase. Solamente hay que escoger las pruebas más favorables a nuestra postura y olvidar el resto.

      Verdaderamente. Se me ocurre un par de ejemplos: aceptar lo que opina un 3% de los expertos y no lo que dice un 97%; aceptar los resultados de 1 (un) modelo climático pero no los de todos los demás; hablar del aumento en el hielo de los últimos 3 años e ignorar la clara tendencia de retroceso no sólo de las capas polares sino de la enorme mayoría de los glaciares del mundo. Solo algunos ejemplos; notar que 1998 fue el año más caluroso (empatado con el 2010, recordemos), e ignorar que la última década fue la más calurosa registrada.

      Sólo algunos ejemplos.

      En fin, ¿vas a expecificar tu postura o no? Es un poco cansador no saber contra qué se está argumentando. ¿Hay calentamiento global? ¿Es antropogénico? ¿En qué proporción? ¿Los modelos sirven o no? ¿Por qué las cantidades masivas de gases de efecto invernadero (GEI) emitidas por el hombre NO estarían teniendo un efecto invernadero? ¿Por que el aumento observado de GEI no causaría un efecto invernadero?

      PD: Una de las razones por las cuales no hay que hacer Gish Gallup, es porque cuando te la responden (aunque sea parcialmente, como yo hice), tenés que responder a cada objeción. Hay varias refutaciones que quedaron sin refutar aunque estoy seguro que debés tener mucho que decir. Pero mejor así, presentá tu caso y centrémonos en un par de temas específicos pero básicos (sin perderse en los detalles como anteriormente).

    • #30547
      DrGEN
      Miembro

      Daneel:

      Quote:
      Me gusta tu argumento circular: no hay consenso porque no hay pruebas, y no hay pruebas porque no hay consenso.

      Eso lo estás diciendo vos.

      Te pedí que por favor no pongas en mi boca cosas que aparecen en tu cabeza.

      Podrías señalar en qué parte digo que «no hay pruebas porque no hay consenso»???

      Quote:
      Mayor calentamiento durante la noche que el día no sólo es perfectamente consistente con un efecto invernadero, ¡sino que es inconsistente con la idea de que el Sol es la causa!

      Es inconsistente con el efecto invernadero! Los climatólogos lo saben.

      Por otro lado no estoy diciendo que el Sol sea la causa. Leé bien, vos podés.

      Mostré estudios que relacionan ambas cosas y te comenté que hay modelos que hablan de otros drivers (nubes).

      Las inconsistencias y contradicciones que «refutaste»:

      Quote:
      Baja sensibilidad climática =/= el clima siempre cambió.

      Efectivamente, el clima siempre cambió, y qué tiene que ver con la baja sensibilidad climática?

      Ilustranos… o no trates de hacer colar un non sequitur más grande que el obelisco.

      Quote:
      Los modelos no sirven =/= Los modelos desmienten el CCA

      En serio no lo entendiste?

      Te lo explico: cuando hablé de «los modelos» me referí a los utilizados para pronosticar el apocalipsis. No significa que ningún modelo sirva!

      Vamos… hay gente grande que lee esto, no los/nos tomes por idiotas.

      Quote:
      Usar variabilidad natural como evidencia a favor del CCA (eventos climáticos extremos) =/= Usar variabilidad natural como evidencia contra el CCA (1998 fue muy cálido, este febrero es más frío de lo normal, los últimos 3 años las capas polares ganaron hielo)

      Te lo digo por tercera y última vez: lo de los hielos lo saqué a colación para evidenciar la mentira de los medios («los hielos se siguen derritiendo») cuando los datos muestran otra cosa.

      Eso se usa como propaganda, ya sabrás…

      Ya que hablás, no de 3 años, la tendencia de los últimos 10 años es negativa (enfriamiento). Dirás que 10 es poco, que tomemos 30, y yo te voy a preguntar por qué? por qué no tomar 300 o 3000?

      Es lógico que exista cierto calentamiento natural, ya que estamos saliendo de una pequeña edad de hielo, y sabés qué pasa luego de las mismas? Sí, el planeta se calienta! Incluso cuando el humano ni siquiera sabía prender una fogata.

      Quote:
      Los climatólogos utilizan eventos climáticos extremos como evidencia a favor del CCA =/= Los climatólogos dicen que los eventos climáticos extremos no son evidencia a favor del CCA

      Exacto, así de inconsistentes son.

      Quote:
      Hay un fraude global =/= Los datos de la NASA y las NU desmienten el CCA

      El fraude dentro de la comunidad climatológica está documentado.

      Algunos datos de la NASA/GISS no le convienen a la religión, perdón, teoría del AGW.

      Quote:
      Es el Sol =/= El Sol sólo fue responsable de la mitad del calentamiento

      Algunos lo sostienen, otros no (Dr.Roy Spencer) y proponen teorías alternativas para explicar el aumento observado, que por cierto, se ajustan más a la realidad que la teoría del AGW.

      Quote:
      Es el Sol =/= No hay calentamiento

      Algunos lo sostienen, otros no. Existe un pequeño calentamiento, 0.6ºC en el último siglo.

      Es causado por el hombre? Pareciera que no, de cualquier manera no está claro.

      Es malo o catastrófico ese calentamiento? Pareciera que no, de cualquier manera no es ni remotamente tan preocupante como lo pinta el IPCC.

      Quote:
      Es el Sol =/= Hay más calentamiento durante la noche que durante día

      Que haya más calentamiento durante las noche que durante el día está en contradicción con la hipótesis del calentamiento debido a gases de efecto invernadero (y obviamente el CO2).

      Quote:
      La mitad del calentamiento sería producido por hollín =/= No hay calentamiento antropogénico.

      Primero que habla del calentamiento de ciertas regiones y no del calentamiento global.

      Y más importante, cuando se habla de calentamiento global antropogénico se entiende que nos referimos al generado por el CO2 antropogénico.

      Vamos… o acaso también los modelos del IPCC tienen en cuenta el calor generado por los microondas, las estufas de cuarzo y los asaditos del domingo?

      En serio creés que no se van a dar cuenta los lectores?

    • #30548

      DrGEN dijo:

      Quote:
      Mayor calentamiento durante la noche que el día no sólo es perfectamente consistente con un efecto invernadero, ¡sino que es inconsistente con la idea de que el Sol es la causa!

      Es inconsistente con el efecto invernadero! Los climatólogos lo saben.

      Sacado del AR4 del IPCC.

      Quote:
      In the last 50 years for the land areas sampled, there has been a significant decrease in the annual occurrence of cold nights and a significant increase in the annual occurrence of warm nights (Figure 1). Decreases in the occurrence of cold days and increases in hot days, while widespread, are generally less marked

      Por otro lado no estoy diciendo que el Sol sea la causa. Leé bien, vos podés.

      Decime, entonces, qué quiere decir esto:

      DrGEN dijo:También un paper con más de 600 referencias, concluye que sólo el forzamiento solar es responsable de la mitad del calentamiento desde 1860!

      ¿?

      Quote:
      Baja sensibilidad climática =/= el clima siempre cambió.

      Efectivamente, el clima siempre cambió, y qué tiene que ver con la baja sensibilidad climática?

      Ilustranos… o no trates de hacer colar un non sequitur más grande que el obelisco.

      Si la sensibilidad es baja, significa que es estable, por lo que no puede estar cambindo todo el tiempo. Si cambia mucho, significa que es inestable y, por lo tanto, tiene más sensibilidad.

      Por otro lado, nunca terminaste tu argumento de «el clima siempre cambió». El clima siempre cambió y entonces…. ¿?

      Quote:
      Los modelos no sirven =/= Los modelos desmienten el CCA

      En serio no lo entendiste?

      Te lo explico: cuando hablé de «los modelos» me referí a los utilizados para pronosticar el apocalipsis. No significa que ningún modelo sirva!

      Vamos… hay gente grande que lee esto, no los/nos tomes por idiotas.

      Ah! Claro. Osea que los únicos modelos que no sirven son lo que contradicen tu posición.

      Quote:
      Usar variabilidad natural como evidencia a favor del CCA (eventos climáticos extremos) =/= Usar variabilidad natural como evidencia contra el CCA (1998 fue muy cálido, este febrero es más frío de lo normal, los últimos 3 años las capas polares ganaron hielo)

      Te lo digo por tercera y última vez: lo de los hielos lo saqué a colación para evidenciar la mentira de los medios («los hielos se siguen derritiendo») cuando los datos muestran otra cosa.

      Eso se usa como propaganda, ya sabrás…

      Ya que hablás, no de 3 años, la tendencia de los últimos 10 años es negativa (enfriamiento). Dirás que 10 es poco, que tomemos 30, y yo te voy a preguntar por qué? por qué no tomar 300 o 3000?

      Es lógico que exista cierto calentamiento natural, ya que estamos saliendo de una pequeña edad de hielo, y sabés qué pasa luego de las mismas? Sí, el planeta se calienta! Incluso cuando el humano ni siquiera sabía prender una fogata.

      Pero.. ¿no era que en los últimos 15 años no hay cambio estadísticamente significativo? ¿Hay cambio o no? ¿Por qué tomar 30 años o más? Porque te brinda mayor poder estadístico para establecer, justamene, la tendencia a largo plazo. ¿POr qué no usar 3000 años? Porque los procesos que se están evluando no operan en esa escala.

      Es lo mismo que con un paciente de cáncer. No vas a pensar que porque durante una semana el tumor se mantuvo o retrocedio ligermaente está curado, pero tampoco podés considerar un lapso de 70 años.

      Quote:
      Los climatólogos utilizan eventos climáticos extremos como evidencia a favor del CCA =/= Los climatólogos dicen que los eventos climáticos extremos no son evidencia a favor del CCA

      Exacto, así de inconsistentes son.

      Lo inconsistente es que vos afirmás ambas cosas a la vez.

      Quote:
      Hay un fraude global =/= Los datos de la NASA y las NU desmienten el CCA

      El fraude dentro de la comunidad climatológica está documentado.

      Algunos datos de la NASA/GISS no le convienen a la religión, perdón, teoría del AGW.

      Veo que tenés una herramienta perfecta para filtrar datos. Los que confirman el CCA = parte de la conspiración; los que lo contradicen = «datos que no le convienen a la religión del CCA».

      Quote:
      Es el Sol =/= El Sol sólo fue responsable de la mitad del calentamiento

      Algunos lo sostienen, otros no (Dr.Roy Spencer) y proponen teorías alternativas para explicar el aumento observado, que por cierto, se ajustan más a la realidad que la teoría del AGW.

      Eso no quita que en el mismo post sostuvieras ambas cosas.

      Quote:
      Es el Sol =/= No hay calentamiento

      Algunos lo sostienen, otros no. Existe un pequeño calentamiento, 0.6ºC en el último siglo.

      Es causado por el hombre? Pareciera que no, de cualquier manera no está claro.

      Es malo o catastrófico ese calentamiento? Pareciera que no, de cualquier manera no es ni remotamente tan preocupante como lo pinta el IPCC.

      De nuevo, la inconsistencia radica en afirmar ambas cosas al mismo tiempo.

      Quote:
      Es el Sol =/= Hay más calentamiento durante la noche que durante día

      Que haya más calentamiento durante las noche que durante el día está en contradicción con la hipótesis del calentamiento debido a gases de efecto invernadero (y obviamente el CO2).

      Ya respondido más arriba.

      Quote:
      La mitad del calentamiento sería producido por hollín =/= No hay calentamiento antropogénico.

      Primero que habla del calentamiento de ciertas regiones y no del calentamiento global.

      Y más importante, cuando se habla de calentamiento global antropogénico se entiende que nos referimos al generado por el CO2 antropogénico.

      Vamos… o acaso también los modelos del IPCC tienen en cuenta el calor generado por los microondas, las estufas de cuarzo y los asaditos del domingo?

      Si no tiene que ver con el calentamietno global.. ¿a qué viene? ¿Cómo contradice el CCA algo que luego vos mismo decís que no tiene nada que ver con éste?

      Como habrás visto, muchas de esas inconsistencias radican en que no presentás tu posición con claridad y te limitás a tirar objeciones desarticuladas en vez de establecer un conjunto de hechos, evidencias y datos sistematizados. Este también es un síntoma del gish gallup. Cuando se muestran esas inconsistencias, te distanciás de ellas. «Yo nunca dije eso, sólo digo lo que otros dicen» no es una táctica honesta de argumentación. A menos que te comprometas (al menos de forma tentativa) a lo que afirmás, no hay objeción que valga. Yo admito que no tengo el conocimiento para analizar muchos de los papres, pero si comento lo que afirman los científicos, lo hago mostrando qué es lo que opino yo; como parte de mi entendimiento acerca del proceso. Si hay una contradicción en mi esquema, no puedo simplemente externalizarla, significa que hay algún problema en mi entendimiento que debe ser arreglado.

      Como dije antes, no se puede llevar un intercambio cuando no sé qué es lo que estás diciendo. ¿La Tierra se está calentando o no? ¿El Sol es la causa o no? ¿en qué medida es el sol? ¿En qué medida son los humanos? ¿Qué tan grande va a ser? Mientras no presentes una ideas coherente de lo que estamos discutiendo, lo único que vamos a lograr es un conjunto de hechos y figuras sin relación alguna y muchas veces contradictorios. Por ejemplo, argumentar que el Sol no es el responsable del calentamiento no tiene sentido si directamente no creés que exista. Hasta ahora y por la forma en la que presentás argumentos parece que tenés una opinión ya formada (que todo esto del CCA es un fraude y es mentira) y lo único que hacés e buscar y publicar datos sueltos que la confirman. Demostrá que no es así presentando en forma básica y general qué es lo que creés.

    • #30549
      DrGEN
      Miembro

      Podría contestar punto por punto a las «contradicciones» pero también puedo hacerlo globalmente: No entendiste mi comentario.

      No son contradicciones en mi postura. Ni siquiera dije que yo estaba de acuerdo, por ejemplo, con quienes afirman que la mayor parte se debe al Sol.

      Lo que yo mostré en mi comentario es que existen contradicciones entre la realidad y los modelos y contradicciones entre los climatólogos de primer nivel. Todo esto demuestra que el tan mentado «consenso» (en la ciencia climatológica) es un invento, una mentira grande como el sistema solar.

      El único consenso es el político.

      Ahora que entendiste (espero) que no estoy diciendo que el sol es el culpable y después que no, supongo que también entenderás mi postura: soy escéptico.

      La evidencia no es suficiente.

      Ahora aclarándote un poco las cosas:

      Dijiste:

      Quote:
      Si la sensibilidad es baja, significa que es estable, por lo que no puede estar cambindo todo el tiempo. Si cambia mucho, significa que es inestable y, por lo tanto, tiene más sensibilidad.

      No Daneel, cuando hablamos de sensibilidad climática nos referimos a el cambio en relación a duplicar la concentración de CO2.

      Una sensibilidad baja, incluso de 0ºC (lo que significaría que el CO2 no influye en la misma) no descarte ni remotamente que el clima cambie!

      Si pensás eso pareciera que para vos el único driver es el CO2… o sea, para vos lo único que modifica la temperatura es el CO2! jajaja… en serio pensás eso? Y las nubes, y el sol, y los océanos, y…. etc, etc, etc.

      Ahora entiendo que los modelos sean tan simples :P (pero que no se ajusten a lo observado es un detalle…)

      Por cierto, si el único driver es el CO2… qué pasó en las glaciaciones? :P

      (no hace falta que contestes, son retóricas y sé que sabes que el CO2 no es el único driver… por eso es llamativo tu comentario sobre la sensibilidad climática)

      Después dijiste:

      Quote:
      Pero.. ¿no era que en los últimos 15 años no hay cambio estadísticamente significativo? ¿Hay cambio o no? ¿Por qué tomar 30 años o más? Porque te brinda mayor poder estadístico para establecer, justamene, la tendencia a largo plazo. ¿POr qué no usar 3000 años? Porque los procesos que se están evluando no operan en esa escala.

      Y después acusás de cherrypicking… hay que tener cara che! :P

      Semi Off-Topic:

      Hace un par de días, por casualidad llegué al blog de Agostinelli y me reí bastante, me causó risa ver tu comentario sorprendido porque Ale Borgo (sí, el del CFI Argentina!) sea escéptico con respecto al AGW! qué esperabas? :P

    • #30550

      DrGEN dijo:

      Podría contestar punto por punto a las «contradicciones» pero también puedo hacerlo globalmente: No entendiste mi comentario.

      No son contradicciones en mi postura. Ni siquiera dije que yo estaba de acuerdo, por ejemplo, con quienes afirman que la mayor parte se debe al Sol.

      Lo que yo mostré en mi comentario es que existen contradicciones entre la realidad y los modelos y contradicciones entre los climatólogos de primer nivel. Todo esto demuestra que el tan mentado «consenso» (en la ciencia climatológica) es un invento, una mentira grande como el sistema solar.

      El único consenso es el político.

      Eso es como decir que que un biólogo niegue la evolución, refuta la idea de que haya un consenso; es evidencia anecdótica. ESO es cherry picking. Lo que hay que evaluar no es sólo el numerador (cantidad de climatólogos que no aceptan el consenso), sino también el denominador (cantidad de climatólogos en total). Si hacés eso llegás al famoso 97% de climatólogos que aceptan el consenso.

      Quote:
      Ahora que entendiste (espero) que no estoy diciendo que el sol es el culpable y después que no, supongo que también entenderás mi postura: soy escéptico.

      La evidencia no es suficiente.

      Eso no es aclarar la posición. ¿Qué es lo que no creés? ¿Cómo creés que las cantidades masivas de CO2 emitidas por el hombre no están produciendo un efecto invernadero? ¿Hay calentemitno o no? ….

      Ahora aclarándote un poco las cosas:

      Dijiste:

      Quote:
      Si la sensibilidad es baja, significa que es estable, por lo que no puede estar cambindo todo el tiempo. Si cambia mucho, significa que es inestable y, por lo tanto, tiene más sensibilidad.

      No Daneel, cuando hablamos de sensibilidad climática nos referimos a el cambio en relación a duplicar la concentración de CO2.

      Una sensibilidad baja, incluso de 0ºC (lo que significaría que el CO2 no influye en la misma) no descarte ni remotamente que el clima cambie!

      Si pensás eso pareciera que para vos el único driver es el CO2… o sea, para vos lo único que modifica la temperatura es el CO2! jajaja… en serio pensás eso? Y las nubes, y el sol, y los océanos, y…. etc, etc, etc.

      Ahora entiendo que los modelos sean tan simples :P (pero que no se ajusten a lo observado es un detalle…)

      Por cierto, si el único driver es el CO2… qué pasó en las glaciaciones? :P

      (no hace falta que contestes, son retóricas y sé que sabes que el CO2 no es el único driver… por eso es llamativo tu comentario sobre la sensibilidad climática)

      Y todavía no terminaste tu argumento. «El clima siempre cambió» … ¿y entonces?.

      Quote:
      Después dijiste:

      Quote:
      Pero.. ¿no era que en los últimos 15 años no hay cambio estadísticamente significativo? ¿Hay cambio o no? ¿Por qué tomar 30 años o más? Porque te brinda mayor poder estadístico para establecer, justamene, la tendencia a largo plazo. ¿POr qué no usar 3000 años? Porque los procesos que se están evluando no operan en esa escala.

      Y después acusás de cherrypicking… hay que tener cara che! :P

      Explicá, entonces, qué tiene de malo utilizar las escalas temporales correctas. En la oración siguiente lo ilustré con la analogía con un paciente con cáncer. Si estamos hablando de un proceso cuyo efecto se observa en la escala de décadas, ¿por qué sería relevante evaluar la tendencia en la escala de millones de años?

      Semi Off-Topic:

      Hace un par de días, por casualidad llegué al blog de Agostinelli y me reí bastante, me causó risa ver tu comentario sorprendido porque Ale Borgo (sí, el del CFI Argentina!) sea escéptico con respecto al AGW! qué esperabas? :P

      No es tanto sorpresa como desilusión. Igual, aunque ya no es extraño encontrar negacionistas entre las filas del pensamiento crítico, no deja de ser trágico que el escepticismo se vaya al lado obscuro. ¿Qué esperaba?

      ¿Leiste la respuesta de Borgo? ¿Te das cuenta que critica algo de lo que ni siquiera tiene idea?

      (Para quienes lean esto sin leer aquello, Borgo criticó el lenguaje «poco científico», según él, del IPCC al usar calificativos como «es probable». Lo que Borgo aparentemente no sabía, es que en el propio documento del IPCC hay un anexo donde se establece con precisión qué significa cada expresión de probabilidad. «Muy probable», por ejemplo, significa que es con una probabilidad mayor al 90%)

    • #30551
      saibaba
      Miembro

      Primero aclaro que no he leído todo, sino he echado una ojeada y leí algunas discusiones últimas.

      Es interesante lo que ocurre en este debate, en donde a pesar de que pareciera que la mayoría de los científicos coinciden en su opinión sobre el cambio climático, hay otros que no, y no se puede determinar cuál sería el «porcentaje» de especialistas que debieran estar de acuerdo para que los políticos actúen en consecuencia.

      Nunca habrá un 100% de consenso en nada, ni un 100% de seguridad de ninguna cosa.

      Así que, ¿cuánto porcentaje hace falta para que la opinión científica se traduzca en una ley hecha y derecha?

      Además, siempre va a haber millonarios capaces de poner a dudar todas las cosas, de manera que estas cosas se dilaten.

      Es una puja política.

      Me parece que, aunque no haya evidencia «suficiente», en un caso como éste debiera bastar que haya evidencia «bastante sugestiva», por decirlo así.

      Justifico esta postura en una cuestión de mera «prevención».

      Se está hablando de un riesgo de cataclismo mundial, y muchos expertos están preocupados.

      Esto debiera ser suficiente para lanzar medidas legales «preventivas».

      Porque, después de todo, puede ocurrir que obtengamos el 100% de certeza sólo cuando veamos los cataclismos venírsenos encima, y ya nada podamos hacer al respecto.

      La duda que tengo es si acaso no hay un vacío legal en este tipo de cosas.

      Si la ciencia es «lo fiable» en nuestro mundo, ¿la palabra científica no debiera ser ley? ¿No debiera ser la comunidad científica quien decida en estos asuntos?

      Si se trata de dudar de los datos, se puede dudar de todo, dato a dato se puede cuestionar: que si el aparato que tomó los datos era bueno, que si alguno tergiversó los números, que si se introdujo ruido, que si hubo justo alguna variación momentánea pero no significativa… o quién sabe qué otras excusas que ni logro imaginar.

      ¿Alguien sabe cómo es el trasfondo legal asociado a la opinión científica?

      Porque de eso se trata este problema, digo yo, y no tanto de teorías científicas.

      No es lo mismo especular con un valor de 9,8 o 9,82 para la gravedad, que decidir por sí o por no si «no se hace nada ante el cambio climático».

      Mientras hay duda e incertidumbre, parece ser una «cara o cruz» ante el fin del mundo: «fin o no-fin».

      No es aceptable tomarse el asunto así. Y menos cuando parece haber tantos expertos realmente preocupados.

      ¿O acaso los que están preocupados son sólo dos o tres locos perdidos?

    • #30552
      saibaba
      Miembro

      Lo que dijo DrGen unos posts atrás está errado: la ciencia empírica no tiene «pruebas» concretas de ninguna cosa. Las teorías científicas se admiten sólo por un tiempo hasta que aparece la necesidad de suplantarlas por otras mejores.

      Galileo no «demostró» ni «probó» nada con sus experimentos. Él sólo enunció una ley que se vio respaldada por experimentos.

      Pero ninguna secuencia finita de experimentos puede validar una afrimación que abarca infinitos casos. Eso es lógicamente inaceptable.

      Decir: «todos los cuerpos se atraen entre sí según la ley de la gravedad»

      es algo que nunca se podrá probar desde el punto de vista lógico,

      porque para probarlo realmente es necesario verificar todos los pares de cuerpos del Universo, y comprobar que efectivamente así ocurre.

      Pasar de los experimentos a las leyes (o modelos formales) es realiza una «razonamiento inductivo», lo cual es, desde el punto de vista lógico, lo mismo que mentir.

      La ley de gravedad es una ley «temporal» que se sostiene porque ningún cuerpo relevante de experimentos le ha jugado en contra.

      Pero bien podría ocurrir que se descubra que, en algún contexto aún desconocido, dicha ley ya no funcione, o funcione distinto, o con otros valores, o lo que sea.

      Claro que yo no me voy a tirar de un piso veinte para ver si «justo ocurre un fallo experimental» y me salvo de caer al piso. Pero puede ocurrir.

      Y en caso de que algo así ocurra (un objeto que no caiga), habrá que modificar las leyes físicas para que expliquen los casos anómalos.

    • #30553

      Yo no diría que se trata de un «cataclismo mundial». Se trata de un término demasiado cargado y, más importante, vago. Los efectos del calentamiento global están siendo estudiados y se conocen dentro de un rango de incertidumbre. Por ejemplo, un aumento del nivel del mar de entre x e y (no tengo los números a mano).

    • #30554
      saibaba
      Miembro

      Y bueno, yo sé menos que vos del tema.

      Pero en todo caso a lo que apunto es que la palabra de los científicos debiera tener carácter legal.

      Si no, ¿para qué se les paga, para qué se confía en ellos, y para qué se les piden informes y opiniones de «experto» si después no se les toma en cuenta, y los que deciden son unos abogadeitors de mierda que no les importan ni sus propios hijos?

      La opinión «experta» ¿sirve para algo?

      Porque si no, busquemos la opinión de Horangel a ver qué dice sobre el asunto.

    • #30555

      Mhh.. El tema es que la ciencia cambia mucho, especialmente cuando se trata de vías de investigación nuevas. Además el consenso científico es algo muy difícil de calcular. Si sabemos que existe en lo referido a evolución, relatividad, etc… es porque se trata de teorías robustas que han pasado la prueba del tiempo.

      La opinión experta sirve como guía y debería informar las acciones políticas. Pero darle estatus de ley automático a las opiniones científicas no creo que pueda dar buenos resultados y sería un despelote. Lo que debemos hacer (y en parte para eso está el CEA) es para generar un «meme» cultural de valoración de la ciencia. Hacer que si un político usa nuestro conocimiento para limpiarse el culo, no pueda obtener un cargo nunca más en su vida. (Es un poco extremo, pero tiene buen efecto :P).

    • #30556
      N3RI
      Superadministrador

      Bueno, mi amigo DrGen primero frenó el debate con un raro capricho sobre la definición de dos palabras que si no las decíamos como él decía, no continuaba; que por cierto ¿aclaró cuál era la definición que buscaba o el parate fue al pedo?

      Y ahora, nuevamente, otro evidente capricho, de no querer aclarar su postura, que es re confusa y que se la pedimos varios, varias veces. Flaco, ACLARÁ TU POSTURA Y QUÉ ES LO QUE ESTÁS NEGANDO.

    • #30557
      N3RI
      Superadministrador

      Sobre el «consenso»

      Ya aclaramos unas páginas atrás que el consenso no sólo es político, sino que también es científico. De hecho, es más científico que político, porque ya que tanto les gusta hablar de porcentajes, el porcentaje de políticos en contra de la teoría del calentamiento global antropocéntrico es y fue siempre mucho mayor al porcentaje de científicos climatólogos en contra. También aclaramos que la ciencia sí se maneja por «consenso», pero que así como la palabra «teoría» no significa lo mismo en el ámbito científico que en el mundano, así también cuando hablamos de «consenso» científico no hablamos de meras opiniones, sino de científicos que hacen los mismos experimentos, toman las mismas mediciones, y llegan a las mismas conclusiones. ESO es consenso científico.

      También parece que ambas partes estamos de acuerdo en este debate con ese numerito del 97% de expertos climatólogos que están a favor de la teoría del calentamiento global antropocéntrico. Salvo que tal vez DrGen opina que el porcentaje es menor.

      Aclarado eso, yo les pregunto, ¿qué porcentaje de consenso sería «aceptable»? ¿Sólo el 100%? ¿El 90%? ¿El 80% es aceptable? ¿El 99%?

      Porque hay muchos campos de la ciencia en los que no se puede lograr un 100% de consenso. Incluso en el caso de teorías muy robustas siempre hay científicos que están estudiando otros posibles modelos. Pero hay veces que la toma de datos dista de ser exacta, por ejemplo, la arqueología, pero es la suma de todos los datos lo que le da fuerza a la teoría, en su conjunto.

      Algo que yo noto que Daneel insiste mucho y DrGen hace oídos sordos es que no hay disenso en los puntos básicos del calentamiento global. Lo mismo que pasa con la relatividad, la evolución, etc… que existan detalles o modificaciones, incluso otros factores además del CO2, no quiere decir que toda la teoría esté mal.

      Sobre «la actitud escéptica»

      Como estamos en el círculo escéptico, me pregunto y les pregunto también ¿cuál es la actitud realmente más escéptica ante este tema? Negar de lleno lo que la gran mayoría de los científicos expertos en un tema afirman ¿es una actitud escéptica o es más bien una actitud negacionista? ¿No habría que analizar todos los datos, en vez de sólo los que están en contra?

      Afirmar que la comunidad científica es muy sana y funciona bien en todos los ámbitos, menos el que yo no estoy de acuerdo, hablar de conspiraciones, verso y engaño mundial es realmente parte del pensamiento escéptico? ¿Cómo se puede acusar a una parte específica de la comunidad científica de conspiraciones motivadas por el dinero y no dudar de toda la comunidad científica entera?

      ¿Tener una postura ambigua (y no aclararla aunque te lo pidan varias veces) en vez de aclarar puntualmente en qué se está en desacuerdo es realmente escéptico?

      ¿No debería un escéptico presentar todos los datos y analizarlos y no sólo los que estén de acuerdo con la opinión propia y nieguen la contraria?

    • #30558
      N3RI
      Superadministrador

      Sobre «la conspiración climática».

      ¿Podemos acusar livianamente a la comunidad científica de fomentar una teoría sólo por intereses económicos? ¿No es el proceso científico tal que «haría saltar» si realmente ocurriera? ¿Es posible una comunidad científica que en su mayoría sea deshonesta, corrupta y con doble agenda?

      ¿Cómo puede una persona racional y escéptica afirmar que la NASA no fue parte de una conspiración mundial para inventar un alunizaje (cosa a la que se dedican) pero que sí es parte de una conspiración mundial para inventar un problema climático global (cosa a la que no se dedican, sino que colaboran con la toma de datos)?

      ¿Las fotos del alunizaje no fueron falsas, pero los datos, los gráficos y las fotos que muestran el calentamiento de la atmósfera, el retroceso de los hielos, etc, etc… esos sí son falsos?

      ¿En el tema de conspiraciones climáticas, no tiene mucho más sentido lógico sospechar que los intereses corruptos son los que niegan el calentamiento y no los que lo afirman? ¿Qué intereses hay detrás de cada postura? ¿Quiénes tienen oscuros intereses en este tema? ¿Greanpeace, que no quiere salvar el planeta, sino que realmente quiere «fingir» que salva el planeta? ¿las pocas empresas que fabrican molinos de viento o paneles solares a nivel mundial, que cuentan con un presupuesto infinitamente pequeño para sobornar políticos y científicos? ¿los aborígenes brasileros que quieren que todo el mundo sea reforestado? ¿o las petroleras, las madereras, y otras empresas del sector energético que tienen absoluto monopolio mundial, que son la base de la economía mundial, que si modifican el valor del barril de petróleo modifican todos los precios de todo en el mundo, que ya han sido descubiertas en centenares de casos de corrupción y soborno, que tienen lobbies mundialmente reconocidos?

      ¿tiene mayor sentido acusar de oscuros intereses económicos al 97% de climatólogos a favor o al 3% en contra?

    • #30559

      Ni hablar que no son los climatólogos quienes llegan en en autos de alta gama y vacacionan en cruceros.

    • #30560
      saibaba
      Miembro

      Está bueno que aprovechen las playas mientas existan.

      Eso sí, que les pregunten a los otros climatólogos cuando y dónde conviene irse de vacaciones, no vaya a ser que justo se te venga el apocalipsis en plena estadía.

    • #30561
      N3RI
      Superadministrador

      el día que se bombardeen países del medio oriente para ir a plantar en ellos molinos de viento, ese día yo creeré que hay intereses económicos a favor de la teoría del calentamiento global.

    • #30562
      Sir Arquimedes
      Participante

      Hoy leo todos los post y la enorme y caliente discusion de la que me abstengo de participar, tal vez por mi ignorancia, solo teno una pregunta:

      qie significa gish gallup??

    • #30563
      Quote:
      La “Gish Gallop” (literalmente, Galope de Gish) es una técnica nombrada en honor a su más famoso representante Duane Gish. Duane es un creacionista de cierta nota muy conocido por sus debates. Este germen particularmente patógeno se sienta en su silla y en los 10 minutos que dura su presentación, presenta una montaña de datos falsos, argumentos falaces y malinterpretaciones del estado de la ciencia confiando que no existe ser humano capaz de responder y corregir semejante cantidad de barrabasadas en el tiempo que tiene para responder. La enorme cantidad de datos asegura, además, que aún cuando su oponente refute varios de ellos, él puede ignorarlo por completo y señalar todo lo que éste no respondió.

      (lo saqué de mi blog :P)

    • #30564
      Sir Arquimedes
      Participante

      Gracias Daneel! eberias chequear que onda con tu blog porque cuando googlee esa palabra (hoy a la madrugada) me aparecio cualquier cosa y nunca tu blog!

    • #30565

      No sé. Yo probé y lo encontré en la segunda página:

      Upbop.png

    • #30566
      N3RI
      Superadministrador

      Antártida se está volviendo verde por el cambio climático:

      http://www.telegraph.co.uk/earth/earthnews/8413806/Antarctica-going-green-due-to-climate-change.html

      Me hizo acordar a la foto de Maggie Simpson en la postal que le manda a los padres en el futuro, en la playa con el oso polar. Por cierto, uno de mis sueños es visitar la Antártida.

    • #30567
      ryomashi
      Miembro
    • #30568
      N3RI
      Superadministrador

      http://diariocorreo.pe/nota/1956/nasa-el-amazonas-se-seca/

      Los modelos de proyección climática indican que, en el futuro, el aumento de las temperaturas y la alteración de los patrones de precipitaciones pueden disminuir la humedad en la región, provocando una sustitución de la selva húmeda por vegetación tipo sabana leñosa o praderas. De acuerdo con los expertos de la NASA, este fenómeno supondría la emisión a la atmósfera del carbono almacenado en la madera putrefacta, lo que aceleraría el calentamiento global.

    • #30569

      ryomashi dijo:

      http://www.liberalismo.org/articulos/29/ecologismo/

      esa es la pagina de donde saque este articulo:

      http://www.liberalismo.org/articulo/440/29/aprendiendo/vivir/calentamiento/global/catastrofismo/

      Que opinan?

      En principio, creo que nunca nadie debería conseguir su información científica de un citio con tan clara tendencia ideológica. Segundo, me parece que el artículo tiene un montón de falsedades, afirmaciones sin fundamentos y argumentos francamente ridículos. Varias veces confunde eventos locales con fenómenos globales. Dice, por ejemplo, que no hubo cambio en los últimos 11 años… eso es mentira. La Tierra se sigue calentando, el nivel del mar sigue aumentando, los hielos se siguen derritiendo.

      No se acerca ni de lejos a refutar la enorme cantidad de evidencia que indica que la Tierra se está calentando y que el principal responsable son las actividades humanas.

      Su analogía con la bosta de caballo (que ya he visto en otros artículos) es absolutamente ridícula y, de hecho, juega en su contra. Al juguetear con el negacionismo y presuponer que El Mercado (TM) va a solucionar todo, está autocumpliendo las proyecciones del IPCC. Quien reconoce que el cambio climático es un problema y busca medidas para detenerlo y mitigarlo, ayuda a que esas proyecciones no se hagan realidad. Es curioso que el mayor logro de la climatología sería que sus proyecciones no se cumplan.

      Creo que a esta altura, debería ser una regla como la Ley de Godwin que cualquier persona que saca a colación el evento de Climategate pierde el argumento inmediatamente. Especialmente si ni siquiera habla de las varias investigaciones que concluyeron que los científicos no hicieron nada malo y que la ciencia del cambio climático no se basa en los resultados de un solo laboratorio. Lo mismo va para cualquier acusación infudnada de fraude. Es especialmente vergonzoso que compare una campaña publicitaria de conscientización sobre el cambio climático (que son ganancias para las empresas de publicidad, no los climatólogos) con las donaciones de Exxon directamente a los «escépticos». O la financiación por parte del estado estadounidense a la investigación científica diciendo que «es menos de lo que tienen los «escépticos»».

      Cito:

      Quote:
      En rigor, el volumen de fondos de empresas privadas destinado a campañas y estudios escépticos palidece ante las partidas que los Estados dedican a la investigación del cambio climático.

      Bien podríamos acusar a los biólogos de recibir una financiación enorme por pare del estado cuando los «estudios escépticos» sólo reciben una mínima proporción de esta cantidad. Como remate, en ningún momento afirma cuál es la financiación per cápita, que es lo que finalmente importa.

    • #30570
      N3RI
      Superadministrador

      http://www.latimes.com/news/local/la-me-climate-berkeley-20110404,0,772697.story

      uno de los principales escépticos del cambio climático, físico de Berkeley, decidió hacer su propia investigación sobre el cambio climático para desmentir a los «exageradores»; ahora presenta sus estudios y resultados preliminares de su investigación y concluye que sus datos son «muy parecidos a los de investigaciones anteriores», o sea, a favor de que hay una tendencia cierta hacia el calentamiento global. Afirmando que según sus investigaciones, no hay exageración en los datos tomados por anteriores científicos.

      Como buen científico, cambia su postura. Y qué pasa entonces? que los negacionistas le hacen la cruz y lo censuran. Pasan de usarlo de ejemplo y caballito de batalla, de esperar que sus datos «desmientan» los de otros científicos… a hundirlo. Pasan de financiar todo su estudio, a injuriarlo y no darle más un peso.

      si bien su investigación no está terminada aún y puede «volver a cambiar», es más probable que no lo haga. Aún así, aunque esté a favor del consenso, destaco que una investigación puramente estadística no es suficiente para demostrar el cambio climático aontropogénico, pero sí sirve para demostrar que los datos fueron tomados y medidos correctamente (obvio, se hizo varias veces a lo largo y ancho del mundo) y que no son exagerados, como dicen los negacionistas con sus obvios intereses económicos detrás.

    • #30571

      Sí, esto ya se hizo varias veces; se llama peer review. Me acuerdo que hace un tiempo Associated Press presentó datos del clima (temepraturas seguro, no sé si otros datos también) a estadistas para que los analizaran. Lo interesante es que los tipos no sabían qué era lo que estaban analizando, para ellos eran números. Los resultados fueron, para sorpresa de nadie, que el planeta se está calentando.

      «But he said the Berkeley team had been «seriously compromised» by publicizing its work before publishing any vetted papers.»

      Esto es verdad. Con sólo el 2% de los datos analizados, no creo que fuera prudente salir a la prensa. Ahora hay más riesgo de sesgos en la manipulación de datos ya sea en la dirección de corroborar lo que dijo públicamente o al recibir presiones de grupos externos. Vamos a ver.

      Igual, como bien dice el artículo, la temperatura es sólo una de las tantas líneas de evidencia que muestran la existencia del cambio climático.

    • #30572
      DrGEN
      Miembro

      Daneel:

      Quote:
      «En principio, creo que nunca nadie debería conseguir su información científica de un citio con tan clara tendencia ideológica.»

      JAJAAJ, en serio creés que el IPCC no tiene una «clara tendencia ideológica»? :P

      Quote:

      «Creo que a esta altura, debería ser una regla como la Ley de Godwin que cualquier persona que saca a colación el evento de Climategate pierde el argumento inmediatamente.»

      Muy conveniente! Sobre todo ahora que siguen saliendo trapitos sucios y malolientes de ahí… te faltó decir: «decretemos que no vale nombrarlos… sino no juego más» :P

      N3RI:

      «uno de los principales escépticos del cambio climático, físico de Berkeley, decidió hacer su propia investigación sobre el cambio climático»

      Me parece que tus fuentes son… bue, no voy a hablar de tus fuentes.

      A lo que voy es a que el «físico de Berkeley» no es un escéptico sino que apoya la hipótesis oficial del IPCC. Y de hecho, es un tipo que por lo menos intenta hacer ciencia, lo cual es raro y elogiable.

      Daneel:

      Quote:
      «Igual, como bien dice el artículo, la temperatura es sólo una de las tantas líneas de evidencia que muestran la existencia del cambio climático.»

      Este punto siempre me pareció muy obvio y no sé cómo no saltan a atacarlo con más fuerza.

      Explico: la temperatura NO es «sólo una» de las líneas de evidencia. Es la única en la que se puede sostener un supuesto cambio climático antropogénico.

      Es decir: si el responsable del «cambio climático» es el CO2 de origen humano, Y el CO2 es el que aumenta la temperatura global por su efecto de gas invernadero, entonces SI o SI tiene que haber aumento de la temperatura. Ese es el punto y el resto serían «síntomas» (las otras «líneas de evidencia»).

      Toda la hipótesis del AGW está sí o sí basada en el aumento de temperaturas globales (y dependiendo totalmente de que ello ocurra).

      Por eso mismo es que los períodos de enfriamiento no convienen a la causa y… se ocultan como ocurrió en el climagate. (huy, perdón, seguís jugando?)

    • #30573

      DrGEN dijo:

      Daneel:

      Quote:
      «En principio, creo que nunca nadie debería conseguir su información científica de un citio con tan clara tendencia ideológica.»

      JAJAAJ, en serio creés que el IPCC no tiene una «clara tendencia ideológica»? :P

      El IPCC es un ente científico, liberalismo.com claramente no. Por supuesto que todos tenemos nuestra ideología, nuestros sesgos y limitaciones, pero mi punto es que la información científica hay que obtenerla de un ente científico, no político. De todas formas el IPCC no es la única organización que afirma la realidad del CC, no hay ninguna sociedad científica nacional que lo rechace.

      N3RI:

      «uno de los principales escépticos del cambio climático, físico de Berkeley, decidió hacer su propia investigación sobre el cambio climático»

      Me parece que tus fuentes son… bue, no voy a hablar de tus fuentes.

      A lo que voy es a que el «físico de Berkeley» no es un escéptico sino que apoya la hipótesis oficial del IPCC. Y de hecho, es un tipo que por lo menos intenta hacer ciencia, lo cual es raro y elogiable.

      ¡Jajaja! Claro. Este tipo sí hace ciencia. El 97% de los climatólogos… ¿qué hacen?

      Daneel:

      Quote:
      «Igual, como bien dice el artículo, la temperatura es sólo una de las tantas líneas de evidencia que muestran la existencia del cambio climático.»

      Este punto siempre me pareció muy obvio y no sé cómo no saltan a atacarlo con más fuerza.

      Explico: la temperatura NO es «sólo una» de las líneas de evidencia. Es la única en la que se puede sostener un supuesto cambio climático antropogénico.

      Es decir: si el responsable del «cambio climático» es el CO2 de origen humano, Y el CO2 es el que aumenta la temperatura global por su efecto de gas invernadero, entonces SI o SI tiene que haber aumento de la temperatura. Ese es el punto y el resto serían «síntomas» (las otras «líneas de evidencia»).

      Exacto. El causante es el aumento de temperatura, pero las consecuencias de ese aumento son observables y nos pueden indicar, aún si no existiera un termómetro sobre la Tierra, que la temperatura está aumentando. Cuando los glaciares retroceden, los casquetes polares se derriten, las flores florecen antes, aumenta la humedad atmosférica, sube el nivel del mar, etc… podés asegurar con un gran grado de certidumbre que la temperatura está aumentando.

      De todas formas la temperatura sigue aumentando.

      EDIT: Creo que N3RI más abajo lo explicó mucho mejor que yo. Ya que el derretimiento de los hielos y el aumento del nivel del mar tienen que ver con el calor y no con la temperatura.

      Toda la hipótesis del AGW está sí o sí basada en el aumento de temperaturas globales (y dependiendo totalmente de que ello ocurra).

      Por eso mismo es que los períodos de enfriamiento no convienen a la causa y… se ocultan como ocurrió en el climagate. (huy, perdón, seguís jugando?)

      ¿Quién ocultó un período de enfriamiento?

    • #30574
      N3RI
      Superadministrador

      DrGEN dijo:

      Daneel:

      Quote:
      «Igual, como bien dice el artículo, la temperatura es sólo una de las tantas líneas de evidencia que muestran la existencia del cambio climático.»

      Este punto siempre me pareció muy obvio y no sé cómo no saltan a atacarlo con más fuerza.

      Explico: la temperatura NO es «sólo una» de las líneas de evidencia. Es la única en la que se puede sostener un supuesto cambio climático antropogénico.

      Es decir: si el responsable del «cambio climático» es el CO2 de origen humano, Y el CO2 es el que aumenta la temperatura global por su efecto de gas invernadero, entonces SI o SI tiene que haber aumento de la temperatura. Ese es el punto y el resto serían «síntomas» (las otras «líneas de evidencia»).

      Toda la hipótesis del AGW está sí o sí basada en el aumento de temperaturas globales (y dependiendo totalmente de que ello ocurra).

      Por eso mismo es que los períodos de enfriamiento no convienen a la causa y… se ocultan como ocurrió en el climagate. (huy, perdón, seguís jugando?)

      Esto que decís demuestra que no tenés ni el más básico entendimiento de este tema. Ya intenté explicarte la diferencia entre temperatura y calor y se ve que no la entendiste. Estás equivocado, y es como dice Daneel: son todas evidencias conjuntas, y no, no son síntomas del aumento de temperatura, este es sólo otro síntoma más. Son todos síntomas del aumento de CALOR. Si tenés un sistema agua-hielo-aire, y vas agregando CALOR (energía) no obtenés un aumento directo e instantáneo de temperatura en el aire: obtenés derretimiento de hielo, cambio en la humedad del aire (cantidad de nubes), y finalmente aumento de la temperatura.

      Ves? la temperatura puede mantenerse constante o aumentar poquito y el hielo aún así derretirse, y todos los demás síntomas ocurrir, porque no son síntomas del aumento de la temperatura, sino del aumento de energía/calor. Es lo que se conoce como CALOR LATENTE y que tus afirmaciones indican que no tenés idea de que existe o no comprendés, junto con otros importantes conceptos de termodinámica, mecánica de fluidos y teoría de gases, necesarios para entender todo este asunto.

      Ya te dije, imaginate a la Tierra (en realidad, sólo su superficie-clima-biosfera) como si fuera un vaso con agua, hielo y u poco de tierra seca en el borde. Ahora andá calentándolo constantemente (agregando energía/aportando calor) y medí la temperatura del agua, del hielo y del aire (bueno, el aire en realidad no). También medí el nivel del agua, anotá cuánto tarda en subir x mm, luego cuánto tarda en subir 2x, 3x, etc. Analizá en qué momento el nivel del agua llega al pedacito de tierra seca y qué le pasa a este cuando ocurre.

      Y esto va al tema de que la biosfera y el clima terrestre son sistemas autoregulantes, con «retroalimentación negativa». Y justamente estos son los mecanismos que la producen, por eso no tenemos 20ºC más actualmente, el tema es que si lo seguís estirando, el equilibrio se rompe. (obviamente no vamos a esperar a que se derrita todo el hielo polar y de altas cumbres, antes de que eso ocurra ya vamos a vernos afectados y no queremos llegar a tanto)

      Relacionado con esto está tu apreciación que solés afirmar tan a la ligera todo el tiempo de «la temperatura aumentó apenitas 0,7º, no es tan grave». Es gravísimo! porque indica que todos los sistemas reguladores se están rompiendo.

      ¿Entendés que para subir los primeros 0.7º se necesita muchísima más energía/calor acumulada que para subir los próximos 0.7? y que los siguientes 0.7 necesitarán incluso mucho menos energía? el aumento de temperatura no es constante como vos creés, es exponencial. Otra cosa que parece que no entendés. Lo que ocurrió en os últimos 50 años, va a ocurrir en los próximos 25… y luego sólo en 12…

      PD: es genial que te hayas salteado olímpicamente todo el jugoso debate que te dejamos en las páginas anteriores cuando desapareciste y aparezcas ahora sólo a contestar a esta boludez de la «ley de climategate» y del escéptico ex-escéptico (cosa que negaste, pero no pusiste ninguna fuente, te tengo que creer así porque sí que el tipo nunca fue escéptico? te quejás de mis fuentes pero no mostrás ninguna tuya que indique lo contrario)

    • #30575

      Lo de los feedbacks negativos no creo que sea tan así. Hay feedbacks negativos y los hay positivos. El aumento de la cobertura de las plantas al aumentar la temperatura, que aumentan el albedo es un feedback negativo. Pero el derretimiento de los hielos que disminuye la reflectividad es uno claramente positivo. También son positivos el aumento en la humedad (que aumenta el efecto invernadero), el aumento en la temperatura de los océanos, que disminuye su capacidad para diluir CO2 y la liberación de metano por el derretimiento del permafrost. La formación de nubes es un punto de incertidumbre. Si se producen más nubes de baja altura funcionará como un feedback positivo, mientras que el aumento de los cirros va a enfriar un poco el planeta.

      En cualquier caso, no sé si puede decirse «el clima es un proceso con feedbacks negativos». Más bien hay que ponderar los positivos frente a los negativos y determinar cuál es la sensibilidad climática. Varias líneas de evidencia indican que el aumento de la temperatura como respuesta a doblar la concentración de CO2 está entre 2ºc y 4,5ºC siendo 3ºC el valor más probable.

      box-10-2-figure-2-l.png

    • #30576
      N3RI
      Superadministrador

      no, no, no :P nunca quise indicar que fuera un sistema de SOLO feedbacks negativos, es tal cual lo decís vos, hay un montón de variables a considerar, esos ejemplos que das y muchos más. Por eso el tema es tan complicado de analizar. Casos como el de las nubes o la capacidad de absorción del co2 en las profundidades oceánicas y otros ejemplos que dependiendo del caso funcionan como refuerzo positivo o negativo.

      Incluso la vida misma, es un complejo sistema «equilibrador/desequilibrador» :P que hace refuerzo positivo o negativo dependiendo de la situación. Aún así, mi modelo del «vaso de agua con hielo» no está tan mal para explicar lo que quería decir, no era para explicar todo el fenómeno climático.

    • #30577
      N3RI
      Superadministrador

      La cámara de representantes, congreso de eeuu, no aprobó una enmienda que pedía que se reconozca la existencia del cambio climático y que es causado por la actividad humana y que tiene graves consecuencias:

      http://thehill.com/blogs/e2-wire/677-e2-wire/154445-house-votes-down-climate-science-amendment

      Obviamente, hay dos formas de ver la noticia: como otro ejemplo de que la afirmación conspiranoica de que «los políticos» (si tal entidad podría ser que exista y sea única) tienen intereses oscuros a favor de lo del cambio climático, es puras patrañas y nunca ocurrió.

      y la otra es que «oh, por fin nuestros honrados políticos están haciendo las cosas bien y se han dado cuenta de que el cambio climático es un verso; esto es un signo del cambio de mentalidad, celebremos».

      Me pregunto cuál de las dos elegirá drgen :P

      Pero tranquilícense, lo más probable es que el lobby de las empresas de paneles solares se hayan olvidado de pagarles una cuota a los políticos corruptos del mundo para que sigan apoyando el verso climático, seguro que cuando les paguen todo vuelve a la normalidad.

      Comparto esta nota como demostración de que un buen escéptico, debe aceptar y analizar todos los datos y todas las noticias, estén a favor o no de sus opiniones, lo que importa no es eso, sino la ciencia y el pensamiento crítico.

    • #30578

      Nature recientemente abrio un nuevo journal llamado Nature Climate Change. En este numero hay muchos articulos gratuitos, no se si va a seguir asi en el futuro.

    • #30579
      N3RI
      Superadministrador

      Científico escéptico del cambio climático mintió al Congreso de EEUU

      http://www.guardian.co.uk/environment/2011/jan/25/michaels-climate-sceptic-misled-congress

      Patrick Michaels, del Cato Institute y conocido escéptico del cambio climático, afirmó ante el congreso que sólo el 3% de su financiación venía de la industria petrolera (4.2 millones de dólares), más tarde reveló que en realidad era un 40%. Ahora, enfrenta una investigación por mentir en el Congreso de Estados Unidos.

    • #30580

      Jajaja. Patrick Michaels también estuvo entre los que negaban que el tabaco produce cáncer. Apoyó el TASSC, una organización creada por las tabacaleras para sembrar duda sobre ese tema. Curiosamente, Frederick Seitz también fue uno de los involucrados con ese ente… la misma persona que empezó la payasada del Petition Proyect buscando firmas de «científicos» (entre comillas porque la definición que usan es increíblemente amplia) que no acepten el cambio climático antropogénico y el fundador del C. Marshall Institute, otro think-tank que actualmente niega el cambio climático y en su tiempo también negó el agujero de ozono, que la lluvia ácida fuera peligrosa y que el tabaco causara cáncer.

      Buenos muchachos.

    • #30581
      Julian R
      Miembro

      Y despues alguien bardea al IPCC. Oh the irony.

    • #30582

      Si sabés inglés recomiendo el libro Merchants of Doubt (Meraderes de la Duda). Es un libro sustancioso aunque un poco pesado con más de 50 páginas de notas para fundamentar sus afirmaciones y escrito por historiadores de la ciencia (Naomi Oreskes y Eric Conway) que relata las acciones de estos desinformadores profesionales. No sé si se consigue en español pero si seguís el link de arriba lo podés comprar barato (desde U$S21) y con envío gratuito desde Better World Books (las ganancias se donan para combatir el analfabetismo).

      Tejo un artículo de La Nación sobre el libro. No leí el artículo completo pero leyendo por arriba parece dar un resumen de el tema central del libro.

    • #30583

      DrGEN publicó en su blog la traducción de una conferencia del presidente checo Václav Klauz sobre los aspectos económicos del Cambio Climático. Es interesante que el tipo habla de «los economistas esto» y «los economistas aquello». ¿Qué piensan realmente los economistas? Sin entrar en mucho detalle así respondieron los economistas más prestigiosos expertos en cambio climático (esta es la fuente)

      6XSH6.png

      Es decir que la mayoría de «los economistas» creen que hay que actuar para reducir las emisiones de gases invernadero. Podemos decir que contrariamente a lo que dice el Sr. Klauz, «los economistas» sí ven «el resultado comparativo de costo-beneficio de las reducciones de emisiónes de CO2 como favorables».

      DrGEN en uno de los comentarios en su blog dice que siempre preifiere «un análisis defectuoso a la falta de análisis». Pero, ¿quién habló de falta de análisis? Los economistas también trabajan y pensar que no estudiaron el tema es ridículo. Hace meses que posteé en mi blog mostrando que los impactos económicos de la mitigación al cambio climático es mínimo aún en los modelos económicos que no tienen el cuenta el costo extra que produciría.

      XbtRm.jpg

      * En promedio, el impacto en el PBI estadounidense sería de menos de un 1% y el crecimiento de su economía se vería afectado en un 0,03%.

      * El desempleo aumentaría un 0,05% acumulado en las próximas 2 décadas. Una figura que sólo considera los sectores existentes actualmente y no tiene en cuenta la creación nuevos rubros de en economía como producto de las nuevas tecnologías.

      * En promedio, el costo por familia se vería reflejado en un aumento del presupuesto familiar menor al 1%. El aumento sería de entre U$S 84 y U$S 160 por año. Para las familias de menores ingresos, el costo sería mucho menor incluso pudiendo tener una disminución de unos U$S 125 según un análisis.

      * El costo de la electricidad, sorprendentemente, podría reducir. Si bien el costo por watt/hora aumentaría la reducción del consumo y los programas de eficiencia energéticas tendrían a contrarrestar el efecto.

      El «declive económico inevitable» que presagia Klauz no es tal e ignora por completo el declive económico y los costos sociales y medioambientales de seguir emitiendo miles de millones de toneladas de gases de efecto invernadero a la atmósfera. Pero claro, ¿qué se podría esperar de alguien que ni siquiera cree que exista tal cosa como el cambio climático antropogénico?

      PD: Como una acotación también es interesante ver que una de las referencias hace mención del Hartland Institute. Este «tink-tank» es otro de los que recibió dinero de las tabacaleras y negó los peligros del tabaco.

    • #30584
      N3RI
      Superadministrador

      del TED que se hizo en argentina.

    • #30585

      Esta es la declaración de Muller, el creador del Berkeley Earth Surface Temperature Project (BEST). Algunas páginas atrás mencionaron que su registro de temperatura, que iba a ser el «mejor» y apoyado por los «escépticos», es preliminarmente idéntico a los registros actuales.

      «We have done an initial study of the station selection issue. Rather than pick stations with long records (as done by the prior groups) we picked stations randomly from the complete set. This approach eliminates station selection bias. Our results are shown in the Figure; we see a global warming trend that is very similar to that previously reported by the other groups.

      We have also studied station quality. Many US stations have low quality rankings according to a study led by Anthony Watts. However, we find that the warming seen in the “poor” stations is virtually indistinguishable from that seen in the “good” stations

    • #30586
      DrGEN
      Miembro

      La prestigiosa (?) y reciente Nature Climate Change (que tanto le gusta a Daneel) publicó un aterrador estudio sobre la influencia del cambio climático en unos peces:

      http://www.nature.com/nclimate/journal/v1/n2/full/nclimate1084.html

      Lo que se olvidaron de decir es que el tamaño de la muestra del estudio era de… sólo 2 peces!

      http://wattsupwiththat.com/2011/06/04/study-based-on-two-fish-sample-size-challenges-to-australias-climate-comission-go-unanswered/

      Prestigiosa… sí, claro.

      En otro caso que demuestra cómo se utilizan estos mediocres estudios para alarmar a la gente, The Independent reproduce un estudio de la U. de Bristol destinada a asustar a niños: El cambio climático está dejando sordo a Nemo!

      http://www.independent.co.uk/environment/nature/global-warming-threatens-fish-that-inspired-finding-nemo-2291517.html

      Lo que tampoco dicen es lo que se aclara en el propio estudio (negritas mías):

      «The ability of fish to adapt to rapidly changing conditions is not known. Dr Simpson said: “What we have done here is to put today’s fish in tomorrow’s environment, and the effects are potentially devastating. What we don’t know is whether, in the next few generations, fish can adapt and tolerate ocean acidification. This is a one-way experiment on a global scale, and predicting the outcomes and interactions is a major challenge for the scientific community.”»

      Consideraciones que a nadie interesa… el miedo vende más!

    • #30587

      No, lisandro. Lo que se basó en 2 peces eran las medidas de velocidad y esas cosas y el mismo artículo expresamente decía que esos resultados eran para tomarlos con pinzas (preliminares). De hecho, (y cito el artículo de WUWT) el propio paper decía:

      «This result may reflect the small sample size of our experiments, and further work is needed to determine the effect of increasing temperature on swimming activity in banded morwong». Difícilmente es evidencia de malapraxis científica que los propios autores admitan las limitaciones de su estudio.

      Que cagada que ahora el paper esté detrás de la pared de pago.. cuando yo lo leí era de acceso gratuito. Los datos que apoyaban la conclusión principal del estudio (que un segmento de la población de esos peces estaban disminuyendo su tamaño según si estaban por encima del nivel óptimo de temperatura) se basaban en, si no me equivoco, cientos de muestras. [img]http://lh4.ggpht.com/_ayOut9U6i70/Tb3eex0XFdI/AAAAAAAABZM/PkAIorUhaK0/nclimate1084-f1%20%281%29_thumb%5B2%5D.jpg?imgmax=800[/img]

      EDIT: Encontré un pdf del paper.

      Y el segundo ejemplo es más de lo mismo. En vez de demostrar el uso de malaciencia, apoya lo contrario. Los investigadores en su propio paper admiten las limitaciones de su trabajo. No hay nada para ver acá más que un enorme y gigante espantapájaros que la tribu de WUWT quiere derribar.

      Que la prensa generalista lo simplifique y hasta lo distorsione no es novedad. De hecho, la prensa generalista es muy culpable de todo este problema al promover la idea de que todavía existe un debate científico entre los climatólogos sobre la realidad y causas del cambio climático… todo en el interés del falso balance.

      Climate-Change-Infographic.jpg

      Porque además del miedo también hay otra cosa que vende: las controversias (aunque sean manufacturadas) y las teorías conspirativas.

    • #30588

      Sigamos con la desinformción que hay en los medios: a pesar de que la mayoría de los economistas apoyan la reducción de las emisiones de CO2 (ver post anterior) y la mayoría del público parece apoyar las regulaciones de la EPA, los medios presentan la opinión de «expertos» claramente sesgada hacia el bando contrario:

      TJUmx.png

      Fuente de todo eso: http://mediamatters.org/research/201106070010

    • #30589
      Anónimo
      Inactivo

      Hace tiempo vi por Internet un dosumental que supuestamente refutaba la evidencia del calentamiento global. El documental se llama The Great Global Warming Swindle.

      Está publicado en youtube acá:

      Pero lo que muchos de los defensores del documental no saben es que ya fue refutado punto por punto. En un doumento presentado ante la ONCOM (el organismo brítanico que se encarga de la regulación de medio) preparado por cientificos y sometido al proceso de revisión por pares (el pseudodocumental no lo fue) muestra muchisimas mentiras y distorsiones de ese programa de televisión. El documento completo está publicado en esta dirección:

      http://www.ofcomswindlecomplaint.net/FullComplaint.pdf

      Recomiendo que quienes se animen a ver el «documental» no dejen también de leer el documento (muy detallado e interesante). Aparte de refutar los argumentos de los «escepticos» del calentamiento global, se documenta quienes son quienes patrocinan a estos «escepticos». Muchos de ellos pertenecen a instituciones financiadas por compañías petroleras, por ejemplo Exxon.

    • #30590

      Buena onda aunque no sé si me da para leer una respuesta de más de 100 páginas de una :S.

      Como una nota de color, el team de starviewer tampoco cree que exista el calentamiento global y en este post dicen que no hay correlación entre el CO2 y la temperatura. Es todo por las manchas solares, mijo.

      Lisandro no está en buena compañía XD.

      PD: Para los que no entienden el chiste, Starviewer es una de las webs de habla hispana más magufas que uno pueda encontrar. Acepta TODA afirmación pseudocientífica desde predicciones apocalípticas y teorías conspirativas hasta la existencia de aetherofactos y la «teoría de las tres en raya de las placas tectónicas». Además no acepta la evolución y cree que los homosexuales y judíos controlan el mundo. Les recomiendo leer La mentira está ahí fuera que se la pasa dándole masa a éste y otros lunáticos de la misma calaña.

    • #30591
      DrGEN
      Miembro

      Elio (AKA Daneel):

      Por suerte esa no es mi compañía…

      aunque estos que seleccionan los papers para lograr el «consenso» sí son la tuya:

      http://wattsupwiththat.com/2011/06/09/lindzen-on-getting-the-special-treatment-for-publishing-papers/

    • #30592

      Ah, genial. El rechazo de un paper es evidencia de que toda la climatología está corrupta. Ninguno de los referees consideró que el paper era apto para publicación:

      6VjJP.png

      Para colmo, el paper luego sí fue aprobado en otro journal. ¿Esto no sería evidencia de que NO hay una conspiración?

      No le estoy dando mucha bola al tema pero cabe mencionar el paper Said y Wegman, ¿no? Uno en el que los autores acusan a la comunidad de climatólogos de prácticas poco éticas. Irónicamente ese paper fue retractado recientemente por demostrarse que varios párrafos eran copipaste de wikipedia. También muchos consideran sospechoso que haya sido aceptado para la publicación sólo 6 días después de ser recibido y que los registros de la revisión por pares, según el editor del journal, se perdieron cuando se mudaron de oficina. Más información acá y acá (de nuevo, yo no lo seguí mucho así que puedo haberme equivocado en algún que otro detalle))

    • #30593
      N3RI
      Superadministrador

      además de lo obvio de que que no acepten un único paper no implica nada ni prueba ninguna conspiración (y encima en otro lado si lo aceptaron), lo que no entiendo es… ¿cuál es la acusación?

      La frase «me dieron el tratamiento especial» suena grave, pero no dice nada. Me llama la atención que se quejen de trato diferente, pero no aclaran cuál fue dicho trato, porque en realidad no hay ninguno.

      Si lo leyeron, la fuente, el intercambio de mail, ocurrió más o menos así:

      1. este paper no era una investigación original, sino que es una especie de «respuesta» a las fuertes críticas que había recibido un paper anterior (que sí se lo publicaron, y que es el que realmente presentaba la postura y los datos). O sea, el paper (el original) sí fue publicado.

      2. el «tratamiento especial» cuál fue? pues que detectaron que los dos reviewers que proponían los autores… uno no era experto en el tema (condición necesaria para que se acepte como reviewer) y el otro… es un colaborador directo del autor, que publicó junto a él en varias oportunidades… o sea, su amigote de toda la vida.

      3. le dicen amablemente que el paper parece interesante, pero que lo más justo es cambiar de reviewers, le proponen 4 posibles candidatos. El autor, los rechaza, abiertamente por el simple hecho de ser «alarmistas» y propone 4 candidatos propios, abiertamente aclara que los elige porque son (ya saben qué palabra estoy pensando, pero digamos no-alarmistas). Se los aceptan, contactan con los 4, 2 no pueden porque no sé qué cosas de ellos y le aceptan los otros 2. Le proponen completar con 2 más, los cuales fueron propuestos por los 2 que él mismo propuso, y le preguntan amablemente si tiene alguna objeción, dice que si, porque también «son alarmistas» y propone otro, se lo aceptan. y así quedan 4 reviewers nuevos: 2 abiertamente no-alarmistas (según las propias palabras del autor) 1 que considera «agnóstico» en el tema y 2 propuestos por la organización…

      4. después de un tiempo, recibe una carta en la que le dicen que fue rechazado el paper, le muestran las revisiones y le piden que haga las modificaciones pertinentes, porque el tema era interesante, y que lo vuelva a mandar.

      5. él mismo, decide no volverlo a mandar. Hace las modificaciones que le pidieron pero no se los manda de nuevo, sino que sólo les contesta la carta. Manda el paper modificado a otra organización, que sí se lo acepta.

      Fin del cuento.

      O sea, simplemente le dijeron «no, un paper revisado por dos amigotes tuyos no te vamos a aceptar», eligieron 4 nuevos, 2 elegidos/propuestos por él mismo, y 2 que le propuso la junta de revisión y él aceptó. Los 4 rechazan el paper (incluídos los que él propuso). Lo invitan a modificar los puntos flojos y a enviar la nueva versión del paper pero no lo hace. En otro lado se lo aceptan (el paper revisado).

      ¿Eso es todo? Y dónde está la demostración de conspiración? Lástima que no lo volvió a mandar. Lástima que no eligió mejores revisores desde un principio. Lástima que no aclara que el paper original sí fue publicado, y que la versión revisada de este también fue publicado. Lástima que se trata de un caso aislado. Lástima que no aclara que no hay ninguna irregularidad de parte del PNAS y varias del autor. ¿Entonces?

      Pero claro, decir «los conspiradores alarmistas del cambio climático no aceptan papers para revisión y nos tratan de forma especial» suena requete grave.

    • #30594
      DrGEN
      Miembro

      Que la climatología está corrupta no está en duda, ya se probó eso en el Climagate.

      Si demuestra algo lo que puse es cómo se sigue manteniendo el «consenso».

    • #30595

      Chau, lo perdimos en el agujero negro de la conspiranoia.

    • #30596
      N3RI
      Superadministrador

      Repito, ¿qué es lo que se supone que demuestra ese link?

      ¿Hace falta aclarar que el tipo se está quejando de que no lo dejaron elegir reviewers favorables, que a eso llama «trato diferente», cuando es más que obvio que los escritores de los papers NO PUEDEN elegirlos? Y que encima, sí se los dan a elegir, le preguntan todas las veces si le parece bien este o aquel, cuando eso en general no se hace?

      ¿Hace falta aclarar que al final, eligió directamente 2 de los 4 reviewers y 1 indirectamente… y que los 4 le rechazaron el paper?

      ¿Hace falta aclarar que le dijeron que lo modifique y lo mande de nuevo y no lo hizo porque no se le cantó?

      Repito, ¿qué es lo que se supone que demuestra este link?

      Es evidente que es un caso aislado, sacado de contexto, exagerado y absurdo con la única finalidad de poner el pomposo título «uh, miren cómo el consenso se mantiene rechazando MIS papers».

      Cuando la verdad es que:

      1. el paper original NO fue rechazado, fue publicado

      2. se lo rechazaron 4 reviewers, 2 de los cuales… los eligió directamente él

      3. no es común que uno pueda elegir quién revise sus papers, y el caradura exigía que se lo revise su amigote de toda la vida

      4. le dijeron que modifique y mande de nuevo, y no lo quiso hacer

      y lo más importante, para entender el contexto:

      5. el paper no era una publicación original (la cual SI se la publicaron) sino una supuesta respuesta a las críticas que recibió el original. Y todos los reviewers coincidieron en que NO respondía las principales críticas, las ignoraba y esquivaba. POR ESO se lo rechazaron.

      Repito: ¿Qué se supone que demuestra este link? y cómo se supone que lo demuestra?

    • #30597
      DrGEN
      Miembro

      Nada de conspiranoia Elito, mirá:

      http://motls.blogspot.com/2011/06/editors-reviewers-and-bias.html

      Acá el escéptico soy yo, y el religioso sos vos.

    • #30598

      Ajá.. Un blogpost de lo mismo que comentaste antes. Se…

      Quote:
      What I find frustrating is that there’s clearly no solution that would use the same set of climate scientists that exists today. (…)The climatology community is clearly screwed these days. Whatever rules you will impose, it will ultimately be the case that the dishonest and deluded majority will be heard and will be able to strengthen their position further, by increasing rejections of the papers and the people who don’t pay lip service to the party line.

      (…)

      Climatology has entered a vicious cycle because the percentage of dishonest and fanatical members of the climatological community has exceeded a certain critical mass. Above this critical mass, the mechanisms guarantee that what the research is converging to is not the truth as reflected by the empirical evidence but rather the perfect partisanship and universal parroting of the Gore-style lies about a coming judgement day.

      BAM! Así es como se llega a mantener una posición completamente infalseable. Cualquier evidencia a favor del cambio climático puede ser rechazada como parte de la conspiración. Más aún, significa que publicaciones en posts escritas por físicos teóricos sobre climatología son más relevantes que las publicaciones en journals de los científicos climáticos.

      Ya que hablaste de religión, cualquier semejanza con el movimiento creacionista no es pura coincidencia. Sólo les falta que hagan lobby para «enseñar la controversia»… ah no, ya lo están haciendo.

      Y porque no estaríamos completos sin una violación de la Ley de Godwin:

      Quote:
      Nazi Germany has never become so capable of suppressing its «contrarians» as the modern Alarmist Climatology.

      (seguido por ¡4 párrafos! de comparación)

      En fin, el rechazo de un paper que el autor no corrigió y que luego fue aceptado en otro journal es evidencia de una conspiración entre el 97% de los climatólogos y la totalidad de las academias de ciencias de nivel nacional e internacional… sólo para quien ya creía en eso de antes. Por supuesto, esta es la única posición aceptable para alguien cuyas opiniones sobre hechos científicos son diametralmente opuestas a las de la comunidad científica (ya lo veo a DrGEN buscando un link al Petition Project). O son todos incompetentes o son nazis malvados que conspiran para quitarnos las libertades.

      KLd33.jpg

    • #30599
      N3RI
      Superadministrador

      He notado que es re fácil correr con preguntas a los negacionistas, en todos los foros noto que sólo se dedican a tirar frases conspiranoicas y links a blogs de ellos mismos escribieron y que si les hacen preguntas, no contestan y desaparecen (y si vuelven, no es a contestar, sino a seguir tirando links que no demuestran nada).

      Por eso se me ocurrió que podríamos escribir algo así como «15 preguntas para correr a los negacionistas del cambio climático». Y también podríamos escribir (aunque seguro ya hay en inglés) «15 respuestas típicas a las principales frases negacionistas». Porque son re típicas sus frases, siempre las mismas.

      Podríamos hacer lo mismo con negacionistas de la teoría de la evolución, de paso.

    • #30600

      Si te gustaron los mails que salieron a la luz con «climategate«, no te podés perder las amenazas que reciben los climatólogos:

      Quote:
      arfgh shaddup you fckn wanker…go push you your yuppie bllshit propaganda to your useful idiot mates and shut the fck up…

      [clipped] how the fck do you know that? how old is our continent? what and how many extreme rain events have their been in the last 250 million years…yr a dckhead-and they made you a professor-wot out of a fckn cornflake packet?

      Quote:
      You [clipped] are nothing but a Traitorous Lying Asshole !!

      The quicker that Cunts like you and your kind Die the better !!!!!!!

      Quote:
      …what a joke you wankers are! There will be a day of facing the music for the [clipped] type frauds. What a fucking idiot, the last decade has been the hottest in recorded history….. [clipped] you are a fucking fool!

      En noticias relacionadas, cliatólogos australianos buscan protección policial tras ser amenazados de muerte.

      Quote:
      The Australia National University (ANU) in Canberra said that it has moved a number of its climate scientists to a secure facility after they received a large number of threatening emails and phone calls.

    • #30601
      N3RI
      Superadministrador

      «Ya no hay ninguna posibilidad de que se frene el deshielo del Ártico». Así de contundente se mostró ayer Carlos Duarte Quesada, uno de los mayores expertos mundiales a nivel global y coordinador de la expedición científica Malaspina, el mayor proyecto científico que desarrolla España, con la participación de más de 400 científicos. El biólogo profesor de investigación del CSIC ofreció ayer en A Coruña la charla Malaspina 2010, la mayor aventura de la ciencia española, organizada por la Fundación Caixa Galicia.

      http://www.lavozdegalicia.es/sociedad/2011/06/14/0003_201106G14P36991.htm

    • #30602

      ¿Qué onda lo del oxígeno y la rediación ultravioleta?

    • #30603
      Suyay
      Participante

      Yo no entiendo mucho del tema, pero no se supone que estamos todavía saliendo de la ultima era del hielo? Es lógico que las temperaturas aumenten, por mas que las hayamos acelerado o no por la contaminación. Que hay contaminación es seguro. Y que hay calentamiento también. Pero que una cosa sea la estricta causante de la otra parece que esta en discusión todavía.

    • #30604

      Suyay dijo:

      Yo no entiendo mucho del tema, pero no se supone que estamos todavía saliendo de la ultima era del hielo? Es lógico que las temperaturas aumenten, por mas que las hayamos acelerado o no por la contaminación. Que hay contaminación es seguro. Y que hay calentamiento también. Pero que una cosa sea la estricta causante de la otra parece que esta en discusión todavía.

      Sí, seguro que no. :P

      Las evidencias detrás del calentamiento global antropogénico (AGW) van MUCHÍSIMO más allá de la simple observación de que tanto la temperatura como el CO2 estan aumentando. Entendemos bastante sobre los procesos causantes del AGW. Observaciones directas de satélites y estaciones en el suelo muestran que hay menos radiación escapando al espacio y volviendo hacia la Tierra. La disminución está centrada en el número de onda 667 1/cm que coincide con la banda de absorción del CO2. También se observa que mientas la tropósfera (baja atmósfera) se está calentando, la estratósfera (alta atmósfera) ¡se está enfríando! Esto es una huella innegable que indica un calentamiento por efecto invernadero: el aumento de CO2 absorbe la radiación infrarroja que escaparía al espacio y calentaría la estratósfera.

      Y también sabemos que el aumento del CO2 es producto de las acciones humanas. Una forma es matemática: emitimos más de 30 mil millones de toneladas anuales de carbono esto es, si mal no recuerdo, 100 veces más que los emitidos por la actividad volcánica. Algunos argumentan que 30 mil millones de toneladas no es nada comparado con las emisiones de las plantas y los océanos pero eso es sólo la mitad de la historia. La absorción natural de CO2 compensa de sobra las emisiones naturales por lo que las actividades humanas resultan en un balance neto positivo. De hecho, si no fuera porque los océanos están absorbiendo aproximadamente un 55% de las emisiones humanas estaríamos mucho más en el horno. (La absorción oceánica de CO2 se evidencia en la acidificación oceánica, el CO2 al diluirse en el agua produce ácido carbónico). También se observa una disminución en el Oxígeno molecular en la atmósfera, lo que es consistente con un aumento del CO2 producto de la combustión de combustibles fósiles (el O2 en el CO2 tiene que venir de algún lado).

      Otra forma de atribuir el CO2 es mediante el análisis de los isótopos de carbono.

      «El Carbono tiene tres isótopos distintos: 14C, 13C y 12C, siendo éste último el más común. Al ser distintos atómicamente, también reaccionan químicamente de forma diferente. Así, las plantas tienen más facilidad para absorber los más livianos por lo que su relación 13C/12C es menor que la de la atmósfera (un 2%). Como los combustibles fósiles son esencialmente plantas prehistóricas, éstos tienen esa misma composición. Usando anillos de árboles y núcleos de hielo, podemos reconstruir la relación isotópica en la atmósfera través del tiempo. Los resultados son claros: la relación 13C/12C disminuyó un 0,15% desde 1850. Aunque esto suena poco, no lo es; comparado con la diferencia entre el período glacial y el interglacial es de 0,03%.»

      (fuente: un post mío :P).

      Finaaalmente (mis disculpas por la longitud), está el tema de que los modelos climáticos que no incluyen factores antropogénicos son incapaces de reproducir la tendencia positiva.

      LLxWf.pngsmiJK.pngSkEfM.png

      (imágnes robadas de Homínidos: Que no te engañen, no es el Sol el causante del cambio climático que él robó del IPCC :P)

      La línea negra son los datos observados, lo rojo son los resultados de los modelos que incluyen el forzamiento antropogénico y lo azul son los que sólo tienen el forzamiento natural.

    • #30605
      Suyay
      Participante

      Bueno, parece bastante convincente. Pero bueno, igual un día el sol se va a apagar y se termina el calentamiento global :P

    • #30606
      Malena
      Participante
      Quote:
      Bueno, parece bastante convincente. Pero bueno, igual un día el sol se va a apagar y se termina el calentamiento global

      Los soles suelen terminar con un par de explosiones. Así que el planeta se va a rostizar. ;P

    • #30607
      DrGEN
      Miembro
      Quote:
      He notado que es re fácil correr con preguntas a los negacionistas,

      En mi caso, soy un escéptico, pero opté por no responderles porque no entienden ni siquiera ese punto básico: la diferencia entre un escéptico y un negacionista.

      (para ser completamente sincero, ví ese comentario de casualidad ya que ni me gasto en leer lo que ponen)

    • #30608
      Suyay
      Participante

      Malena dijo:

      Quote:
      Bueno, parece bastante convincente. Pero bueno, igual un día el sol se va a apagar y se termina el calentamiento global

      Los soles suelen terminar con un par de explosiones. Así que el planeta se va a rostizar. ;P

      Ja, es verdad, pero me cagaba el chiste :P

    • #30609
      N3RI
      Superadministrador

      DrGEN dijo:

      Quote:
      He notado que es re fácil correr con preguntas a los negacionistas,

      En mi caso, soy un escéptico, pero opté por no responderles porque no entienden ni siquiera ese punto básico: la diferencia entre un escéptico y un negacionista.

      (para ser completamente sincero, ví ese comentario de casualidad ya que ni me gasto en leer lo que ponen)

      Todos somos escépticos acá, Lisandro. Otra vez subestimando la gente con tus «no entienden». ¿Qué te hace pensar que en un «foro de escépticos argentinos» nadie, excepto vos, entiende la diferencia entre escéptico y negacionista? Es absurdo que afirmes tal cosa, todos la entendemos perfectamente, somos gente muy lista y muy bien informada. ¿Vos entendés la diferencia entre un escéptico y un religioso? (digo, por lo que le dijiste a Daneel.

      Igual yo no hablaba de vos, «tu caso», tengo entendido, no es el caso de un negacionista, además, no importa «tu» caso (no estamos debatiendo sobre vos, sino sobre el cambio climático; aunque vos no aparezcas el debate va a continuar).

      Yo estaba hablando directamente de los negacionistas. Si vos, o cualquiera se siente identificado con esa descripción, o se comporta de esa manera en que se comportan los negacionistas, probablemente lo sea.

      Está muy claro cómo se comportan los negacionistas en un debate, y de eso hablaba yo en ese comentario, justamente: de la diferencia entre un verdadero escéptico y un negacionista.

      – Como ya dije, el negacionista evita responder las preguntas que se le hace durante el debate.

      – Como dije, el negacionista en vez de concentrarse en los datos duros y principales… plantea el tema desde los detalles de menos importancia o secundarios (que si un climatólogo está de novio con la hija de un fabricante de paneles solares, que si no consigo publicar un miserable paper, que si encontré un par de mails en los que dos científicos dicen que odian a los negacionistas, que si me llaman negacionista me ofendo, que si hizo frío en Helsinki)

      – Al escéptico le interesa más el tema del debate que los improperios, contesta lo primero e ignora lo segundo. El negacionista hace al revés, se dedica más a atacar a los demás participantes del debate de forma personal, y si aparece a contestar algo dicho, no es para contestar algo del tema, sino para contestar algo que cree que dijeron sobre él.

      – El escéptico pregunta y está abierto a contestar todas las preguntas que se le hagan.

      – El escéptico si no sabe algo, lo admite y busca información al respecto. Y si hay algo que no está del todo claro en la actualidad, lo admite abiertamente, como parte de las características del sistema complejo estudiado. El negacionista no contesta.

      – El escéptico cita como fuente papers, o al menos estudios independientes y respetables y, como buen escéptico, duda de los que no lo son. El negacionista cita blogs en blogspot.com; generalmente escritos por él mismo y con post sin fuentes.

      – El negacionista duda de los papers, pero no duda nunca de los post escritos en un blog, eso es sagrado.

      – El negacionista cita fuentes escritas por gente que no es especialista en el tema, incluso abiertamente ignorantes o con obvios intereses.

      – El negacionista siempre duda de los intereses de los científicos climatólogos (x ej: obtener financiamiento, del gobierno, para sus investigaciones) pero si apoyan su postura, jamás pone en duda los intereses de «empresarios», «políticos», «instituciones financiadas no por el gobierno, sino por empresas petroleras o de lobbies», etc, .

      – El negacionista siempre subraya el hecho de que los medios exageran a favor del cambio climático. El escéptico en cambio, se da cuenta de que los medios siempre exageran. Punto. Tanto a favor como en contra del cambio climático, les es indistinto, ellos exageran hacia el lado que más llame la atención.

      – El escéptico cita fuentes especializadas, tanto cuando reafirman un dato o investigación, como cuando la ponen en duda. El negacionista siempre, pero siempre cita sólo a otros negacionistas. Si alguien se dice «escéptico» y sus fuentes son exclusivamente negacionistas, no es escéptico, es negacionista.

    • #30610
      N3RI
      Superadministrador

      Me quedó tan largo que tranquilamente podría ser un post titulado: ¿Cómo distinguir en un debate sobre Cambio Climático, entre un verdadero escéptico y un obvio negacionista?

    • #30611

      DrGEN dijo:

      Quote:
      He notado que es re fácil correr con preguntas a los negacionistas,

      En mi caso, soy un escéptico, pero opté por no responderles porque no entienden ni siquiera ese punto básico: la diferencia entre un escéptico y un negacionista.

      (para ser completamente sincero, ví ese comentario de casualidad ya que ni me gasto en leer lo que ponen)

      http://vimeo.com/24669583

      (vía: ¡¡No entienden nada!!)

    • #30612
      Suyay
      Participante

      jajaja!

    • #30613

      Por que el video ese? I don’t get it!!! :P

      Esta muy bueno!

    • #30614

      Miren esto:

      kC2Td.jpg

      Esta es una foto de una presentación del «escéptico» Lord Christopher Monckton. Ley de Godwin aplica. Entre otras joyitas:

      Quote:
      The conference was organised by the American Freedom Alliance, a think-tank which is currently involved in a long-running legal battle with a California science education centre. The AFA wanted to screen a documentary which featured scientists attacking Darwin’s theory of evolution in favour of intelligent design, but the education centre cancelled the screening.

      (fuente: http://www.abc.net.au/unleashed/2765990.html – Vía: http://scienceblogs.com/deltoid/2011/06/monckton_says_that_if_you_acce.php)

      EDIT:

      Ufff! No sabía esto. Monckton también dice tener una cura para el cáncer, los resfríos, la gripe, la esclerosis múltiple. Y en su bio en el Science & Public Policy Institute afirma haber ganado el Premio Nobel de la Paz.

    • #30615
      N3RI
      Superadministrador

      seguro que tampoco lo dejan publicar tranquilo sus papers.

    • #30616

      Y, carece por completo de publicaciones científicas así que es posible.

    • #30617

      Otro negacionista diciendo que quienes aceptan la existencia del cambio climático antropogénico son nazis:

      Quote:
      Those of us who know something about the history realize that Frau Jill Singer’s ideological predecessors have used the very same «trick» in Auschwitz-Birkenau

      Dice Lubos Motl

    • #30618
      N3RI
      Superadministrador

      hoy hace frío, ergo, los climatólogos mienten.

    • #30619
      Carmen
      Miembro

      recién escuché una nota en la BBC, sobre el estado de los océanos. Afirman que se acidifican, y como consecuencia, le complica la vida al plancton. Y así, se anticipan una seríe de consecuencias razonablemente esperadas que, a largo plazo, no pintan bien para la situación del calentamiento global.

      salu2

      carmen

    • #30620

      A la acidificación oceánica es la cara poco conocida del cambio climático y es muy jodida. No se sabe los efectos biológicos que podría tener pero ya se observan que los corales están sufriendo. Hay un aumento en los coral bleachings (blanqueo de coral, según wiki) y en las enfermedades. Es un problema muy grave ya que muchísimas especies dependen de esos ecosistemas.

    • #30621
      DrGEN
      Miembro
      Quote:
      Otro negacionista diciendo que quienes aceptan la existencia del cambio climático antropogénico son nazis:

      Motls «negacionista»??? JAJAJA!

      Y después N3RI dice que conocen la diferencia entre negacionista y escéptico.

      Bue, en todo caso tal vez él la conozca, pero evidentemente Daneel no tiene idea.

      De hecho, parece molestarle que alguien sea escéptico en este tema.

      Y es entendible, Daneel es como un ateo pero que sigue creyendo en Zeus y se molesta cuando los demás ateos apliquen el mismo razonamiento que él, sólo que a todos los dioses…

    • #30622

      N3RI dijo:

      hoy hace frío, ergo, los climatólogos mienten.

      Parece joda, pero hay gente que cree que ese razonamiento es correcto:

      Quote:
      Comienza el verano en el hemisferio Norte y el crudo invierno en el hemisferio sur donde justo al comienzo ya se baten récords de frío y de zonas de nevadas donde no lo hacia desde hace 100 años, por tanto no es de extrañar que la temperatura media del planeta siga bajando, le pese lo que le pese a los calentologos y a su cada vez peor negocio del CO2 o bonos de Carbón

      Vía: EL FRAUDE DEL CAMBIO CLIMÁTICO: Argentina: La ola de frío polar no cesa.

      Y no es la primera vez que este tipo comete el mismo error: http://fraudecambioclimatico.blogspot.com/2011/06/demasiada-nieve-en-california-primavera.html

      @DrGEN

      Qué lástima que tus intervenciones en este thread hayan descendido al nivel de quejarse de la terminología en vez de realizar una buena discusión. En este post hablé de algunas de las varias líneas de evidencia que nos indican que el calentamiento global es principalmente antropogénico… un espacio rico en blancos para refutar.

      Si realmente querés que hablen de «escépticos» del cambio climático, entonces comportate como uno y elaborá argumentos. No hace falta que los postees acá, te acepto que los publiques en tu blog (en vez de limitarte a pequeños chascarrillos sin demasiada substancia) y NO censures mis comentarios en él ( 😉 ).

    • #30623
      N3RI
      Superadministrador

      hoy hace más frío que el otro día, ergo, la tendencia en el clima es a que haga cada día más frío. Los climatólogos mienten, pagados por greenpeace y los fabricantes de focos de bajo consumo.

      PD: y si hablamos sobre Cambio Climático? Es mucho pedir?

    • #30624
      ryomashi
      Miembro

      Siempre que veo gente hablando sobre el origen antropogénico del calentamiento global, aparecen siempre con la pizarrita. En la pizarrita hay dos líneas. Una azul que representa cual hubiera sido la evolución de la temperatura normal del planeta basada en factores en los que anda tiene que ver el hombre.

      Otra linea, la roja, representa cual fue la variación de temperatura del planeta. Esta línea, por supuesto, esta mucho más arriba que la línea azul. Como la línea roja empieza a hacer locuras a la altura de la revolución industrial, se deduce que el factor que hizo que se fuera a la mierda fue la acción del hombre.

      Me parece un argumento convincente. Me gustaría saber que opinan los escépticos del calentamiento global.

      Pd: el post esta escrito con respeto y sin segundas intenciones.

    • #30625

      ryomashi dijo:

      Siempre que veo gente hablando sobre el origen antropogénico del calentamiento global, aparecen siempre con la pizarrita. En la pizarrita hay dos líneas. Una azul que representa cual hubiera sido la evolución de la temperatura normal del planeta basada en factores en los que anda tiene que ver el hombre.

      Otra linea, la roja, representa cual fue la variación de temperatura del planeta. Esta línea, por supuesto, esta mucho más arriba que la línea azul. Como la línea roja empieza a hacer locuras a la altura de la revolución industrial, se deduce que el factor que hizo que se fuera a la mierda fue la acción del hombre.

      Me parece un argumento convincente. Me gustaría saber que opinan los escépticos del calentamiento global.

      Pd: el post esta escrito con respeto y sin segundas intenciones.

      Me imagino te referís a estas, ¿no?

      LLxWf.pngsmiJK.pngSkEfM.png

      (imágnes robadas de Homínidos: Que no te engañen, no es el Sol el causante del cambio climático que él robó del IPCC :P)

      Me imagino que te dirán que estas figuras se basan en modelos computarizados (lo cual es verdad) que no son fiables (lo cual es básicamente un juicio de valor).

    • #30626
      smaga
      Miembro

      Hola, comparto el argumento de la gente del Servicio Metereológico Nacional (en la sección climatología), considerando al cambio climático antrópico como una hipótesis.

    • #30627

      Sí, es bastante pedorra esa página, si te soy honesto. Que traten el calentamiento global antropogénico como una «hipótesis» que «de ser comprobada» traería consecuencias serias es, la verdad, una muestra de lo atrasado que está este país en materia científica en algunas cosas. Especialmente considerando que desde 2005 que no se actualiza. Considerando que no hay ni una academia nacional de ciencias que niegue la veracidad del calentamiento global antropogénico y que muchísimas de ellas tienen posiciones escritas que expresamente la apoyan, que el SNM diga eso es un hazmerreír para el resto del mundo.

      Un poco más en serio y dejando la hipérbole de lado, la información que dan es bastante correcta pero el spin que le dan me parece que da para la confusión. Además tiene muchas palabras claves del negacionismo. No sólo el tema de la «hipótesis que debe ser comprobada» sino cosas como que el CO2 es «natural» o un «gas minoritario». Dicen que «i bien este calentamiento es totalmente consistente con la idea de un efecto de invernadero potenciado, no es posible aún identificar la contribución al mismo de las variaciones climáticas naturales y antrópicas, supuesto que estas últimas ya existan» cuando en realidad desde 1950 puede observarse el factor antrópico con claridad (como muestran los gráficos en mi post anterior)

      Por otro lado, también tienen lenguaje que es bien claro: «No obstante, si bien existen incertidumbres, el conocimiento actual es suficiente para afirmar que el riesgo es genuino y serio» o «(…) las actividades humanas están cambiando paulatinamente la composición química de la atmósfera. Si este proceso antrópico continúa sin control, esto podría dar lugar a un cambio climático global con consecuencias muy importantes para la vida» o «El potencial cambio climático antrópico, de seguir las emisiones de los gases de invernadero al ritmo actual, podría dar lugar a una temperatura media global en superficie superior a la ocurrida en los últimos 150.000 años. La velocidad de cambio de la temperatura podría superar a la ocurrida en los últimos 10.000 años. El aumento del nivel medio del mar sería 3 a 6 veces más rápido que lo ocurrido en los últimos 100 años.»

    • #30628
      smaga
      Miembro

      Si decís «…cuando en realidad desde 1950 puede observarse el factor antrópico con claridad (como muestran los gráficos en mi post anterior)..», no creo que sería tan relevante que no se actualize la página desde el 2005.

      Yo pienso que es acertado (por lo tanto no serían un «hazmerreir») citar al cga como una hipotesis, ya que la discusión todavía permanece abierta; aún, cuando sí hay instituciones con posiciones alternas.

    • #30629

      Es que no es cierto que «la discusión todavía permanezca abierta». El consenso científico sobre el CGA es vasto. El 97% de los climatólogos coincide con que el calentamiento global existe y es producido por el hombre. Repito: NO HAY UN VERDADERO DEBATE CIENTÍFICO SOBRE EL TEMA. Toda la controversia sobre el CGA es a nivel político y social y NINGUNA academia de ciencias de nivel nacional o mayor niega el CGA. ¿Que hay algunos científicos con posiciones disidentes? Seguro, pero eso pasa en todos los campos. ¿Hay biólogos, químicos, y demás científicos que niegan la teoría de la evolución? Sí, pero eso no significa que haya un debate científico.

      El link que mandás no es hacia la posicióń de una institución sino un abstract de una presentación en un congreso. Ni siquiera es una publicación en una revista y no tuvo casi revisión por pares. Pero ¿qué es lo que dice?

      Quote:
      El análisis numérico de la radiación solar holocena mediante el contenido de 14C permitió visualizar enfriamientos a escala global con una periodicidad de aproximadamente 2250 años, los que salvo para la Pequeña Edad de Hielo, coinciden temporalmente con la ocurrencia de las oscilaciones del NMM descritas. Ello indica que hoy nos encontraríamos casi a mitad de camino hacia el próximo pico “caliente”, que ocurrirá dentro de aproximadamente 600 años. Ello es una fuerte evidencia a favor del origen natural para el cambio climático que se registra en la actualidad

      Es decir, encontraron una «periodicidad» en el nivel del mar y la actividad solar y dentro de 600 años nos tocaría una alta radiación y alto nivel del mar. Su extrapolación a que esto tiene ALGO que ver con el actual proceso de calentamiento global no está fundamentada. Para empezar, las temperaturas están ACTUALMENTE mucho más altas de lo que estuvieron en los útlimos miles de años y MUY por encima de la variabilidad natural.

      0u9qn.gif

      Y si el CGA sigue su curso, se espera un aumento de entre 2º y 5º para 2100.

      KScQa.gif

      ¿Y se supone que ese calentamiento va a seguir hasta su máximo en… 2700? ¿Y me está diciendo que esto tiene una periodicidad de… 2250 años?

      Pero para peor, lo que están proponiendo es básicamente que el Sol es el responsable. Pero esto es imposible. No sólo la actividad solar no está aumentando sino que los patrones que se observan son indicativos de un calentamiento por efecto invernadero.

      Observaciones directas de satélites y estaciones en el suelo muestran que hay menos radiación escapando al espacio y volviendo hacia la Tierra. La disminución está centrada en el número de onda 667 1/cm que coincide con la banda de absorción del CO2. También se observa que mientas la tropósfera (baja atmósfera) se está calentando, la estratósfera (alta atmósfera) ¡se está enfríando! Esto es una huella innegable que indica un calentamiento por efecto invernadero: el aumento de CO2 absorbe la radiación infrarroja que escaparía al espacio y calentaría la estratosfera.

      El CGA es una teoría sólida y bien fundamentada que comenzó incluso antes de la publicación del Origen de las Especies. En 1859 John Tyndall ya había descubierto que el CO2 era un gas de efecto invernadero y también por esa época se comenzó a especular sobre el efecto climático de la combustión de Carbón. Más cercano en el tiempo, hace más de 30 años que la comunidad científica terminó de discutir sobre teorías alternativas. El trabajo en cambio climático es en los detalles. En calcular exactamente la magnitud de los cambios, su distribución geográfica y temporal y demás cuestiones de segundo orden.

    • #30630
      Suyay
      Participante

      Yo me acuerdo que no hace mucho tiempo había visto a una Dra. que creo que era profesora en la UBA o al menos egresada de la UBA y proponía que tal vez el calentamiento global no era del todo por culpa del hombre, sino porque todavía estamos saliendo de la era del hielo. Eso me trastoco un poco los esquemas que tenia hasta el momento y me hizo dudar un poco. Ahora, no digo que no exista el calentamiento global y que no sea causado por el hombre. Pero tengo algunas preguntas al respecto:

      -¿No se tienen en cuenta los demás parámetros?

      -¿Quién fue que empezó a cuestionar el tema?

      -¿Las consecuencias son realmente tan nefastas como vaticinaban hace unos años? ¿No es acaso peor el daño producido por la contaminación ambiental (aparte del CO2)?

    • #30631
      N3RI
      Superadministrador

      – si, obviamente se tienen en cuenta los demás parámetros, sino no sería ciencia

      – no lo sé en concreto

      – si, es posible que lo sean (no sabremos realmente hasta que ocurran, pero la probabilidad es altísima) La mayoría de los casos de contaminación ambiental, a diferencia del calentamiento global, lo que tienen es justamente eso: «no son globales». Que vos cortes todos los árboles de España, perjudica a España nomás, que haya problemas en un reactor nuclear de Japón, perjudica a Japón nomás, que contamines el riachuelo, perjudica a Buenos Aires nomás. Hay pocos casos de contaminación con la capacidad de hacer daño globalmente. Deforestar todo el amazonas tal vez, eliminar la capa de ozono, matar todo el krill, la gran mancha del Pacífico, un derrame «masivo» de petróleo como lo que ocurrió en el Golfo de México podría ser otro. Pero en general, hay pocos casos que el hombre contamine en unas ciudades y el efecto «se distribuya» a nivel mundial.

      Y de más está aclarar que cualquier científico o persona común preocupada por el cambio climático antropocéntrico también está igual de preocupada por esos otros casos de contaminación ambiental (aparte del CO2). No es un caso de «salvemos a las ballenas, que se jodan los osos panda»

    • #30632
      Quote:
      -¿Quién fue que empezó a cuestionar el tema?

      ¿Te referís a quién empezó a investigar sobre el efecto del CO2 en la temperatura global o quién empezó a cuestionar la ciencia del cambio climático? Para la primera respuesta, te recomiendo ver al menos los primeros 10 minutos de este video de Naomi Oreskes

      y para la segunda, esta entrevista; también con Naomi Oreskes.

      Sé que es una mierda responder con videos pero creo que éstos son mucho más informativos de lo que yo podría ponerlo :P.

    • #30633
      smaga
      Miembro

      Que haya o no consenso sobre algo no es argumento para comprobar una hipótesis científica. En lo que me baso para concluir que la discusión está abierta es en papers (con lo cuál quedan excluídos lo político y lo social) cuyas conclusiones no marcarían al hombre como el principal causante de esta etapa de calentamiento.

      Respecto del link que mandé, creo que su posición está fundamentada ya que si pasó antes, pueda estar pasando ahora; y si miramos gráficos más extensos de el que proponés, se ve claramente su posición (1) (2).

      Viendo esos gráficos, decir «…las temperaturas están ACTUALMENTE mucho más altas de lo que estuvieron en los útlimos miles de años…» creo que no sería correcto.

      Otra cosa, no entiendo que es «…casi revisión por pares…», es o no revisado, pero «casi» es la primera vez que lo veo así.

    • #30634
      saibaba
      Miembro

      Lo que pasa es que hay dos niveles en esta cuestión:

      * El nivel puramente científico,

      * El nivel puramente político.

      Respecto lo político, la pregunta es «¿qué hay que hacer?»

      Lo que hay que hacer depende de la opinión de los científicos, que son los que realmente saben, con cierto grado de certeza, lo que está ocurriendo.

      Si actualmente hay un consenso de 97% de científicos, todos expertos y competentes, de que el calentamiento global es culpa de los combustibles fósiles, entonces eso es lo que hay que tener en cuenta.

      Cuando hay división de opiniones, y tenés que tomar una decisión basada en esas opiniones, ¿a quién vas a escuchar?

      Obviamente a la mayoría, y sobretodo cuando la mayoría es tan abrumadora.

      ¿Por qué escuchar a la minoría? Sólo por capricho, para seguir teniendo ganancias con el petróleo.

      Porque si no fuera por el petróleo, no habría toda esta polémica, y nadie dudaría en tomarse como «buena» la opinión mayoritaria de todos esos científicos.

      Te pongo un ejemplo más concreto.

      Te haces revisar por 100 médicos y 97 de ellos te dicen que tenés cáncer, y los otros 3 te dicen que no, o que no están del todo seguros… o vaya a saber qué.

      ¿Vas a hacer un tratamiento para el cáncer o no?

      Yo me lo haría, y urgente.

      ¿No te parece?

      Esta negación del problema del CO2 me parece un cinismo muy grande.

      Seguramente hay otros factores que influyen, y quizá las cosas no sean tan simples.

      Pero eso «hoy» no se sabe. Lo que se «sabe», con el grado de certeza que la ciencia puede dar, que además es lo único más «certero» que más o menos uno tiene, es que hay que eliminar las emisiones del CO2. O sea, dejar de usar petróleo, y por eso ir en contra de las grandes companías petroleras, y el modo de vida industrial de todos los países, sobretodos los más desarrollados.

      Ése es el problema. Hay que cambiar el sistema del mundo entero si uno quiere salvar el planeta.

      Esto me hace acordar a algunas personas que se tiran las cartas esperando que les salga una carta favorable.

      No sale, entonces se tiran de nuevo, y así hasta que les sale lo que quieren ver u oír.

      Estos hacen lo mismo.

      Especulan con los papers científicos a ver si en algún momento alguno les va a decir que no tienen que dejar de quemar petróleo.

      Pero eso no va a pasar.

      Estuve viendo los videos que colgó Daneel, y las explicaciones de Naomi se ocupan de todos los detalles. Y explica muy bien por qué es el CO2 humano y no otra cosa lo que está ocurriendo.

      Se nota, aunque no sea más que un Powerpoint, que detrás hay un esfuerzo concienzudo para establecer la verdad de los hechos.

    • #30635

      Smaga, te están vendiendo pescado podrido. Esos gráficos no representan la temperatura media GLOBAL. Fijate que el primer gráfico dice expresamente que es la temperatura en Groenlandia. El segundo no lo dice pero también es local. No sólo se nota por la ENORME variabilidad y la gran cantidad de RUIDO en los datos sino porque el eje de ordenadas tiene temperatura absoluta (rondando los -35ºC) en vez de anomalías sobre la media.

      En una corazonada me imaginé que se trataba del GISP2 y sí. En efecto se trata de ese intento por parte de Anthony Watts de hacer pasar la temperatura local como global.

      Incidentemente, el GISP2 es un núcleo de hielo excavado y estudiado por Dr. Richard Alley. Un excelente climatólogo y difusor de la ciencia. Te recomiendo ver su presnetación en la American Geophisical Union. Es muy gacioso y explica muy bien. (si te interesa, la AGU tiene un archivo con un montón de charlas.

      Y hablando de fuentes, yo no usaría WUWT para nada; notorio negacionista con una larga historia de vender ruido como si fueran datos y de apoyar CUALQUIER teoría del calentamiento global con tal de que no implique al CO2. Desde que no existe, hasta los rayos cósmicos pasando por la isla de calor urbano a ciclos astronómicos. El negacionismo de esta gente tiene una característica muy definitoria y es la inconsistencia. No construyen una hipótesis alternativa consistente y sostenida por los hechos sino que tiran mierda y confusión a la vez que sostienen ideas mutuamente excluyentes.

      En cuanto a que «si pasó antes, puede pasar ahora», es una visión extremadamente simplista. ¿Creés que en toda la comunidad de climatólogos no se tuvo en cuenta una posibilidad así? Hay muchísimas líneas de evidencia (algunas de las cuales presenté en ese post) que indican que el calentamiento global NO es natural y NO es causado por el Sol. Pero esa idea igual tiene un grano de verdad. Si es cierto que en el pasado el clima fue más extremo, entonces más razón para limitar las emisiones de CO2, ¡podríamos volver a esas condiciones!

      El clima cambia en respuesta a muchas variables que lo afectan en diferentes escalas temporales. Hay cambios en la órbita de la Tierra y la precesión de los equinoccios, actividad volcánica, cambios en la luminosidad solar, etc… El CO2 no es el único factor importante (aunque Richard Alley argumenta en su charla es que el más importante). La escala de miles de años, como dice la conferencia, no tiene relevancia con lo que pasa con el CO2 que afecta al clima en la escala de varias décadas a cientos de años.

      (Lo que quería decir con que casi no hay revisión por pares es que los requisitos para una conferencia son mucho más laxos que para la publicación de un paper. De hecho, estrictamente hablando no hay peer review en el sentido que se hace en las revistas).

      En cuando al consenso, sí es importante. ¿Vos sos climatólogo? ¿Trabajás en investigación atmosférica? Me imagino que no. Yo tampoco (aunque espero hacerlo en un futuro :D) por lo que es una irresponsabilidad y un arrebato de arrogancia pensar que un lego como vos o yo puede evaluar correctamente la evidencia sobre una cuestión tan compleja como el cambio climático. Sí, el científico trabajando en una disciplina no le sirve que algo sea «consenso» y va a buscar la evidencia bien detallada y criticarla en la literatura científica (publicando papers). Pero el lego que mira todo desde afuera no tiene mucho más alternativa que confiar en las conclusiones de los expertos; y la conclusión en este caso es que el calentamiento global existe y es causado principalmente por el hombre. Si tenés curiosidad y querés saber cómo se sabe eso y cómo se investiga, ¡perfecto! Averiguá e invetigá pero no en blogs negacionistas ideológicos como WUWT que no hacen prácticamente referencia alguna a la literatura científica sino en sitios serios como el de NOAA, la NASA, AGU, Academia Nacional de Ciencias, etc…

      Por ejemplo, si querés datos sobre el paleoclima, ¿por qué no pasarse por el sitio del NOAA a tal efecto?

      Y para una revisión del estado del consenso te recomiendo el artículo de Naomi Oreskes: The Scientific Consensus on Climate Change

    • #30636
      smaga
      Miembro

      Daneel, continuando mi respuesta al mensaje de ayer, mencionás el enfriamiento de la estratósfera como patrón de calentamiento por efecto invernadero, el tema es que desde 1996, esa capa de la atmósfera dejó de enfriarse, mientras que el co2 sigue aumentando.

      saibaba, entiendo, pero el tema es que el consenso es muy cuestionado (1) (2). Además, no nos olvidemos que nosotros, los en-desarrollo, necesitamos (y en mayor medida, creo) de los combustibles fósiles.

      Daneel, me parece que no me están «vendiendo» nada, los datos están y muestran períodos más cálidos como por ejemplo el Período Cálido Medieval.

      Siguiendo con las fuentes, ya tengo mi opinión formada, y al igual que wuwt, skeptical science también tiene sus problemas.

      Respecto de los datos del NOAA, le di un vistazo y la única reconstrucción global de más de mil años es la de Mann.

      Me quedé sin tiempo.

    • #30637
      saibaba
      Miembro

      smaga dijo:

      saibaba, entiendo, pero el tema es que el consenso es muy cuestionado

      Claro, cuestionado por el otro 3%.

      En la ciencia no hay nada seguro, todas las teorías son provisorias, así que siempre va a haber disenso, y siempre va a haber posiciones opuestas.

      Pero cuando se tiene que tomar una decisión en base a opiniones científicas, un consenso de 97% es lo que tiene que inclinar la balanza.

      Seguramente todavía hay quienes cuestionan la relatividad de Einstein, o dicen que la tierra es plana, y hasta los hay que lo gritan fuertemente.

      ¿Y a quién le importa?

      Los argumentos de quienes disienten han sido tomados en cuenta y luego rebatidos, como se ve al menos brevemente en el video de la tal Naomi.

      Y entonces no es un «fuerte» cuestionamiento.

      No estoy en posición de defender más este asunto, dado que de cambio climático la verdad no sé un pomo, y encima me estoy enterando acá en el foro que el asunto ya estaba resuelto científicamente desde los años 90.

      El 97% que tiene «consenso» es digno de confianza, porque no son tipos que se ponen de acuerdo sólo porque «les gusta la idea», sino que todos se han puesto a contestar y rebatir los argumentos de los restantes del 3%.

      Se trata de científicos que han analizado las cosas en detalle.

      En cuanto a que los países en desarrollo dependen del petróleo, y bueno, mala suerte, que se caguen. O mejor, nos cagamos.

      Pero decime una cosa: ¿para qué carajo sirve sostener nuestra economía en base al petróleo si nos vamos a morir todos de cáncer y vamos a reventar como sapos cuando se descalabre todo por el calentamiento global?

      No sé, es lo mismo que abrir una caja de ahorro a los 90 años de edad.

      Te sirve sólo para comprarte el cajón.

      Yo no puedo entender el razonamiento «capitalista» en este asunto, que es de lo más ridículo.

      No tiene ningún sentido conservar el capital para un futuro próximo en el que no vamos a existir.

      Es mejor padecer otra crisis económica que morir en masa como ratas aplastadas.

      No sé, a mi el instinto de supervivencia me puede más que el instinto de ganar unos billetitos más.

      No entiendo cuál es la lógica de hacer justo lo que te lleva directo a morir.

    • #30638

      smaga dijo:

      Daneel, continuando mi respuesta al mensaje de ayer, mencionás el enfriamiento de la estratósfera como patrón de calentamiento por efecto invernadero, el tema es que desde 1996, esa capa de la atmósfera dejó de enfriarse, mientras que el co2 sigue aumentando.

      Sin hacer (ni poder hacer) una crítica del paper o el contexto de la literatura científica, cito del abstract:

      «The reversing trend may relate to a possible recovery of stratospheric ozone concentration»

      El CO2 no es la única variable en juego, por lo que hay que tener en cuenta el peso de TODA la evidencia y ver qué

      ¿Te acordás de la tropopausa? Bueno, como el efecto de los GEI es calentar la tropósfera y enfriar la estratósfera, podemos ver mejor esa huella al analizar la altura de la tropopausa.

      iNYXR.png

      (fuente: Climate Change 2007: Working Group I: The Physical Science Basis 9.4.4.2 Changes in Tropopause Height)

      Está aumentando y es imposible explicarlo basándose en factores naturales.

      Otro razgo distintivo de los negacionismos: la proyección. En los blogs negacionistas se la pasan criticando a los climatólogos argumentando que se centran en el CO2 y no tienen en cuenta otros factoes… Salvo que ellos ignoran todos los demás factores cuando les conviene.

      Quote:
      saibaba, entiendo, pero el tema es que el consenso es muy cuestionado (1) (2). Además, no nos olvidemos que nosotros, los en-desarrollo, necesitamos (y en mayor medida, creo) de los combustibles fósiles.

      Irrelevante a la cuestión. El CCA existe y al CO2 calienta igual sin importarle si los países subdesarrollados lo necesitan o no.

      Quote:
      Daneel, me parece que no me están «vendiendo» nada, los datos están y muestran períodos más cálidos como por ejemplo el Período Cálido Medieval.

      Siguiendo con las fuentes, ya tengo mi opinión formada, y al igual que wuwt, skeptical science también tiene sus problemas.

      Respecto de los datos del NOAA, le di un vistazo y la única reconstrucción global de más de mil años es la de Mann.

      De hecho, sí te están vendiendo mierda. Lo hacen con todo el mundo.

      PHO8k.jpg

      El «consenso» no es sólo el IPCC, éste es sólo un órgano que se encarga de ralizar el equivalente a una revisión sistemática de la literatura. El consenso tiene que ver con que la vasta, enorme mayoría de los expertos en un tema llegaron a una conclusión sólida sobre un tema.

      Y mirá si no van a estar vendiendo porquerías. El link que mandaste sobre el MWP nunca lo había visto pero es un clásico. En vez de hacer un gráfico unificado y con un buen análisis estadístico para poder analizar la tendencia por sobre el ruido, te tiran un montón de gráficos incomparables entre sí que ni siqueira son de temperaturas y luego afirman «el MWP fue global».

      Agarro un link al azar: Liu et. atl. una reconstrucción climática en China y veo el siguiente gráfico:

      R7lKE.png

      Oh, impresionante. No ponen un link al pdf así que voy a la fuente. Ups. Está detrás de una pared de pago, menos mal que yo tengo acceso institucional y puedo leerlo… me pregunto cuántos otros podrán. Busco ese gráfico… no lo cuentro. Ah, pero dice que esa imagen está «adaptada» de ese paper. Entonces busquemos qué otros gráficos hay…

      6wWpU.gif

      Oh, interesante. La primera serie de datos es una reconstrucción de las temperaturas de China según un tal Yang et. at. Es interesante que en el gráfico que encontré en la página que mandaste no se ve ese pequeño salto en la temperatura.

      Ah, porque usaron la temperatura del lago Qinghai, no la de China.

      Na, esto es demasiado trabajo. ¿Por qué no plotear en un gráfico varias reconstrucciones de la temperatura media del hemisferio norte junto con el regristro instrumental y listo? Es mucho más claro y representativo.

      HljFk.jpg

      (fuente: Climate Change 2007: Working Group I: The Physical Science Basis 6.6.1.1 What Do Reconstructions Based on Palaeoclimatic Proxies Show?)

      Ah, mucho mejor. Más claro y mucho más útil. Y no es de extrañar, ya que las temperaturas locale son MUY varibables y heterogéneas:

      ujQNq.png

      (fuente: idem)

      Insisto. Te tratan de vender ruido por datos, confusión por conocimiento y negacionismo por escepticismo.

    • #30639
      smaga
      Miembro

      saibaba, creo que peer-review es un cuestionamiento bastante fuerte, y no hacen falta unos «pocos» que cuestionen a la mayoría, solo uno bien respaldado basta (A. Einstein). Y no creo que cortar la mitad del suministro de energía de nuestro país va generar solo «otra crisis económica».

      Me gustaría ver las fuentes sobre «… si nos vamos a morir todos de cáncer y vamos a reventar como sapos cuando se descalabre todo por el calentamiento global?…». Me llama un poco la atención el silencio de Daneel a tus comentarios.

    • #30640
      ryomashi
      Miembro

      Yo había escuchado/leído por ahí que las consecuencias para los países desarrollados serían mínimas si adoptaran energías renovables. Pero es eso «leí por ahí» quería saber si alguien más sabía algo.

      Para los países en desarrollo una política de protección ambiental los haría mierda. Lo que mas necesitan son inversiones.

      No se cual es la situación latinoamericana al respecto: no estamos tan bien como europa o estados unidos pero no estamos tan mal como África.

    • #30641
      saibaba
      Miembro

      ¿Y dónde está el peer-review de los que vos decís smaga?

      ¿Y por qué 97% de expertos no lo aceptan?

      Yo creo que si ese 97% no acepta una publicación, es que el consenso es aún más fuerte.

      Ese 97% es revisión por pares.

      No es sólo la revisión de un referee lo que cuenta en este caso, sino que todos los demás científicos han rebatido esos resultados.

      No es un consenso democrático.

      Las pruebas dicen que el calentamiento global es culpa del CO2 humano.ri

      ¿Para qué me pedís pruebas, si a vos las pruebas no te convencen?

      Sólo te convence cuando oís lo que te gusta oír.

    • #30642
      saibaba
      Miembro

      No entender la gravedad del calentamiento global es vivir en una nube de pedos.

    • #30643

      ryomashi dijo:

      Yo había escuchado/leído por ahí que las consecuencias para los países desarrollados serían mínimas si adoptaran energías renovables. Pero es eso «leí por ahí» quería saber si alguien más sabía algo.

      Mirá, según un análisis de varios modelos económicos:

      – En promedio, el impacto en el PBI estadounidense sería de menos de un 1% y el crecimiento de su economía se vería afectado en un 0,03%.

      – El desempleo aumentaría un 0,05% acumulado en las próximas 2 décadas. Una figura que sólo considera los sectores existentes actualmente y no tiene en cuenta la creación nuevos rubros de en economía como producto de las nuevas tecnologías.

      – En promedio, el costo por familia se vería reflejado en un aumento del presupuesto familiar menor al 1%. El aumento sería de entre U$S 84 y U$S 160 por año. Para las familias de menores ingresos, el costo sería mucho menor incluso pudiendo tener una disminución de unos U$S 125 según un análisis.

      – El costo de la electricidad, sorprendentemente, podría reducir. Si bien el costo por watt/hora aumentaría la reducción del consumo y los programas de eficiencia energéticas tendrían a contrarrestar el efecto.

      Es importante aclarar que esas comparaciones no toman en cuenta el COSTO de no hacer nada. Es decir, eso sería lo esperable si tratamos de mitigar el cambio climático pero resulta que era un fraude.

      Fijate en este post mío que tiene el link y todo eso. También hace un tiempo se hizo una encuesta entre economistas expertos en cambio climático que publican en revistas con revisión por pares y básicamente encontraron que todos dicen que los costos de no mitigar el cambio climático son mucho mayores que los beneficios. Otra cosa interesante es que la mayoría considera que las incertidumbres en los modelos climáticos son un argumento para mayor acción.

      Igual, soy mucho más ignorante del tema económico que el atmosférico así que no puedo dar demasiado insight ahí.

      Quote:
      Para los países en desarrollo una política de protección ambiental los haría mierda. Lo que mas necesitan son inversiones.

      No sé. Sabiendo que el futuro está en la producción de energía limpia y renovable y que se trata de un mercado con mucho crecimiento en donde la innovación y la inventiva cuentan más que la mera inversión de capital, a mí me parecería muy razonable entrar en ese juego lo antes posible. De todas formas ningún tratado razonable va a pedirle medidas ambientales estrictas a Uganda, Haití o el Congo así que el punto no viene al caso.

      De hecho, los países que más van a ser afectados por el cambio climático son los subdesarrollados. No sólo es porque no tienen el capital para realizar las obras de infraestructura y servicio social que la adaptación va a requerir, sino porque muchos de estos países están en los trópicos. En esa zona la variabilidad interanual es mucho menor que en altas latitudes por lo que aún menores aumentos en la temperatura pueden causar mucho más estrés.

      Acá se ve en gráfico por si no se entendió lo que quise decir.

      wRZpC.png

      Es decir que, irónicamente, los que más van a sufrir el cambio climático son los países que menos emiten. De hecho, si se plotea la relación señal/ruido vs. CO2 per cápita, la ironía se vuelve ironía gráfica:

      QpfAa.png

      (fuente de ambos gráficos: [url=Early onset of significant local warming in low latitude countries

      ]Early onset of significant local warming in low latitude countries[/url])

      Estamos en una «tormenta perfecta» en la que los países que más emiten son los que menos razones tienen para dejar de emitir.

      smaga dijo:

      Me llama un poco la atención el silencio de Daneel a tus comentarios.

      El tiempo es finito y el post de saibaba lo leí por arriba. Igual creo poder reconocer un hipérbole cuando lo veo.

      OjyPW.jpg

    • #30644
      smaga
      Miembro

      Daneel Olivaw dijo:

      Sin hacer (ni poder hacer) una crítica del paper o el contexto de la literatura científica, cito del abstract:

      «The reversing trend may relate to a possible recovery of stratospheric ozone concentration»

      El CO2 no es la única variable en juego, por lo que hay que tener en cuenta el peso de TODA la evidencia y ver qué

      Si la tendencia se revirtió posiblemente por la recuperación del ozono, el enfriamiento se produjo entonces mayormente por la pérdida de este último, por lo tanto, no habría un claro indicador del CGA.

      Igualmente, mi respuesta fué a la afirmación sobre el efriamiento de la estratósfera como «huella innegable del calentamiento por efecto invernadero». Bueno, la estratósfera, desde el 96 dejó de enfriarse. Entoces ahora pasemos a debatir sobre la altura de la tropopausa:

      Daneel Olivaw dijo:

      Está aumentando y es imposible explicarlo basándose en factores naturales.

      Está aumentando, pero no sería un claro indicador del cambio climatico:

      (1) del abstract «…Therefore, as an indicator of climate

      change, long-term changes in the tropopause may carry less information about changes

      throughout the vertical temperature profile than has been suggested by previous studies

      using reanalyses and global climate models…»

      (2) de la introducción «…suggesting that the tropopause pressure (or height) may only carry limited information about global warming in the troposphere.

      Daneel Olivaw dijo:

      Irrelevante a la cuestión. El CCA existe y al CO2 calienta igual sin importarle si los países subdesarrollados lo necesitan o no.

      No es irrelevante ya que él centra el las bases escépticas del CGA en el petróleo, a eso va mi respuesta.

      Daneel Olivaw dijo:

      Y mirá si no van a estar vendiendo porquerías. El link que mandaste sobre el MWP nunca lo había visto pero es un clásico. En vez de hacer un gráfico unificado y con un buen análisis estadístico para poder analizar la tendencia por sobre el ruido, te tiran un montón de gráficos incomparables entre sí que ni siqueira son de temperaturas y luego afirman «el MWP fue global».

      Los datos están, que no haya un buen análisis es otra cosa.

      Daneel Olivaw dijo:

      Agarro un link al azar:

      Me parece que no fué al azar…

      Daneel Olivaw dijo:

      Oh, interesante. La primera serie de datos es una reconstrucción de las temperaturas de China según un tal Yang et. at. Es interesante que en el gráfico que encontré en la página que mandaste no se ve ese pequeño salto en la temperatura.

      Ah, porque usaron la temperatura del lago Qinghai, no la de China.

      Ok, yo no tengo acceso académico,no hubo forma de que hubiera podido ver eso, pero, ¿qué pasará si agarramos los otros cuatro sobre China?. En el que está arriba de ese, se muestra «ese pequeño salto». Y este es otro para tener en cuenta

      Daneel Olivaw dijo:

      Na, esto es demasiado trabajo. ¿Por qué no plotear en un gráfico varias reconstrucciones de la temperatura media del hemisferio norte junto con el regristro instrumental y listo? Es mucho más claro y representativo.

      Este es un análisis análogo al tuyo (salvando las diferencias) sobre el clásico palo de hockey.

      Daneel Olivaw dijo:

      Ah, mucho mejor. Más claro y mucho más útil. Y no es de extrañar, ya que las temperaturas locale son MUY varibables y heterogéneas:

      Sin embargo, ambos gráfcos tienen la misma tendencia.

      Daneel Olivaw dijo:

      Te tratan de vender ruido por datos, confusión por conocimiento y negacionismo por escepticismo.

      Insisto, no tratan de vender nada, los datos están

    • #30645
      smaga
      Miembro

      saibaba dijo:

      ¿Y dónde está el peer-review de los que vos decís smaga?

      Seguí los link del debate.

      saibaba dijo:

      ¿Y por qué 97% de expertos no lo aceptan?

      No se, habría que preguntarles.

      saibaba dijo:

      ¿Para qué me pedís pruebas, si a vos las pruebas no te convencen?

      Sólo te convence cuando oís lo que te gusta oír.

      No veo ninguna fuente.

    • #30646
      smaga
      Miembro

      saibaba dijo:

      No entender la gravedad del calentamiento global es vivir en una nube de pedos.

      Denunciá al Servicio Metereológico Nacional por malversación de fondos.

    • #30647

      Sobre la tropopausa y la estratósfera, como dije son parte de la evidencia, no «LA» evidencia. Así como con la evolución, existe una convergencia que nos indica una sola respuesta. ¿Puede que el enfriamiento previo se debiera a la pérdida del Ozono? Sí pero, ¿en qué medida? ¿Cómo podemos saberlo? La verdad que no estoy seguro. Probablemente analizando la distribución latitudinal de las tendencias. Sería algo interesante para preguntarle a un climatólogo.

      ebbCB.png

      cYBOy.png

      (fuente: An update of observed stratospheric temperature trends [gratis :P])

      En cualquier caso, si la teoría el cambio climático antropogénico se refuta notando que desde 1993 se revierte la tendencia, ¿la teoría del cambio climático heliogénico no debería refutarse por la tendencia en bajada aún en latitudes sin pérdida de ozono? Sin embargo Watts y sus secuaces la siguen promoviendo cada vez que pueden.

      Y en cuanto al aumento de la altitud de la tropopausa (que está relacionada con el enfriamiento de la estratósfera) de nuevo repito que resulta imposible explicarlo mediante procesos naturales y es muy bien explicado mediante el forzamiento antropogénico.

      iNYXR.png

      Se puede hilar maś fino sobre los resultados, seguro. Pero, de nuevo, la dirección de la evidencia es clara. Y, del abstract que pasaste:

      «These tropopause trends are accompanied by significant stratospheric cooling and smaller tropospheric warming. This association of tropopause height and stratospheric temperature trends, together with the presence of a significant QBO signal in tropopause height, suggests that, at these lowest frequencies, the tropopause is primarily coupled with stratospheric temperatures. Therefore, as an indicator of climate change, long-term changes in the tropopause may carry less information about changes throughout the vertical temperature profile than has been suggested by previous studies using reanalyses and global climate models»

      Lo que yo entiendo de esto es, no sólo que la estratósfera se está enfriando (contradiciendo tu punto anterior), sino que la altura de la estratósfera tendría limitada información sobre el perfil vertical de la temperatura de lo que se pensaba, a muy bajas frecuencias. QBO = oscilación quasi-bienal. Es decir, dos años.

      Sí, el link fue al azar. Agarré cualquiera.

      Y el link que mostrás es otro intento más de vender pescado podrido. ¿No te resulta curioso que en la leyenda no dice qué es la linea negra? Esos son los registros con termómetros mientras que el resto son proxis. Nuevamente tratando de perpetuar el mito del «hide the decline». Lo único que hicieron fue sacar del gráfico la parte de la reconstrucción de las temperaturas que SABÍAN que tenía un problema (que es tan conocido que hasta tiene un nombre: el problema de la divergencia, en la cual algunos árboles de altas latitudes muestran una desviación desde los ’60).

      Por otro lado, esa no es la temperatura de China, que es la relevante al punto que estaba haciendo de que las temperaturas actuales de la zona son superiores a la del MWP. No hace falta que tengas acceso académico, el gráfico de las temperaturas medias en China es la primer linea de este gráfico (sacado del paper que referencian en la página que pasaste)

      6wWpU.gif

      Mi punto es que los autores de ese sitio agarraron la temperatura de un lago de china que mostraba, quizás, mayor temperatura cerca del MWP en vez de agarrar la temperatura de todo China que muestra CLARAMENTE que la temperatura es mucho menor. Y, para peor, las reconstrucciones globales tomadas en su conjunto muestran, nuevamente, que la temperatura actual es mucho mayor que durante el MWP.

      HljFk.jpg

      Igual te lo advierto a vos así como se lo advertí a DrGEN, al entrar en territorio conspiracional estás entrando también en el territorio de lo infalseable. En una situación en la que cualquier estudio o evidencia puede ser desestimado con una sola palabra: climategate.

      smaga dijo:

      Denunciá al Servicio Metereológico Nacional por malversación de fondos.

      Pues vos deberías denunciar a las academias nacionales de Autralia, Bélgica, Brasil, CAmerún, Canada, el Caribe, China, Francia, Ghana, Alemania, Indonesia, Irlanda, Italia, India, Japón, Kenia, Madagascar, Malasia, México, Nigeria, Nueva Zelandia, Rusia, Senegal, Sudáfrica, Sudán, Suecia, Tanzania, Turquía, Uganda, Reino Unido, Estados Unidos, Zambia, Zimbawe. También a la Asociación Americana para el avance de la Ciencia, NOAA, Unión Geofísica Estadounidense, Federacion europera de geólogos, etc

    • #30648
      saibaba
      Miembro

      Bueno, no me parece que smaga tenga que pedirme explicaciones a mí sobre este asunto.

      Yo sólo veo todo esto desde afuera, y entonces lo más que puedo hacer es defender o no la opinión de los científicos, y no los detalles técnicos.

      Lo del «peer-review» es un contrasentido.

      Smaga dice que la posición de que el CO2 no se produce por acción humana está avalado por el «peer-review».

      Pero «peer-review» es «revisión por pares», hasta donde yo sé de «inglés».

      Si el 97% de los expertos en el tema dice que la verdad es la «opuesta», entonces eso es «peer-review», porque los «expertos» son los «pares», los colegas de profesión de ese 3% que tiene la posición contraria.

      Ellos han rebatido los argumentos de esta minoría, y entonces es dicha minoría la que debe aportar pruebas realmente sólidas de lo que están afirmando, y no lo han logrado.

      Cuando Einstein lanzó su teoría, era una minoría, y tenía razón, pero debido a lo revolucionario de su posición, tuvo que pasar una alta resistencia de los «pares» que logró convencer a la comunidad científica.

      Los negacionistas, por lo que se ve, no han convencido a nadie, y sus argumentos han sido rebatidos una y otra vez.


      Es natural que el consenso no sea del 99.99999%, justamente porque el clima global es algo más complejo de modelar o entender que la física de Einstein. Y además hay intereses económicos y políticos, así que también es más probable que haya científicos que «tuerzan» sus opiniones, sólo si les pagan.

      No sé si smaga llama «peer-review» a otra cosa.

      A lo mejor se refiera a lo siguiente:

      * Un señor NN publica un paper en una revista XX.

      * Otro señor MM revisa el paper en calidad de árbitro, y le da el OK.

      * La revista XX dice, bueno: ya está, y lo publican.

      Se supone que eso alcanza para «peer-review» en muchos trabajos científicos de importancia «intermedia».

      Pero para un tema importante como el cambio climático, en donde hay intereses creados, eso no alcanza, porque cualquiera puede inventar una revista, invitar a un amigo a publicar el paper que le interesa, invita a otro amigo a que haga un referato trucho, y ya está. Ese es tu «peer-review». Una truchada total.

      En tal caso, el verdadero «peer-review» se logra con el consenso de todos los «pares».

      Pero al parecer eso no es posible, y por lo que he visto, «sólo» hay un 97%.

      En este caso me parece más que suficiente, porque no es una votación política, sino que es opinión experta, basado en hechos y lógica.

      Y es más sencillo entender al restante 3% como gente equivocada, o comprada, etc.

      Como esa gente no puede convencer a los demás científicos, se han dedicado a hacer campaña de desinformación.

      Son ellos quienes tienen que dar argumentos sólidos.


      Bueno, acá ha surgido, según lo que he dicho, una discusión acerca de cuánto porcentaje de consenso de expertos, y de cuáles expertos, «alcanza» para tener una opinión científica oficialmente aceptada por la comunidad científica.

      Está claro que un 50% indica una posición dividida.

      Un 100% es fantástico.

      ¿Y un 97%?

      En cualquier caso, me parece más fácil preguntarle al 3% restante por qué piensan lo contrario del restante 97%, y discutir con ellos, que ir a preguntarles uno por uno a los del 97%.

      Y jodo con el 97% porque es el número que ví por acá en los posts y los videos.

      Pero igual estaría bueno que estas cosas estén todas publicadas en el dominio público.

      Lo que dice smaga de que no hay acceso académico a ciertas cosas, es una de las cosas que critico a la ciencia.

      Porque si cualquiera se quiere largar a hacer un «razonamiento» y entender de qué se trata todo, no le queda más remedio que «pagar».

      En la flecha de «implicación» hay que poner un signo $ arriba, si no, el Modus Ponens no funca.

      Es ridículo.

      Bueno, no quiero hablar más de esto.

      Yo ya entendí o me convencí de que el CO2 se produce por acción humana.

      Saludos

    • #30649

      Tenés razón en que la publicación de un artículo en una revista no es el final del peer review sino recién el principio. Y también en que para el lego (que incluye a otros científicos en otras disciplinas) lo que más importa es, en definitiva, el consenso científico de expertos.

      Ahora, eso no va a convencer a ningún lego que ya esté convencido de que el CC es un fraude justamente por eso. Si uno cree una posición opuesta a la vasta mayoría de la comunidad científica sólo quedan dos opciones: o son todos unos idiotas, o están conspirados para falsear o esconder datos. En cualquier caso, si uno está aferrado a una conclusión en vez de al método, entonces tiene piedra libre para desestimar cualquier cosa.

    • #30650
      smaga
      Miembro

      Daneel Olivaw dijo:

      Sobre la tropopausa y la estratósfera, como dije son parte de la evidencia, no «LA» evidencia.

      ¿Puede que el enfriamiento previo se debiera a la pérdida del Ozono? Sí pero, ¿en qué medida? ¿Cómo podemos saberlo? La verdad que no estoy seguro.

      Daneel Olivaw dijo:

      También se observa que mientas la tropósfera (baja atmósfera) se está calentando, la estratósfera (alta atmósfera) ¡se está enfríando! Esto es una huella innegable que indica un calentamiento por efecto invernadero

      Si no entiendo mal, «huella innegable» y «LA evidencia» son conceptos muy ligados. Y si no estás seguro, creo que no sería, entonces, innegable.

      Daneel Olivaw dijo:

      Probablemente analizando la distribución latitudinal de las tendencias. Sería algo interesante para preguntarle a un climatólogo.

      La verdad que sí. Igualmente, insisto, la altura de la tropopausa accarrea poca información sobre sobre este tema.

      Daneel Olivaw dijo:

      En cualquier caso, si la teoría el cambio climático antropogénico se refuta notando que desde 1993 se revierte la tendencia, ¿la teoría del cambio climático heliogénico no debería refutarse por la tendencia en bajada aún en latitudes sin pérdida de ozono?.

      Lo que se refuta con ese estudio es que la estratósfera en general no se está enfriando desde el 96. Desconozco la respuesta a tu pregunta.

      Daneel Olivaw dijo:

      Y en cuanto al aumento de la altitud de la tropopausa (que está relacionada con el enfriamiento de la estratósfera) de nuevo repito que resulta imposible explicarlo mediante procesos naturales y es muy bien explicado mediante el forzamiento antropogénico.

      Dos estudios independientes concuerdan que la altura de la tropopausa contiene limitada información sobre el calentamiento global de la tropósfera.

      En base a esos estudios, los modelos que presentás se deberían actualizar, así que todavía no estaría «muy bien explicado».

      Daneel Olivaw dijo:

      Se puede hilar mas fino sobre los resultados, seguro. Pero, de nuevo, la dirección de la evidencia es clara.

      Creo que no es clara

      Daneel Olivaw dijo:

      Y, del abstract que pasaste:

      «These tropopause trends are accompanied by significant stratospheric cooling and smaller tropospheric warming. This association of tropopause height and stratospheric temperature trends, together with the presence of a significant QBO signal in tropopause height, suggests that, at these lowest frequencies, the tropopause is primarily coupled with stratospheric temperatures. Therefore, as an indicator of climate change, long-term changes in the tropopause may carry less information about changes throughout the vertical temperature profile than has been suggested by previous studies using reanalyses and global climate models»

      Yo pondría en negrita también la última oración.

      Daneel Olivaw dijo:

      Lo que yo entiendo de esto es, no sólo que la estratósfera se está enfriando (contradiciendo tu punto anterior).

      Me parece que no lo contradice. El primer estudio concluye que desde el 96 se detuvo (y hasta se revirtió) la tendencia a enfriarse 8sin olvidarnos que el CO2 siguió aumentando). Al que vos hacés referencia, toma en cuenta la serie de datos desde el 80 al 04 y como no se llegó a los valores donde comenzó esta serie, concluyen que se está enfriando.

      Daneel Olivaw dijo:

      Y el link que mostrás es otro intento más de vender pescado podrido. ¿No te resulta curioso que en la leyenda no dice qué es la linea negra?

      No, porque siempre se supo.

      Daneel Olivaw dijo:

      Esos son los registros con termómetros mientras que el resto son proxis. Nuevamente tratando de perpetuar el mito del «hide the decline». Lo único que hicieron fue sacar del gráfico la parte de la reconstrucción de las temperaturas que SABÍAN que tenía un problema (que es tan conocido que hasta tiene un nombre: el problema de la divergencia, en la cual algunos árboles de altas latitudes muestran una desviación desde los ’60).

      Yo no lo llamaría «es tan conocido», para mí sería «un poco escondido»

      Daneel Olivaw dijo:

      Por otro lado, esa no es la temperatura de China, que es la relevante al punto que estaba haciendo de que las temperaturas actuales de la zona son superiores a la del MWP. No hace falta que tengas acceso académico, el gráfico de las temperaturas medias en China es la primer linea de este gráfico (sacado del paper que referencian en la página que pasaste)

      Para el link de Yang necesito una contraseña. Por suerte, aparece en otros de tus links

      Lo que me llama la atención es que en el que marca un mayor calentamiento actual, el número de series usados en años más recientes se redujo a la mitad.

      Daneel Olivaw dijo:

      Igual te lo advierto a vos así como se lo advertí a DrGEN, al entrar en territorio conspiracional estás entrando también en el territorio de lo infalseable. En una situación en la que cualquier estudio o evidencia puede ser desestimado con una sola palabra: climategate.

      Ni lo nombré al climategate. Te hice referencia a Mcintyre por el trabajo en sí sobre el proxy problemático. Lo que él opine es problema de él.

      Daneel Olivaw dijo:

      Pues vos deberías denunciar a las academias nacionales de Autralia, Bélgica, Brasil, CAmerún, Canada, el Caribe, China, Francia, Ghana, Alemania, Indonesia, Irlanda, Italia, India, Japón, Kenia, Madagascar, Malasia, México, Nigeria, Nueva Zelandia, Rusia, Senegal, Sudáfrica, Sudán, Suecia, Tanzania, Turquía, Uganda, Reino Unido, Estados Unidos, Zambia, Zimbawe. También a la Asociación Americana para el avance de la Ciencia, NOAA, Unión Geofísica Estadounidense, Federacion europera de geólogos,

      No, no lo necesito porque no acusé a nadie de «vivir en una nube de pedos», o «hazmerreir» o «vender pescado podrido». Son hipótesis diferentes, nada más.

    • #30651

      smaga dijo:

      Si no entiendo mal, «huella innegable» y «LA evidencia» son conceptos muy ligados. Y si no estás seguro, creo que no sería, entonces, innegable.

      No vamos a discutir cuestiones semánticas. Creo que ambos sabemos cómo funciona la ciencia y que ninguna evidencia por sí sola sirve para afirmar o refutar una teoría. Es la convergencia de las evidencias lo que importa. En este caso, la altura de la tropopausa (que sólo puede ser explicado mediante causas antropogénicas) es una de ellas.

      Quote:
      Dos estudios independientes concuerdan que la altura de la tropopausa contiene limitada información sobre el calentamiento global de la tropósfera.

      En base a esos estudios, los modelos que presentás se deberían actualizar, así que todavía no estaría «muy bien explicado».

      ¿Qué es lo que dice ese estudio? «Como un indicador de cambio climático, cambios a largo plazo en la tropopause proveerían menos información (…) de lo que se había sugerido»

      ¿Qué es lo que no dice? «La altura de la tropopausa no tiene ninguna información sobre la atribución del calentamiento global».

      ¿Qué es lo que dice en el IPCC?:

      «Model simulations with individual forcings indicate that the major drivers of the model tropopause height increases are ozone-induced stratospheric cooling and the tropospheric warming caused by greenhouse gas increases (Santer et al., 2003a). However, earlier model studies have found that it is difficult to alter tropopause height through stratospheric ozone changes alone (Thuburn and Craig, 2000)»

      http://www.ipcc.ch/publications_and_data/ar4/wg1/en/ch9s9-4-4-2.html

      Como dije, hay muchos factores que afectan al clima. Con el conocimiento que tenemos podemos decir que se ve la señal del CO2 en la altura de la tropopausa y el enfriamiento de la estratósfera. Si tu hipótesis es que NO es el CO2 antropogénico entonces es TU responsabilidad proponer un mecanismo que explique los datos.

      El CO2 antropogénico lo explica muy bien y no se puede encontrar una explicación natural:

      iNYXR.png

      Además seguís hablando del 96 pero eso no es más que un caso de Cherry Picking. Es lo mismo que cuando dicen «no hay calentamiento desde 1998». ¿Por qué el 96 sería importante? ¿Por qué analizarías la tendencia desde el 96 expecíficamente? Estoy seguro que si uno lo intenta lo suficiente va a poder encontrar un montón de períodos en los cuales la temperatura superficial disminuyó o se mantuvo constante, o que la estratosférica aumentó… ¡incluso podés decir que desde 2007 el hielo del Ártico se mantuvo constante! (Claro, porque el abismal mínimo de 2007 fue básicamene empatado en 2011).

      La cuestión es que habrá que ver por qué la tropopausa no siguió aumentando. Uno de los papers que linkeaste dice:

      Quote:
      Using such CCM integrations, it is found that the tropopause pressure (height) will continue to decrease (increase) in the future, but with a trend weaker than that in the recent past. The reduction in the future tropopause trend is shown to be directly associated with stratospheric ozone recovery. A significant ozone recovery occurs in the Southern Hemisphere lower stratosphere of the CCMs, and this leads to a relative warming there that reduces the tropopause trend in the twenty-first century.

      The future tropopause trends predicted by the CCMs are considerably smaller than those predicted by the Intergovernmental Panel on Climate Change Fourth Assessment Report (AR4) models, especially in the southern high latitude.

      Así que, aparentemente, hay un mecanismo plausible (recuperación de la capa de ozono) para explicar los datos y que no invalida su utilidad en los estudios de atribución (aunque ésta sea mayor o menor).

      Y en la parte de conclusiones dice:

      Quote:
      The globally averaged tropopause pressure hPTPi has been suggested as a sensitive indicator of anthropogenic climate change. Because of the combined effect of tropospheric warming by greenhouse gases and stratospheric cooling by ozone depletion, the global tropopause pressure (height) has been decreasing (increasing) over last two decades (Randel et al. 2000; Seidel et al. 2001; Sausen and Santer 2003; Santer et al. 2003a,b, 2004; Seidel and Randel 2006)

      Y esto es lo que dice el otro paper que linkeaste:

      Quote:
      Temperature changes in the lower stratosphere show cooling of $0.5 K/decade over much of the globe for 1979–2007, with some differences in detail among the different radiosonde and satellite data sets. Substantially larger cooling trends are observed in the Antarctic lower stratosphere during spring and summer, in association with development of the Antarctic ozone hole.

      An update of observed stratospheric temperature trends (PDF)

      Y las tendencias (de ese mismo paper)

      JaXLE.png

      Quote:
      Yo no lo llamaría «es tan conocido», para mí sería «un poco escondido»

      ¿Me estás jodiendo? Leete la sección del IPCC sobre el MWP y las reconstrucciones. Bien a la vista dice:

      Quote:
      All of the large-scale temperature reconstructions discussed in this section, with the exception of the borehole and glacier interpretations, include tree ring data among their predictors so it is pertinent to note several issues associated with them. (…)

      This ‘divergence’ is apparently restricted to some northern, high-latitude regions, but it is certainly not ubiquitous even there. In their large-scale reconstructions based on tree ring density data, Briffa et al. (2001) specifically excluded the post-1960 data in their calibration against instrumental records,

      El problema de la divergencia de los datos es conocido y debatido en la comunidad científica. Está ahí para que todos los vean.

      Si querés acusar a alguien de «esconder» algo, podrías empezar con el que compiló esa página con gráficos de temperaturas para probar que el MWP fue más cálido que ahora. Yo diría que «escondió el incremento».

      Esta es una imagen que hice con el gráfico que presenta esa página (un cierto lago de China) con las temperaturas de «toda» China que saqué del link que pasaste del NOAA.

      dbsCI.png

      NO es algo científico ya que no está a escala y, en realidad, son cosas completamente distintas (uno es una reconstrucción de todo un país y la otra es una reconstrucción de un isótopo y su relación con la temperatura). El punto que quiero hacer, es que esa página trata de vender ruido por datos. Te tira un montón de gráficos en la cara pero no los pone en su contexto y, peor aún, OMITE gráficos importantes. Para demostrar que el MWP fue más caliente que la época actual no haría falta tirar 30 gráficos locales, son sólo un gráfico con la reconstrucción de la temperatura media global (hemisférica en este caso) bastaba. Uno como este:

      HljFk.jpg

      http://www.ipcc.ch/publications_and_data/ar4/wg1/en/ch6s6-6.html

    • #30652

      Hablando de el perfil vertical del cambio en la temperatura. Me encontré esta página con datos satelitales. Me llamó la atención esta serie de gráficos que muestran la anomalía de la temperatura y la tendencia. Cada gráfico representa aproximadamente una capa más alta de la atmósfera:

      1v7Ug.png

      hTmyf.png

      nnLG6.png

      SwbWr.png

    • #30653
      smaga
      Miembro

      Daneel Olivaw dijo:

      No vamos a discutir cuestiones semánticas. Creo que ambos sabemos cómo funciona la ciencia y que ninguna evidencia por sí sola sirve para afirmar o refutar una teoría. Es la convergencia de las evidencias lo que importa. En este caso, la altura de la tropopausa (que sólo puede ser explicado mediante causas antropogénicas) es una de ellas.

      Insisto, la información (según estudios recientes) que aporta la altura de la tropopausa sobre el cambio climático es limitada.

      Daneel Olivaw dijo:

      ¿Qué es lo que dice ese estudio? «Como un indicador de cambio climático, cambios a largo plazo en la tropopause proveerían menos información (…) de lo que se había sugerido»

      O sea que los modelos subestiman esta variable.

      Daneel Olivaw dijo:

      ¿Qué es lo que no dice? «La altura de la tropopausa no tiene ninguna información sobre la atribución del calentamiento global».

      Claro que no, ese no es el fin del estudio.

      Daneel Olivaw dijo:

      ¿Qué es lo que dice en el IPCC?:

      «Model simulations with individual forcings indicate that the major drivers of the model tropopause height increases are ozone-induced stratospheric cooling and the tropospheric warming caused by greenhouse gas increases (Santer et al., 2003a). However, earlier model studies have found that it is difficult to alter tropopause height through stratospheric ozone changes alone (Thuburn and Craig, 2000)»

      Según estudios del 2000 y del 2003. Ya hay trabajos más recientes como los que linkeé.

      Daneel Olivaw dijo:

      Como dije, hay muchos factores que afectan al clima. Con el conocimiento que tenemos podemos decir que se ve la señal del CO2 en la altura de la tropopausa y el enfriamiento de la estratósfera. Si tu hipótesis es que NO es el CO2 antropogénico entonces es TU responsabilidad proponer un mecanismo que explique los datos.

      No tengo ni idea del mecanismo para explicar los datos. Lo que SÍ se puedo proponer (como ya lo hice) son los datos mismos:

      – a pesar que el CO2 sigue en aumento (y su efecto invernadero como fuente principal de calentamiento), los satélites muestran que los últimos 15 años (de 33) no hubo un efriamiento de la estratósfera.

      – ya argumenté sobre la altura de la tropopausa.

      Daneel Olivaw dijo:

      El CO2 antropogénico lo explica muy bien y no se puede encontrar una explicación natural:

      Con los modelos, pareciera que no.

      Daneel Olivaw dijo:

      Además seguís hablando del 96 pero eso no es más que un caso de Cherry Picking.

      Prácticamente la mitad de los datos medidos por satélites muestran la detención (y hasta la inversión) de la tendencia. Se vé claramente en el último gráfico que linkeás.

      Daneel Olivaw dijo:

      «no hay calentamiento desde 1998». ¿Por qué el 96 sería importante?

      Porque las emisiones de CO2 siguen en aumento.

      Daneel Olivaw dijo:

      ¿Me estás jodiendo? Leete la sección del IPCC sobre el MWP y las reconstrucciones. Bien a la vista dice…

      El problema de la divergencia de los datos es conocido y debatido en la comunidad científica. Está ahí para que todos los vean.

      Menos en el gráfico.

      Daneel Olivaw dijo:

      Si querés acusar a alguien de «esconder» algo, podrías empezar con el que compiló esa página con gráficos de temperaturas para probar que el MWP fue más cálido que ahora. Yo diría que «escondió el incremento».

      Podría ser, me gustría leer el estudio completo para ver qué es lo que dice (sobre todo respecto de la reducción a la mitad de las series usadas en años recientes.).

      No te olvides de lo que muestran los otros gráficos.

      Daneel Olivaw dijo:

      Esta es una imagen que hice con el gráfico que presenta esa página (un cierto lago de China) con las temperaturas de «toda» China que saqué del link que pasaste del NOAA.

      Me parece que lo pegaste al revés.

      Daneel Olivaw dijo:

      El punto que quiero hacer, es que esa página trata de vender ruido por datos. Te tira un montón de gráficos en la cara pero no los pone en su contexto y, peor aún, OMITE gráficos importantes. Para demostrar que el MWP fue más caliente que la época actual no haría falta tirar 30 gráficos locales, son sólo un gráfico con la reconstrucción de la temperatura media global (hemisférica en este caso) bastaba.

      Ya lo dije, los datos están, que no haya un buen análisis es otra cosa.

    • #30654

      Sí hay un buen análisis y lo linkeé varias veces. También linkeé el gráfico que muestra cómo los modelos pueden explicar la altura de la tropopausa sólo si se tienen en cuenta los forzamientos humanos. Que el nivel de información sea menor o mayor del que se pensaba antes no invalida la cuestión básica. Y, de nuevo, el CO2 NO es el único factor importante en el clima y dado que los mismos papers que linkeaste vos presentan una explicación posible para los datos que NO implican la refutación del cambio climático antropogénico (la recuperación de la capa de ozono) y que no presentás ningún mecanismo alternativo para presentar la tendencia negativa general sin hacer relación al CO2, sigo sin ver dónde está la relación con él.

      Más aún. Si tu argumento es que el CO2 NO está teniendo el efecto de enfriar la estratósfera y calentar la tropósfera, eso está en conflicto con la ciencia básica. Al aumentar el CO2, sólo basándose en física y química, esperaríamos que la temperatura aumentara y de esa misma manera. Quien sostenga que eso NO está pasando deberá, como debe hacerse en ciencia, presentar un mecanismo que:

      1. Explique la tendencia general consistente con esa expectativa.

      2. Explique por qué el CO2 NO lo afecta.

      Puede ser que haya pegado el gráfico al revés, ¿cambia en algo mi punto? No. No importa cómo los compares, el gráfico que te venden en esa página omite el registro instrumental y, en efecto, esconde el aumento.

      Esta es la versión dada vuelta:

      ZRccp.jpg

    • #30655
      saibaba
      Miembro

      ¿Qué piensan del pronóstico catastrófico que aparece en Clarin?

      Habla del fin absoluto de la humanidad en este siglo.

      http://www.clarin.com/mundo/grados-abismo_0_569343154.html

    • #30656

      saibaba dijo:

      ¿Qué piensan del pronóstico catastrófico que aparece en Clarin?

      Habla del fin absoluto de la humanidad en este siglo.

      http://www.clarin.com/mundo/grados-abismo_0_569343154.html

      Lo que está fuera de lugar es la cita de Stern. Decir que va a ser el final de la humanidad es irresponsable. La realidad es que es muy difícil de saber cuánto nos va a impactar al largo plazo. Sabemos que va a ser jodido («desastre ecológico de consecuencias impredecibles» me parece que es correcto) pero en cuanto a la cuantificación de esos impactos, francamente no sé a qué nivel de certidumbre estamos. Pero aún así, para extinguir a la raza humana en su totalidad me parece que se va a necesitar más. Si pudimos sobrevivir la edad de hielo hace decenas de miles de años, me imagino que podremos sobrevivir un calentamiento global. Eso sí, que la humanidad «sobreviva» no dice nada de en qué condiciones.

    • #30657
      saibaba
      Miembro

      Bueno. Creo que una de las cosas negativos que tiene este tema del Calentamiento Global es que no viene acompañado de un análisis exacto de las consecuencias en el modo de vida de la gente, los problemas de salud, las inundaciones, el hambre, guerras, entre otras cosas.

      Si este análisis existe, no tiene la suficiente publicidad.

      O tal vez no tiene la suficiente fuerza lógica como para convencer a quienes toman decisiones políticas.

      En todo caso, lo que me parece a mí es que, el día que alguien se digne a hacer un análisis exacto del impacto social y ambiental del Calentamiento Global, ya va a ser tarde. Sólo va a servir para calcular cuánto sufrimiento nos queda por delante, y cuántas décadas vamos a vivir, y más o menos cuántos de nosotros tienen probabilidades de sobrevivir.

      Algo que denuncia el artículo de Clarin es que el informe que mencionan, en que muestran que se llegó ya a un punto de no retorno, parece que a nadie le importó.

      Yo pienso que se abusa de la complejidad teórica del problema de predecir qué es exactamente lo que va a suceder, para seguir especulando con las finanzas y los intereses económicos.

      El hecho de saber que el clima mundial se va a desestabilizar, quiere decir que todo nuestro modo de vida se volverá impredecible. Esta imprevisión de por sí ya es un asunto grave.

      No se trata de decir: «como no sabemos lo que va a pasar, es lo mismo que si no va a pasar nada». Esa parece ser la actitud de los más escépticos, o los más aferrados a sus intereses económicos.

      Cuando en realidad hay que preocuparse.

      Me parece más o menos igual de terrorífico decir que «la humanidad se va a extinguir este siglo» que decir «no tenemos ni idea de cómo se va a comportar el clima, si va a haber inundaciones o no, si va a haber grandes pestes o no, si va a haber grandes hambrunas, caos y guerras, o no, ni qué va a ser de nosotros».

      Si semejantes incertidumbres dependieran sólo de cuestiones políticas, es distinto, porque significa que la gente podría evitarlo también políticamente.

      Pero un clima desbocado y sus consecuencias no tienen solución política.

      Al clima no se le puede mentir, ni imponer regímenes de gobierno, ni plazos, ni fronteras.

      No sé. Me parece ver a un montón de babiecas que están como «fumados», que no reaccionan ante un problema que exige máxima acción y responsabilidad.

      Si querés llamale «hipérbole».

      Yo quiero sobrevivir, y que mis hijos, nietos y demás puedan vivir en un mundo normal, sin pestes, sin caos, y que no sea un «sálvese quien pueda».

      Si no se hace nada, no sé qué pesadilla va a ser el mundo porque no se puede calcular, pero que va a ser una pesadilla, es algo que no dudo.

    • #30658

      Es que ese análisis es muy difícil o imposible de hacer. Se pueden hacer proyecciones pero esas son tan exactas como lo es nuestro conocimiento y está más que claro que nuestro conocimiento no es perfecto. Hay muchísima gente estudiando cada parte del sistema climático y cómo el cambio climático va a afectar distintas variables. Pero si ya de por sí esos análisis tienen incertidumbres (lo que no quiere decir que sean inútiles) imaginate lo que es realizar un análisis global metiendo todas esas lineas de evidencias juntas. Lo mejor que tenemos es el Grupo de Trabajo II del IPCC.

      Como bien decís, las incertidumbres en el futuro no son razón para no actuar. De hecho, es todo lo contrario. La mayoría de los economistas expertos en el tema afirman que la incertidumbre es más razón para actuar rápido.

      Hay 3 cosas para hacer frente al cambio climático. Mitigación, Adaptación y Sufrimiento. Vamos a hacer un poco de cada una; la proporción de cada una depende de nosotros.

    • #30659

      Oportunamente, hace unos días se publicó esto:

      A.E. Viau, M. Ladd, K. Gajewski, The climate of North America during the past 2,000 years reconstructed from pollen data, Global and Planetary Change, Available online 2 October 2011, ISSN 0921-8181, 10.1016/j.gloplacha.2011.09.010.

      Abstract: The temperature of the warmest month was reconstructed for the past 2000 years using 748 pollen sites from the North American Pollen Database. The modern analogue technique was used to quantify paleoclimate conditions using a modern pollen database with calibration sites from across North America. Across North America, both the Medieval Warm Period (MWP) and Little Ice Age (LIA) were cooler than the present (AD1961-1990). The MWP was warmer than the LIA over at least the boreal and eastern portions of the continent and perhaps across the continent. These reconstructed anomalies during the MWP and LIA are significant anomalies from the long-term neoglacial cooling. The atmospheric circulation was likely dominated by a poleward shift of the summer Subtropical High Pressure system in the North Atlantic during the MWP.

      De nuevo está detrás de una pared de pago, pero esta creo que es la figura clave:

      UHBQH.gif

      Las zonas azules son las que actualmente son tienen mayor temperatura que durante la edad media.

      Igual es más para boludear que otra cosa porque, como dije, un sólo estudio de un sólo país difícilmente es concluyente. Para concluir, como algunos negacionistas quieren, que durante la edad media hubo mayor temperatura que durante las últimas décadas lo que hay que hacer es una reconstrucción global (o de 1 hemisferio, al menos) y comparar la temperatura media global en esa época con la de ahora. Y como vengo mostrando, eso ya se ha hecho y la conclusión es clara:

      HljFk.jpg

      http://www.ipcc.ch/publications_and_data/ar4/wg1/en/ch6s6-6.html

    • #30660
      smaga
      Miembro

      Daneel Olivaw dijo:

      Sí hay un buen análisis y lo linkeé varias veces.

      Yo linkeé otros.

      Daneel Olivaw dijo:

      También linkeé el gráfico que muestra cómo los modelos pueden explicar la altura de la tropopausa sólo si se tienen en cuenta los forzamientos humanos.

      Yo linkeé estudios que concluyen que la altura de la tropoapusa contiene limitada información sobre el cambio climático. Además, lo que me llama la atención de ese gráfico es la estabilidad de la anomalía antes de que se obtengan datos. ¿Por qué reacciona ante un erupción volcanica y no ante las oscilaciones del clima?

      Daneel Olivaw dijo:

      Que el nivel de información sea menor o mayor del que se pensaba antes no invalida la cuestión básica.

      No la invalida pero tampoco confirman a nuestras emisiones como la causa principal.

      Daneel Olivaw dijo:

      Y, de nuevo, el CO2 NO es el único factor importante en el clima y dado que los mismos papers que linkeaste vos presentan una explicación posible para los datos que NO implican la refutación del cambio climático antropogénico (la recuperación de la capa de ozono)

      Pero estamos repitiendo todo de nuevo. Ya lo argumenté antes.

      Daneel Olivaw dijo:

      y que no presentás ningún mecanismo alternativo para presentar la tendencia negativa general sin hacer relación al CO2, sigo sin ver dónde está la relación con él.

      Tampoco veo ninguna explicación para explicar el no enfriamiento en la mitad de los registros satelitales de la temperatura de la estratósfera, a pesar de que el CO2 va en aumento.

      Daneel Olivaw dijo:

      Más aún. Si tu argumento es que el CO2 NO está teniendo el efecto de enfriar la estratósfera y calentar la tropósfera, eso está en conflicto con la ciencia básica.

      Es que ese no es mi argumento, así que lo que le sigue está de más. El tema es CUANTO calentamiento se debe al CO2.

      Daneel Olivaw dijo:

      Puede ser que haya pegado el gráfico al revés, ¿cambia en algo mi punto? No. No importa cómo los compares, el gráfico que te venden en esa página omite el registro instrumental y, en efecto, esconde el aumento.

      Sí, estaba al revés y punto.

      Fijate que ahí tenés una posible explicación de la omisión: el registro instrumental no se podría comparar con los proxies. Y te repito, me gustaría leer el estudio completo.

      Creo que la conclusión del SMN de tomar al CGA como una hipótesis es totalmente válida.

    • #30661
      smaga
      Miembro

      … y de interesante lectura sobre períodos cálidos anteriores. Copio todo el abstract para que no haya acusaciones innecesarias:

      A new temperature reconstruction with decadal resolution, covering the last two millennia, is presented for the extratropical Northern Hemisphere (90–30°N), utilizing many palaeo-temperature proxy records never previously included in any large-scale temperature reconstruction. The amplitude of the reconstructed temperature variability on centennial time-scales exceeds 0.6°C. This reconstruction is the first to show a distinct Roman Warm Period c. ad 1–300, reaching up to the 1961–1990 mean temperature level, followed by the Dark Age Cold Period c. ad 300–800. The Medieval Warm Period is seen c. ad 800–1300 and the Little Ice Age is clearly visible c. ad 1300–1900, followed by a rapid temperature increase in the twentieth century. The highest average temperatures in the reconstruction are encountered in the mid to late tenth century and the lowest in the late seventeenth century. Decadal mean temperatures seem to have reached or exceeded the 1961–1990 mean temperature level during substantial parts of the Roman Warm Period and the Medieval Warm Period. The temperature of the last two decades, however, is possibly higher than during any previous time in the past two millennia, although this is only seen in the instrumental temperature data and not in the multi-proxy reconstruction itself. Our temperature reconstruction agrees well with the reconstructions by Moberg et al. (2005) and Mann et al. (2008) with regard to the amplitude of the variability as well as the timing of warm and cold periods, except for the period c. ad 300–800, despite significant differences in both data coverage and methodology.

      Más evidencia a favor del SMN.

    • #30662

      smaga dijo:

      Yo linkeé otros.

      No, de hecho no lo hiciste. Linkeaste a una página llena de gráficos locales que encima mostré que, al menos en un caso, es desinformación pura.

      Quote:
      Yo linkeé estudios que concluyen que la altura de la tropoapusa contiene limitada información sobre el cambio climático. Además, lo que me llama la atención de ese gráfico es la estabilidad de la anomalía antes de que se obtengan datos. ¿Por qué reacciona ante un erupción volcanica y no ante las oscilaciones del clima?

      No la invalida pero tampoco confirman a nuestras emisiones como la causa principal.

      A ver. Conocimiento anterior: la altura de la tropopausa es un dato útil para atribuir el clima. Dato que agregan los estudios: es menos útil de lo que creíamos. Dato que NO agregan: es completamente irrelevante.

      Quote:
      Tampoco veo ninguna explicación para explicar el no enfriamiento en la mitad de los registros satelitales de la temperatura de la estratósfera, a pesar de que el CO2 va en aumento.

      Entonces serás ciego y no leíste ni tus propios papers. ¿Por qué no hay enfriamiento estratosférico a pesar de que el Co2 sigue aumentando? Porque, como ya lo repito muchas veces, el CO2 no es el único factor del clima. Los propios papers que citaste indican una explicación: recuperación de la capa de ozono.

      Quote:
      Es que ese no es mi argumento, así que lo que le sigue está de más. El tema es CUANTO calentamiento se debe al CO2.

      Señalás que la estratósfera no se enfrió ni la tropopausa aumentó en altura desde el 96 a pesar de que el CO2 sigue aumentando. Sabemos, por primeros principios, que el aumento en el CO2 va a producir el efecto que, segun vos, no se ve desde el 96. ¿Y entonces?

      Quote:
      Sí, estaba al revés y punto.

      Fijate que ahí tenés una posible explicación de la omisión: el registro instrumental no se podría comparar con los proxies. Y te repito, me gustaría leer el estudio completo.

      Podrían haber puesto el registro instrumental para, al menos, hacer notar que el proxi no es representativo. Pero no, eligieron omitirlo. Interesante como le das todo el beneficio de la duda a este tipo que en ningún lugar siquiera comentó que la reconstrucción de ese lago en China está LEJOS de ser repersentativa (al menos en la actualidad), pero le quejás de que Mann «escondió» el problema de la divergencia aún cuando está aclarado en la literatura científica, incluido el propio IPCC.

      smaga dijo:

      … y de interesante lectura sobre períodos cálidos anteriores.

      El gráfico que está en el paper que citás:

      DNSME.gif

      Y del abstract:

      «La temperatura de las últimas dos décadas, sin embargo, es posiblemente más alta que durante cualquier período de los últimos 2000 años, aunque esto sólo se ve en el registro instrumental y no en la reconstrucción en sí misma.

      Además, te comento. Que la temperatura actual sea o no más alta que la temperatura en el pasado nada tiene que ver con las causas de ese aumento. Yo pude tener mucha más fiebre hace 3 años que hoy, pero eso no dice NADA de las CAUSAS de la fiebre de hoy.

    • #30663
      smaga
      Miembro

      Daneel Olivaw dijo:

      No, de hecho no lo hiciste. Linkeaste a una página llena de gráficos locales que encima mostré que, al menos en un caso, es desinformación pura.

      No es desinfomación, son datos.

      Daneel Olivaw dijo:

      A ver. Conocimiento anterior: la altura de la tropopausa es un dato útil para atribuir el clima. Dato que agregan los estudios: es menos útil de lo que creíamos. Dato que NO agregan: es completamente irrelevante.

      Entonces no sería una huella innegable del CGA. Y mantengo la pregunta: ¿por qué reacciona ante un erupción volcanica y no ante las oscilaciones del clima?

      Daneel Olivaw dijo:

      Entonces serás ciego y no leíste ni tus propios papers. ¿Por qué no hay enfriamiento estratosférico a pesar de que el Co2 sigue aumentando? Porque, como ya lo repito muchas veces, el CO2 no es el único factor del clima. Los propios papers que citaste indican una explicación: recuperación de la capa de ozono.

      Eso no explica el enfriamianto de la estratósfera como huella innegable, por eso lo pregunto de nuevo.

      Yo escribí «…Si la tendencia se revirtió posiblemente por la recuperación del ozono, el enfriamiento se produjo entonces mayormente por la pérdida de este último, por lo tanto, no habría un claro indicador del CGA…»

      Las acusaciones están de más.

      Daneel Olivaw dijo:

      Señalás que la estratósfera no se enfrió ni la tropopausa aumentó en altura desde el 96 a pesar de que el CO2 sigue aumentando. Sabemos, por primeros principios, que el aumento en el CO2 va a producir el efecto que, segun vos, no se ve desde el 96. ¿Y entonces?

      La estratósfera no se está enfriando desde el 96 (la mitad de los registros satelitales) y la tropopausa contiene limitada información sobre el cambio climático. Son conceptos que no entran en conflicto con la ciencia básica: el CO2 es un gas de efecto invernadero.

      Daneel Olivaw dijo:

      Podrían haber puesto el registro instrumental para, al menos, hacer notar que el proxi no es representativo. Pero no, eligieron omitirlo. Interesante como le das todo el beneficio de la duda a este tipo que en ningún lugar siquiera comentó que la reconstrucción de ese lago en China está LEJOS de ser repersentativa (al menos en la actualidad), pero le quejás de que Mann «escondió» el problema de la divergencia aún cuando está aclarado en la literatura científica, incluido el propio IPCC.

      ¿Qué cosas, no? Hay quejas por las mismas razones en ambos lados. Te repito otra vez, no puedo leer el estudio completo dado que no tengo acceso académico.

      Como podrían, sí, lo podrían haber puesto, igual que podrían haber puesto el gráfico de Briffa.

      Daneel Olivaw dijo:

      Y del abstract:

      «La temperatura de las últimas dos décadas, sin embargo, es posiblemente más alta que durante cualquier período de los últimos 2000 años, aunque esto sólo se ve en el registro instrumental y no en la reconstrucción en sí misma.

      El mayor calentamiento se ve con los termómetros, no con los proxies.

      Daneel Olivaw dijo:

      Además, te comento. Que la temperatura actual sea o no más alta que la temperatura en el pasado nada tiene que ver con las causas de ese aumento. Yo pude tener mucha más fiebre hace 3 años que hoy, pero eso no dice NADA de las CAUSAS de la fiebre de hoy.

      Pero entonces, ¿por qué escribiste esto? «… Para empezar, las temperaturas están ACTUALMENTE mucho más altas de lo que estuvieron en los útlimos miles de años y MUY por encima de la variabilidad natural…»

    • #30664

      Voy a seguir repitiendo este gráfico mientras sigas con lo de la temperatura estratosférica y la tropopausa. Este patrón sólo se puede explicar teniendo en cuenta los forzantes antropogénicos. El agujero de ozono es uno de ellos, como también lo son los gases de efecto invernadero.

      iNYXR.png

      Todo lo demás son detalles que no refutan esta conclusión de primer orden, sólo la refinan.

      Los datos no son información en sí mismos y sí pueden ser desinformación. Si hoy te digo que el mínimo de extensión de hielo Ártico de septiembre de este año fue de 4,6 millones de kilómetros (si mal no recuerdo), te estoy dando un dato pero no te informo un carajo, porque es un dato sin contexto. Y si te digo que en 2008, 2009 y 2010 el hielo del Ártico se recuperó, te estoy dando desinformación, ya que ignora la clara tendencia negativa de todo el resto del registro.

      En este caso está más que claro como al mostrar sólo algunos datos y ocultar otros están desinformando. No sólo presentan datos dispersos sin análisis alguno (y NO muestran los análisis que sí existen) sino que, en el caso específico que seleccioné (de nuevo, al azar), presentaron una figura completamente engañosa que sugiere que las temperaturas actuales están muy cercanas al promedio de los últimos 2000 años cuando en realidad están muy por encima. Para colmo NO notaron que el registro de termómetro muestra algo completamente distinto y NO lo anotaron en un epígrafe.

      Y si por «briffa» te referís a este paper, no es necesario más que leer el abstract y los gráficos ahí publicados para ver que plotearon el registro de termómetros encima de la reconstrucción. Además dentro del paper se habla abiertamente del problema de la divergencia:

      «El período luego de 1960 no fue utilizado [como calibración] para evitar sesgos en los coeficientes de regresión que se generarían por la reducción anómala de densidad de aniños en las décadas recientes que no es forzada por la temperatura [Briffa et al., 1998b]»

      y

      «[Es evidente] la divergencia gradual en la tendencia durante las últimas décadas mencionada con anterioridad y discutida en Briffa et al. (1998b, 2000)»


      Sólo por diversión, elegí otro gráfico de la página del MWP. Esta también fue al azar (tan al azar como puede serlo una persona, en realidad). Se trata de Thompson et al 2003. Esta es la imagen que presentan.

      USU23.gif

      Aún si lo tomamos como válido, no es demasiado convincente. Hay 1 (un) pico que es más cálido que el dato más reciente pero ni siquiera es significativo ya que todo el resto está por debajo y, para colmo, la tendencia promediada ni siquiera supera la línea roja. Obviamente no está el registro instrumental para comparar y el eje vertical no es temperatura sino porcentaje de oxígeno 18 así que es difícil de comparar. Además se supone que es un núcleo de hielo de Sudamérica y no sé si hay reconstrucciones hemisféricas del hemisferio sur que lleguen tan lejos. En cualquier caso, si miramos el paper en cuestión, vemos que trata no sólo de ese núcleo de hielo sino de varios otros. Me pregunto por qué no mostraron esta figura:

      xksln.gif

      Donde se pinta otra imagen. Con más datos, puestos en contexto (información), vemos que, nuevamente, nos tratan de vender ruido por datos.

      En el paper también dicen:

      «El registro compuesto de baja latitud muestra que en el siglo XX se están registrando cambios únicos cuando se los ve desde la perspectiva de los últimos mil años. Los núcleos de hielo también contienen archivos de variabilidad climática y ambiental de escalas de décadas a miles de años y proveen una introspección única tanto en eventos regionales como globales, desde la llamada «pequeña edad de hielo», la fase fría del Younger Dryas hasta la edad glacial tardía. Los datos presentados aquí claramente demuestran que algunos, sino todos, estos archivos únicos se encuentran en peligro inminente de perderse de persistir el calentamiento actual.»


      Y porque me resulta muy gracioso, agarré otro. Rain 2005 que muestra una imagen muy convincente de que la temperatura de la superficie marítima era mucho más alta durante el MWP. Pero el problema es que el título del paper es «El Niño variability off Peru during the last 20,000 years». No es un estudio que busca hacer una reconstrucción de la temperatura, sino sobre la actividad de El Niño/La Niña. Habrá que ver qué tan útil es la temperatura de la superficie marina en una zona que se ve tan afectada por un proceso tan dramático.

      En cualquier caso, me llamó la atención la última línea del paper:

      «Por lo tanto, una debilidad marcada de El Niño durante el Período Cálido Medieval que debe haber enfriado grandes partes de las bajas latitudes sugiere que el clima en los tiempos medievales sólo pueden ser de utilidad limitada como una analogía en las recientes discusiones sobre cambio climático»

    • #30665
      Anónimo
      Inactivo
    • #30666
      N3RI
      Superadministrador
    • #30667
      saibaba
      Miembro

      No entiendo por qué smaga sólo acepta datos parciales, en vez de procurar análisis de datos que muestren un panorama completo, tal como cualquier persona decente haría.

      Yo también podría mostrar los datos estadísticos del rincón de la terraza donde vivo, que nunca le da el Sol, y mostrar una gráfica en la que ahí no afecta el calentamiento global ni la capa de ozono ni nada.

      Hay que tener en cuenta todos los datos, y no sólo los que convienen a tu argumento.

      Para mí es claro que alguien que se niega a propósito en este punto, es un mentiroso.

      ___________

      No tengo ni idea de los argumentos y contraargumentos que se usan en temas climáticos,

      pero tengo las nociones básicas de estadística que tiene todo el mundo.

      Y no hace falta ser muy ducho para darse cuenta que si mostrás «sólo unos datitos forros», sin tener en cuenta «TODOS LOS PUTOS DATOS», entonces es que estás tratando de meterle a la gente un «buzón por el OGT», como un gran HDP.

      Queda feo expresarse así, pero creo que así se entiende más claro lo que está pasando.

      Para decirlo finamente: «poner en evidencia la naturaleza íntima de la falacia».

    • #30668
      N3RI
      Superadministrador
    • #30669

      N3RI dijo:

      http://cnn-ecosphere.com/

      «Sorry, it appears your browser does not support WebGL.

      Please note: some older graphics cards may be unable to load the experience.»

      Me cago en Ubuntu y sus pésimos drivers de video.

    • #30670
      N3RI
      Superadministrador

      Daneel Olivaw dijo:

      «Sorry, it appears your browser does not support WebGL.

      Please note: some older graphics cards may be unable to load the experience.»

      Me cago en Ubuntu y sus pésimos drivers de video.

      yo lo veo en Ubuntu perfectamente, qué placa tenés? nvidia? tiene q andar… además WebGL es estandar :P

    • #30671

      ATI HD 3200 y Google Chrome 15 :S

    • #30672
      smaga
      Miembro

      Daneel Olivaw dijo:

      Voy a seguir repitiendo este gráfico mientras sigas con lo de la temperatura estratosférica y la tropopausa.

      A ver, de uno de tus link «…Ambos procesos resultan en un enfriamiento de la estratósfera y un calentamiento de la tropósfera, lo que tiene, como consecuencia que la primera se contraiga y la segunda se expanda, elevando la altura de la tropopausa…» , pero teniendo en cuenta que la primera NO se está enfriando, no habría entonces una «huella innegable».

      Daneel Olivaw dijo:

      Este patrón sólo se puede explicar teniendo en cuenta los forzantes antropogénicos. El agujero de ozono es uno de ellos, como también lo son los gases de efecto invernadero.

      Con los MODELOS no se podría explicar, y además es sabido que en ocasiones los modelos tienen fallas en reproducir los datos reales.

      Ahora, dos observaciones:

      – Sabemos que la altura de la tropopausa depende de las temperaturas de la estratosfera y troposfera, ¿por qué los modelos no representan los períodos mas cálidos y más fríos antes del 1960?

      – ¿Qué pasa si nos extendemos hasta el año 2011, donde sabemos que la temperatura de la estratósfera mantiene la tendencia a no enfriarse desde el 96?, e insisto, NO es cherry picking «…The considered 15-yr period is

      not very long but sufficient as a basis for a trend calculation…» http://journals.ametsoc.org/doi/pdf/10.1175/1520-0493(1999)127%3C2248%3ATHAWAT%3E2.0.CO%3B2

      Todo esto sabiendo que nuestras emisiones de CO2 siguen en eumento.

      Daneel Olivaw dijo:

      Todo lo demás son detalles que no refutan esta conclusión de primer orden, sólo la refinan.

      Si provee limitada informacio sobre el CGA, no estaría clara la «huella innegable», y no considero que el resultado de un modelo sea una conclusión «de primer orden», por ejemplo, las proyecciones de GISS sobre el contenido calórico de los océanos http://bobtisdale.wordpress.com/2011/05/08/first-quarter-2011-update-of-nodc-ocean-heat-content-0-700meters/

      Daneel Olivaw dijo:

      En este caso está más que claro como al mostrar sólo algunos datos y ocultar otros están desinformando.

      No lo ocultan, en cada gráfico tenés en el título el lugar desde donde provienen los datos y además, ¿cuántos pedidos de FOI hiciste para tener los datos?. Creo que, a diferencia de McIntyre, ninguno.

      Y siguen las evidencias afavor del SMN: http://arxiv.org/PS_cache/arxiv/pdf/1110/1110.1841v1.pdf

    • #30673

      Uh, me revivís una discusión de hace meses. Ya ni me acuerdo de qué carajo hablábamos.

      Me estás diciendo que las tendencias de temperaturas estratosféricas no son evidencia de calentamiento global antropogénico. Como te mostré, los propios papers que citás dicen lo contrario; que las tendencias observadas son perfectamente explicables. Además te di el link del IPCC que muestra que la tendencia NO puede ser explicada mediante factores naturales únicamente.

      También hacés incapié que todo esto sucede mientras las concentraciones de CO2 siguen en aumento, pero también la capa de ozono se está recuperando, las partículas en suspensión (aerosoles) están en aumento y todos los demás factores climáticos que yo no conozco siguen vigentes.

      Entonces, tenemos:

      * la tendencia no puede ser explicada mediante factores naturales,

      * la tendencia concuerda con los modelos que contemplan el forzamiento antropogénico, y

      * no hay hipótesis alternativa que explique los datos mejor que el forzamiento antropogénico.

      Y voy a voler a poner el gráfico:

      iNYXR.png

      Ahí se se palpablemente como si tomamos sólo los factores naturales los modelos no pueden replicar las observaciones pero sí lo hacen cuando se agregan los factores antropogénicos.

      El paper que citaste no está publicado en una revista con revisión por pares. Arxiv es un servicio web al cual cualquiera puede subir su trabajo. Además el European Institute for Climate and Energy no es más que una organización negacionista y Energy & Envoriment (la revista en la cual se supone que se publicó el paper del cual el link que pasás es la infroamción suplementaria) ni siquiera está listado en Web of Knowledge y la editora explícitamente dice que sigue una agenda política. Para peor, el autor Horst-Joachim Lüdecke sostiene que el calentamiento global es de origen Solar, algo completamente insostenible.

      Decir que las evidencias se están acumulando en contra del origen humano del calentamiento global es ignorar por completo la enorme mayoría de la evidencia.

      Quote:
      No lo ocultan, en cada gráfico tenés en el título el lugar desde donde provienen los datos

      Mann elimina datos que sabe que son erróneos basado en un efecto ampliamente conocido y debatido y lo explicita en el paper y el gráfico; eso es «ocultar el declive».

      El tipo este publica un registro de temperaturas en China y omite los datos que se sabe son correctos (el registro instrumental) y que contradicen su posición (que durante el medioevo la temperatura fue mayor que ahora globalmente) y «no ocultan nada».

      Interesante.

      ¿Leíste el resto de los links que busqué las fuentes? Ahí hay otros ejemplos de «selección de datos» y citar papers que concluyen exactamente lo contrario que el autor del sitio web quiere sostener.

    • #30674

      Siguen las evidencias completamente en contra del SMN:

      Anthropogenic and natural warming inferred from changes in Earth’s energy balance

      Tiene este gráfico, muy similar a uno que ya había posteado:

      nBMFC.png

      Y esta cita:

      Quote:
      Nuestros resultados muestran que es extremadamente problable que al menos 74% (+- 12%) the el calentamiento observado desde 1950 haya sido causado por forzamientos radiativos y menos de 26% (+- 12%) por variabilidad interna no forzada

      Our results show that it is extremely likely that at least 74% (+/- 12%, 1 sigma) of the observed warming since 1950 was caused by radiative forcings, and less than 26% (+/- 12%) by unforced internal variability.

      Y estas son las proporciones relativas de forzamientos:

      8Xeps.png

      Como se ve, el forzamiento antropogénico gana por goleada.

    • #30675

      Nuevo episodio de la serie «Escepticos» sobre el Cambio Climatico: http://bcove.me/vfsgppem

      Me asuste cuando lei el titulo :P

    • #30676

      Jajaja. Cuando anunciaron por twitter la lista de programas (hace unos meses) yo también me asusté xD.

    • #30677

      Muy bueno el capítulo. Lástima por la minita de Greenpeace que no perdió la oportunidad para pegarle palos a la energía nuclear. También hay un pequeño error. Gámez dice que el derretimiento del hielo del ártico va a producir un aumento del nivel del mar; esto es falso. El hielo ártico está flotando sobre el agua y al terretirse pasa a ocupar el mismo volumen que antes desplazaba. El aumento del nivel del mar se da principalmente por la dilatación térmica (al aumentar la temperatura aumenta el volumen) y el derretimiento de los hielos continentales como Groenlandia.

      Si vamos a ser aún más quisquillozos, el hielo del ártico contribuye un poco al nivel del mar. El agua salada es más densa que el agua dulce. Al congelarse, el agua pierde toda su salinidad, por lo que los hielos son de agua sin sales. Entonces cuando se descongelan, los hielos disminuyen la salinidad oceánica (un problema serio por derecho propio) disminuyendo la densidad y, por lo tanto, aumentando mínimamente el nivel del mar. Haciendo las cuentas, poco más de un 1% del aumento del nivel del mar se debe al derretimiento. No es significativo, pero es interesante :P (fuente de todo eso que dije: http://www.skepticalscience.com/Sea-level-rise-due-to-floating-ice.html y referencias ahí adentro)

      La música del final es una que se usa en Cosmos, si mal no recuerdo. :D

    • #30678
      N3RI
      Superadministrador

      ¿Un programa de Escépticos dedicado entero a favor del Cambio Climático? ¡Indignante! ¡Gallegos alarmistas! ¿Qué piensan de estos los escépticos… oh wait!

      Seguro que a este programa lo pagan las empresas fabricantes de molinos y paneles solares, que según me contaron, están dominando España. :P

      Se nota que el pelado ese, sea quien sea, tiene cara de no ser lo suficientemente escéptico :P seguro que es un actor pagado, como esos que salen en la tele diciendo que usemos tal o cual producto magufo. El pelado ese debe ser un actor, joder!

    • #30679
      smaga
      Miembro

      Daneel Olivaw dijo:

      Uh, me revivís una discusión de hace meses. Ya ni me acuerdo de qué carajo hablábamos.

      No nos vamos a poner a discutir las fechas…

      Daneel Olivaw dijo:

      Me estás diciendo que las tendencias de temperaturas estratosféricas no son evidencia de calentamiento global antropogénico.

      Te lo digo con pruebas.

      Daneel Olivaw dijo:

      Como te mostré, los propios papers que citás dicen lo contrario;

      No, ya que dicen que la estratósfera no se está enfriando.

      Daneel Olivaw dijo:

      que las tendencias observadas son perfectamente explicables.

      Si tomamos esa explicación para la tendencia del comienzo de la toma de datos, no habría una huella innegable del CGA.

      Daneel Olivaw dijo:

      Además te di el link del IPCC que muestra que la tendencia NO puede ser explicada mediante factores naturales únicamente.

      No puede ser explicada mediante MODELOS.

      Daneel Olivaw dijo:

      También hacés incapié que todo esto sucede mientras las concentraciones de CO2 siguen en aumento, pero también la capa de ozono se está recuperando, las partículas en suspensión (aerosoles) están en aumento y todos los demás factores climáticos que yo no conozco siguen vigentes.

      Bueno, entonces ¿cuál es la participación de cada factor «vigente»?

      Daneel Olivaw dijo:

      Entonces, tenemos:

      * la tendencia no puede ser explicada mediante factores naturales,

      Con los modelos, no.

      Daneel Olivaw dijo:

      * la tendencia concuerda con los modelos que contemplan el forzamiento antropogénico, y

      Repito:

      – Sabemos que la altura de la tropopausa depende de las temperaturas de la estratosfera y troposfera, ¿por qué los modelos no representan los períodos mas cálidos y más fríos antes del 1960?

      – ¿Qué pasa si nos extendemos hasta el año 2011, donde sabemos que la temperatura de la estratósfera mantiene la tendencia a no enfriarse desde el 96?

      Daneel Olivaw dijo:

      * no hay hipótesis alternativa que explique los datos mejor que el forzamiento antropogénico.

      Idem 1.

      Daneel Olivaw dijo:

      El paper que citaste no está publicado en una revista con revisión por pares. Arxiv es un servicio web al cual cualquiera puede subir su trabajo. Además el European Institute for Climate and Energy no es más que una organización negacionista y Energy & Envoriment (la revista en la cual se supone que se publicó el paper del cual el link que pasás es la infroamción suplementaria) ni siquiera está listado en Web of Knowledge y la editora explícitamente dice que sigue una agenda política. Para peor, el autor Horst-Joachim Lüdecke sostiene que el calentamiento global es de origen Solar, algo completamente insostenible.

      Este es el paper (10 segundos de búsqueda) http://multi-science.metapress.com/content/6787484610l220h0/ que obviamenete dice prácticamente lo mismo. Todo lo demás son acusaciones sin tener en cuenta el paper en sí, salvo la hipótesis solar, que como el CGA, son hipótesis

      Daneel Olivaw dijo:

      Decir que las evidencias se están acumulando en contra del origen humano del calentamiento global es ignorar por completo la enorme mayoría de la evidencia.

      Volvemos al tema de la «mayoría»……

      Daneel Olivaw dijo:

      Mann elimina datos que sabe que son erróneos basado en un efecto ampliamente conocido y debatido y lo explicita en el paper y el gráfico; eso es «ocultar el declive».

      Sigue sin aparecer en el gráfico.

      Daneel Olivaw dijo:

      El tipo este publica un registro de temperaturas en China y omite los datos que se sabe son correctos (el registro instrumental) y que contradicen su posición (que durante el medioevo la temperatura fue mayor que ahora globalmente) y «no

      ocultan nada».

      Interesante.

      De vuelta, ¿cuántos FOI´s hiciste?

      Daneel Olivaw dijo:

      ¿Leíste el resto de los links que busqué las fuentes? Ahí hay otros ejemplos de «selección de datos» y citar papers que concluyen exactamente lo contrario que el autor del sitio web quiere sostener.

      Claro que sí, y te respondí tambien.

      Una de las cosas que no aclaraste es tu aparente contradicción:

      -«…Además, te comento. Que la temperatura actual sea o no más alta que la temperatura en el pasado nada tiene que ver con las causas de ese aumento…»

      -«…Para empezar, las temperaturas están ACTUALMENTE mucho más altas de lo que estuvieron en los útlimos miles de años y MUY por encima de la variabilidad natural…»

      Daneel Olivaw dijo:

      Siguen las evidencias completamente en contra del SMN:

      Me da gracia el «completamente»….

      Y sí, hay evidencia que apoya una hipótesis y otras que no.

      Daneel Olivaw dijo:

      Tiene este gráfico, muy similar a uno que ya había posteado:

      Interesante, sobre todo el período de calentamiento de pricipios del siglo 20 y la actual «meseta» de principios de este

      siglo.

    • #30680

      No voy a ir a buscar nuevamente las citas de los papers que linkeaste, smaga. La verdad que me cansa esto. Leelos y vas a ver que…. bueh, los busco y pego:

      Quote:
      Because of the combined effect of tropospheric warming by greenhouse gases and stratospheric cooling by ozone depletion, the global tropopause pressure (height) has been decreasing (increasing) over last two decades (Randel et al. 2000; Seidel et al. 2001; Sausen and Santer 2003; Santer et al. 2003a,b, 2004; Seidel and Randel 2006)

      Quote:
      Temperature changes in the lower stratosphere show cooling of $0.5 K/decade over much of the globe for 1979–2007, with some differences in detail among the different radiosonde and satellite data sets. Substantially larger cooling trends are observed in the Antarctic lower stratosphere during spring and summer, in association with development of the Antarctic ozone hole.

      Quote:
      Using such CCM integrations, it is found that the tropopause pressure (height) will continue to decrease (increase) in the future, but with a trend weaker than that in the recent past. The reduction in the future tropopause trend is shown to be directly associated with stratospheric ozone recovery.

      Pero bueno, el que no quiere ver…

      Quote:
      Este es el paper (10 segundos de búsqueda) http://multi-science.metapress.com/content/6787484610l220h0/ que obviamenete dice prácticamente lo mismo. Todo lo demás son acusaciones sin tener en cuenta el paper en sí, salvo la hipótesis solar, que como el CGA, son hipótesis

      Como te dije, está publicado en Energy & Enviroment, que no es una revista reconocida como con revisión por pares por la Web of Knowledge. Pero leé el abstract.. ¡analizaron 5 (cinco) estaciones meteorológicas. Y ni hablar que su conclusión es que el aumento en la temperatura no se debe a la variabilidad natural y luego sigue, sin demasiada justificaicón, que «estos resutlados contradicen la hipótesis de que el calentamiento global del siglo 20 sea inusual (antropogénico)». ¿!QUÉ!? No hay análisis de «huellas», no hay modelos, no hay análisis de atribución, no hay nada… sólo aseveraciones infundadas. Por otro lado, tenemos miles de estudios de atribución que señalan que la mayoría del calentamiento global reciente es muy probablemente antropogénico (y en mi post anterior linkee a uno de ellos).

      No sé qué carajo tienen que ver los FOI con la deshonestidad de la página que posteaste sobre el Período Cálido Medieval. Es un Arenque Rojo. Lo importante sigue en pie, los datos que presenta son desinformadores y omite deliberadamente (porque no pone gráficos que están claramente visibles en los papers que cita) información importante que contradice lo que quiere probar

      Quote:
      Interesante, sobre todo el período de calentamiento de pricipios del siglo 20 y la actual «meseta» de principios de este

      SkepticsvRealistsv3.gif

      Quote:
      Y sí, hay evidencia que apoya una hipótesis y otras que no.

      Ajá. ¿Y? ¿Cómo proponés evaluar hacia dónde apunta la mayoría de la evidencia?

    • #30681
      saibaba
      Miembro

      ¿O sea que smaga está citando publicaciones sin referato?

      ¿Cómo se puede dar el lujo de plantear argumentos que supuestamente tienen fundamento serio?

      Además alegremente dice el tipo cosas como: «Y… hay distintas hipótesis»,

      como si todas fueran igual de válidas.

      Yo si quiero puedo decir que el calentamiento global lo produce el semisextil de Plutón y Neptuno desde Capricornio…

      Si total, es una «hipótesis» como cualquier otra.

      Otra hipótesis: los extraterrestres han frenado el acelerado calentamiento global a fin de amortiguar el cambio hacia la nueva era, que ya se viene. Evidencias y publicaciones: las miles de notas y libros sobre el advenimiento de la nueva era, los mensajes de los mayas, y en fin, toda esa evidencia que «ahí está, pero smaga no la quiere ver».

      Otra hipótesis válida: La biblia habla de Falsos profetas en el fin de los tiempos. Obviamente estos que pregonan que se acaba el mundo por culpa del no sé qué antropogénico son falsos profetas que ocultan la verdad de que sólo Dios decide cuándo se acaba el mundo y la raza humana. Por lo tanto, esta evidencia bíblica, que lleva 2000 años de vigencia, y que está ahí publicada en todos los idiomas para el que quiera verla, nos dice que el calentamiento global es una mentira que sale de la boca del diablo para desesperanzar a los hombres, y poner a prueba su alma.

      No sé. Si es por soltar hipótesis, hay muchas pelotudeces por ahí.

      ____________

      Yo me he quejado y suelo quejarme de que el sistema de publicaciones científicas tiene defectos y aspectos molestos y hasta anticientíficos (no voy a entrar en detalles acá).

      Pero lo publicado, bien publicado está, y es válido.

      Y en cambio, las publicaciones no-científicas no pueden aceptarse bajo ningún aspecto,

      ni tampoco es honesto introducirlas en una discusión como esta.

      Eso de citar publicaciones sin referato es de puro estafador.

      ______________

      Aquellos que ignoramos los detalles complicados que tiene este tema del calentamiento global necesitamos información confiable, y que la gente que discuta lo haga con seriedad.

      A mí me interesa informarme y aprender sobre este tema.

      Pero me indigna tener que aguantar argumentos totalmente infundados como los de smaga.

      No sé por qué hay que aguantarlo.

      ¿Qué pretende hacer?

    • #30682

      Na, tampoco es tan así. Enery & Enviroment dice tener revisión por pares y no me consta que no sigan el procedimiento. Pero Homeopathy tambíen dice referato. El problema es que E&E es una revista sospechosa por publicar papers paupérrimos que no se publicarían en otro lado y su directora es una «escéptica» del cambio climático que explícitametne afirmó «seguir una agenda política».

      El paper que cita smaga es un ejemplo clarísimo. Analizando 5 (cinco) estaciones meteorológicas y aparentemente sin hacer ningún análisis de atribución, concluye que sus resultados contradicen el origen antropogénico del calentamiento global y presenta una hipótesis que fue refutada hace décadas.

    • #30683
      saibaba
      Miembro

      O sea que tiene «revisión por pares» pero esos pares son «amigos de la editora».

      Ese tipo de cosas ya son muy difusas, porque hay que entrar en la paranoia.

      ¿Cómo hago para saber que la revisión por pares de otras revistas es realmente fiable?

      Al final, voy a terminar discutiendo todo igual que smaga, jaja.

      Este tema parece ser un terreno muy pantanoso.

    • #30684

      Sí, es sutil y medio difícil para el lego que está ajeno a todas esas peleas internas y secretos a voces que hay en toda profesión humana, la ciencia incluida.

      Igual paso para compartir este paper que salió hace poco dicutiendo sobre el (supuestamente oculto) problema de la divergencia según el cual el grosor de los anillos de algunos árboles comienzan a perder su correlación con la temperatura de verano. Me llamó la atención por el título :P

      Who is the new sheriff in town regulating boreal forest growth? A Park Williams et al 2011 Environ. Res. Lett. 6 041004.

      PD: es de acceso gratuito.

    • #30685
      smaga
      Miembro

      Daneel Olivaw dijo:

      No voy a ir a buscar nuevamente las citas de los papers que linkeaste, smaga. La verdad que me cansa esto. Leelos y vas a ver que…. bueh, los busco y pego:

      Te olvidaste de este: «…This study presents evidence that the stratosphere is slightly warming since 1996…» y también cuando dicen que la altura de la tropopausa contiene limitada información sobre el CGA. Así que como «huella innegable» deja mucho que desear.

      Daneel Olivaw dijo:

      Como te dije, está publicado en Energy & Enviroment, que no es una revista reconocida como con revisión por pares por la Web of Knowledge. Pero leé el abstract.. ¡analizaron 5 (cinco) estaciones meteorológicas. Y ni hablar que su conclusión es que el aumento en la temperatura no se debe a la variabilidad natural y luego sigue, sin demasiada justificaicón, que «estos resutlados contradicen la hipótesis de que el calentamiento global del siglo 20 sea inusual (antropogénico)». ¿!QUÉ!? No hay análisis de «huellas», no hay modelos, no hay análisis de atribución, no hay nada… sólo aseveraciones infundadas. Por otro lado, tenemos miles de estudios de atribución que señalan que la mayoría del calentamiento global reciente es muy probablemente antropogénico (y en mi post anterior linkee a uno de ellos).

      Si se publica un paper refutando a Lüdecke, no sería válido entonces, mientras tanto, tus críticas son solo eso, críticas.

      Daneel Olivaw dijo:

      No sé qué carajo tienen que ver los FOI con la deshonestidad de la página que posteaste sobre el Período Cálido Medieval. Es un . Lo importante sigue en pie, los datos que presenta son desinformadores y omite deliberadamente (porque no pone gráficos que están claramente visibles en los papers que cita) información importante que contradice lo que quiere probar

      ¿Deshonestidad? Decime dónde en esa página dicen que el período cálido medieval fué más caliente globalmente que en la actualidad. Si no leo mal, el título dice «Período Cálido Medieval: un fenómeno global», para el argumento de los que apoyan la hipótesis del CGA sobre ese período «solo» en el hemisferio norte.

      Respecto del gráfico de SS, así es como lo veo yo: http://www.woodfortrees.org/plot/hadcrut3vgl/from:1850/to:2011/mean:10/plot/hadcrut3vgl/from:1910/to:1940/trend/plot/hadcrut3vgl/from:1980/to:2010/trend/plot/none

      Principios de siglo 20: tendencia similar de calentamiento de fines del mismo siglo y comienzos de este, aún cuando nuestras emisiones eran muy inferiores a las de hoy.

      Daneel Olivaw dijo:

      Ajá. ¿Y? ¿Cómo proponés evaluar hacia dónde apunta la mayoría de la evidencia?

      Otra vez eso de «la mayoría»….

      Y, repito:

      «…y todos los demás factores climáticos que yo no conozco siguen vigentes…»

      Bueno, entonces ¿cuál es la participación de cada factor «vigente»?

      Sobre los modelos que linkeaste tantas veces:

      – Sabemos que la altura de la tropopausa depende de las temperaturas de la estratosfera y troposfera, ¿por qué los modelos no representan los períodos mas cálidos y más fríos antes del 1960?

      – ¿Qué pasa si nos extendemos hasta el año 2011, donde sabemos que la temperatura de la estratósfera mantiene la tendencia a no enfriarse desde el 96?

      Y por último:

      Una de las cosas que no aclaraste es tu aparente contradicción:

      -«…Además, te comento. Que la temperatura actual sea o no más alta que la temperatura en el pasado nada tiene que ver con las causas de ese aumento…»

      -«…Para empezar, las temperaturas están ACTUALMENTE mucho más altas de lo que estuvieron en los útlimos miles de años y MUY por encima de la variabilidad natural…»

      Saibaba, si tenés ganas, buscá la definición de hipótesis científica.

    • #30686
      smaga
      Miembro

      saibaba, la estratósfera dejó de enfriarse, fuente http://www.jstage.jst.go.jp/article/sola/5/0/53/_pdf

      Decime por qué ese argumento está «infundado»

      No leas mis mensajes si no tenes ganas de aguantarme, fácil.

    • #30687

      Principios de siglo 20: tendencia similar de calentamiento de fines del mismo siglo y comienzos de este, aún cuando nuestras emisiones eran muy inferiores a las de hoy.

      Ajá.. ¿y quién es el que dice que las emisiones de CO2 son el único factor que afecta el clima?

      Igual, gracias por darme la razón. Tu selección de datos es sorprendente. Yo, como un buen escéptico, prefiero ver TODA la evidencia, no seleccionar los datos para que me den la razón.

      Quote:
      Otra vez eso de «la mayoría»….

      Sí, la mayoría. Como dije, el verdadero escéptico analiza TODA la evidencia y ve hacia donde apunta. Para eso hay que ver hacia donde apunta la mayoría de la evidencia. ¿Cómo proponés hacer eso?

      Quote:
      «…y todos los demás factores climáticos que yo no conozco siguen vigentes…»

      Bueno, entonces ¿cuál es la participación de cada factor «vigente»?

      No lo sé y no sé si se sabe y hasta qué nivel de precisión. Por lo pronto en mi post anterior linkeé un paper que cuantifica eso. Si no lo querés leer, te dejo mi resumen en mi blog: Cambio Climático, ¿de cuánto somos responsables?

      Acá tenés un gráfico que presenta los forzamientos y su preponderancia:

      [img]http://lh3.ggpht.com/-KjItDqY0Sp4/TupQQnBK-WI/AAAAAAAACEA/qKgnGzr2GF0/image_thumb%25255B2%25255D.png?imgmax=800[/img]

      (click para agrandar)

      O sino este otro gráfico:

      [img]http://lh3.ggpht.com/-BAAotCHwEvI/TupQTcB__tI/AAAAAAAACEQ/_YdW-Mw9n_M/image_thumb%25255B8%25255D.png?imgmax=800[/img]

      (también click para agrandar)

      También lo dice en uno de los papers que citaste, que modela la señal que se observa en la alta atmósfera y concluye que la tendencia va a ser menor que la esperada por el IPCC por la recuperación de la capa de ozono. De nuevo, en completo acuerdo con nuestro conocimiento sobre el cambio climático.

      Quote:
      -«…Además, te comento. Que la temperatura actual sea o no más alta que la temperatura en el pasado nada tiene que ver con las causas de ese aumento…»

      -«…Para empezar, las temperaturas están ACTUALMENTE mucho más altas de lo que estuvieron en los útlimos miles de años y MUY por encima de la variabilidad natural…»

      Que la temperatura haya sido más alta en otro momento no tiene que ver con la causa del calentamiento actual (de hecho, uno de los papers citados en ese infame sitio web afirma algo parecido). Pero también es cierto que las temperaturas actuales (y las que se vienen) son las más altas de los últimos miles de años y que están muy por encima de la variabilidad natural. Como dice el paper que resumo en mi post,

      Quote:
      es “extremadamente poco probable” (menos de un 5% de probabilidad) de que la variabilidad interna (el grisado en los gráficos anteriores) haya contribuido más de un 26% del calentamiento de los últimos 50 años

      Fuente: http://www.legosalogos.com.ar/2011/12/cambio-climatico-de-cuanto-somos.html

      Agrego:

      El paper que linkeás al final, concluye que la recuperación de la capa de ozono es un factor importante en la señal de la temperatura de la alta atmósfera pero:

      Quote:
      It should be pointed out that other greenhouse gases such as CO2 and CH4 are increasing and also affecting stratospheric temperatures (Ramaswamy et al. 2001).

      Que es lo que yo digo. Más aún, si querés una «cuantificación», el paper que citan (Ramaswamy et al. 2001) concluye que:

      Quote:
      Model simulations based on the known changes in the stratospheric concentrations of various radiatively active species indicate that the depletion of lower stratospheric ozone is the major factor in the explanation of the observed global-mean lower stratospheric cooling trend (;0.5– 0.6 K/decade) for the period 1979 –1990. The contribution to this trend from increases in wellmixed greenhouse gases is estimated to be less than one fourth that due to ozone loss.

      y

      Quote:
      FDH model results indicate that both well-mixed greenhouse gases and ozone changes are important contributors to the cooling in the middle and upper stratosphere; however, the computed upper stratospheric cooling is smaller than the observed decadal trend.

      y

      Quote:
      Although the well-mixed greenhouse gas and ozone change effects are undoubtedly manifest to a substantial extent in the global-mean lower stratospheric temperature changes over the past 2–3 decades, the uncertainties in the causal attribution of the long-term trend are larger on the zonal-mean spatial scales.

      Siguiendo con la búsqueda me encuentro con este paper publicado en Science y por el mismo autor principal que empieza diciendo:

      Quote:
      The difference in temperature between 2000 and 1979 has been ascribed mainly to ozone depletion and increases in well-mixed greenhouse gases (4, 6–10).

      Notablemente, la cita [4] es al mismo Ramaswamy et al. 2001 que cité anteriormente, corroborando que mi interpretación de los resultados (se puede explicar la tendencia combinando los efectos del Ozono y los Gases de efecto invernadero) es correcta. El paper además explica otras caraterísticas más detalladas también invocando factores naturales:

      Quote:
      The two steplike temperature transitions, followed by relatively steady periods (Figs. 1 and 3A), are thus a consequence of both anthropogenic and natural factors.

      En definitiva,

      Quote:
      We used a coupled atmosphere-ocean model (11–13) to demonstrate that the complex space-time pattern of the lower stratospheric temperature anomalies is a consequence of the combined temporal changes in natural forcings [solar irradiance (14) and volcanic aerosols (15)] and anthropogenic forcings [well-mixed greenhouse gases (16), stratospheric (17) and tropospheric ozone (18), tropospheric aerosols (18), and land use (13)].

      Anthropogenic and Natural Influences in the Evolution of Lower Stratospheric Cooling

      V. Ramaswamy, M. D. Schwarzkopf, W. J. Randel, B. D. Santer, B. J. Soden, and G. L. Stenchikov

      Science 24 February 2006: 311 (5764), 1138-1141. [DOI:10.1126/science.1122587]

      Finaaaalmente y para no hacerlo tan largo, dejo este paper que crea una base de datos pública corrigiendo varios sesgos y problemas con los satélites. El paper es muy técnico y no entra mucho en detalles sobre las tendencias sino más bien sobre cómo se realizan las correcciones. Sin embargo afirma que,

      Quote:
      While the composition of the stratosphere (i.e., ozone and CO2) plays an important role in global-mean stratospheric temperature trends 518 (Ramaswamy et al. 2001; Shine et al. 2003), this meridional structure of the stratospheric temperature trends revealed from this SSU dataset may be attributed to the acceleration of the stratospheric Brewer-Dobson circulation (BDC) under rising concentrations of greenhouse gases (e.g., Garcia and Randel, 2008), i.e. adiabatic cooling driven by enhanced upwelling in the tropical stratosphere.

      Los datos pueden encontrarse y descargarse acá.

      Espero que esto aclare la situación. Las tendencias de la alta atmósfera son efectivamente evidencia del efecto de los gases de efecto invernadero aunque también se puede encontrar la señal de otros factores. Si no querés llamarlo «huella innegable», llamalo «evidencia de». «No vamos a discutir cuestiones semánticas. Creo que ambos sabemos cómo funciona la ciencia y que ninguna evidencia por sí sola sirve para afirmar o refutar una teoría. Es la convergencia de las evidencias lo que importa. En este caso, la altura de la tropopausa (que sólo puede ser explicado mediante causas antropogénicas) es una de ellas.» (*). «Sobre la tropopausa y la estratósfera, como dije son parte de la evidencia, no «LA» evidencia.» (*)

      Quote:
      ¿Deshonestidad? Decime dónde en esa página dicen que el período cálido medieval fué más caliente globalmente que en la actualidad.

      Esto es lo que dijiste, textualmente, en el post que publicaste el link:

      Quote:
      Daneel, me parece que no me están «vendiendo» nada, los datos están y muestran períodos más cálidos como por ejemplo el Período Cálido Medieval.

      De todas formas, toda mi crítica anterior no se basa en eso sino en la omisión deliberada de datos.

    • #30688
      smaga
      Miembro

      Daneel Olivaw dijo:

      Ajá.. ¿y quién es el que dice que las emisiones de CO2 son el único factor que afecta el clima?

      ¿Cuál es el forzamiento que, según el CGA, tiene mayor influencia en el calentamiento?, fijate que en los gráficos que pusiste en tu último comentario, el forzamiento del CO2 es el que más influye en el calentamiento supuestamente.

      Daneel Olivaw dijo:

      Igual, gracias por darme la razón. Tu selección de datos es sorprendente. Yo, como un buen escéptico, prefiero ver TODA la evidencia, no seleccionar los datos para que me den la razón.

      Ah bueno, entonces ¿por qué seleccionás los últimos miles de años y no los últimos millones?. Y repito «…Principios de siglo 20: tendencia similar de calentamiento de fines del mismo siglo y comienzos de este, aún cuando nuestras emisiones eran muy inferiores a las de hoy…»

      Daneel Olivaw dijo:

      Sí, la mayoría. Como dije, el verdadero escéptico analiza TODA la evidencia y ve hacia donde apunta. Para eso hay que ver hacia donde apunta la mayoría de la evidencia. ¿Cómo proponés hacer eso?

      Repito (otra vez, como hace 4 meses) Que haya o no consenso sobre algo no es argumento para comprobar una hipótesis científica.

      Daneel Olivaw dijo:

      No lo sé y no sé si se sabe y hasta qué nivel de precisión. Por lo pronto en mi post anterior linkeé un paper que cuantifica eso. Si no lo querés leer, te dejo mi resumen en mi blog:

      Acá tenés un gráfico que presenta los forzamientos y su preponderancia:

      Ok, lo que me sigue llamando la atención es qué fué lo que produjo la tendencia al calentamiento a principios del siglo 20 comparado con la última etapa de calentamiento. En ese gráfico no queda para nada claro.

      Daneel Olivaw dijo:

      Que la temperatura haya sido más alta en otro momento no tiene que ver con la causa del calentamiento actual (de hecho, uno de los papers citados en ese infame sitio web afirma algo parecido). Pero también es cierto que las temperaturas actuales (y las que se vienen) son las más altas de los últimos miles de años y que están muy por encima de la variabilidad natural. Como dice el paper que resumo en mi post.

      Claro que no tiene que ver con la causa!!! pero entonces no se podría argumentar que la temperatura actual sea más alta que en alguna época anterior como prueba del CGA.

      Daneel Olivaw dijo:

      El paper que linkeás al final, concluye que la recuperación de la capa de ozono es un factor importante en la señal de la temperatura de la alta atmósfera pero:

      Quote:
      It should be pointed out that other greenhouse gases such as CO2 and CH4 are increasing and also affecting stratospheric temperatures (Ramaswamy et al. 2001).

      Y sin embargo, desde el 96 la tendencia se detuvo, aún con los niveles de emisiones actuales.

      Daneel Olivaw dijo:

      Espero que esto aclare la situación. Las tendencias de la alta atmósfera son efectivamente evidencia del efecto de los gases de efecto invernadero aunque también se puede encontrar la señal de otros factores.

      Me parece que no queda clara la situación, los papers que citás son anteriores al de Liu-Weng 2009. Ahora, y ya lo pregunté antes, si la recuperación del ozono detuvo la tendencia a enfriarse de la estratósfera, ¿cuánto influyó su destrucción en el enfriamiento y cuánto corresponde al los gases de efecto invernadero?.

      Daneel Olivaw dijo:

      Esto es lo que dijiste, textualmente, en el post que publicaste el link:

      Quote:
      Daneel, me parece que no me están «vendiendo» nada, los datos están y muestran períodos más cálidos como por ejemplo el Período Cálido Medieval.

      Yo lo dije, no está en la página y te cito «…Para demostrar que el MWP fue más caliente que la época actual no haría falta tirar 30 gráficos locales, son sólo un gráfico con la reconstrucción de la temperatura media global (hemisférica en este caso) bastaba…»

      Así que tu crítica también se basa en eso.

    • #30689

      Smaga, lo siento pero no puedo seguir una discusión que se actualiza cada dos meses. Especialmente si las repuestas son siempre la misma.

      No voy a repetirlo nuevamente pero ya te mostré varias instancias de estudios publicados en revistas con revisión por pares que muestran que los datos de las temperaturas estratosféricas NO son un problema serio para la teoría antropogénica del cambio climático y son consistentes con nuestro conocimiento actual. No sólo eso sino que los papers que vos ofreciste lo máximo que decían es que esos datos brindarían «limitada información» sobre el desarrollo del calentamiento global.

      Que se sepa o no que causó el calentamiento en los años 20 es irrelevante, un genuino arenque rojo. Es una mera pregunta retórica que trata de meter púa en vez de buscar verdaderas respuestas. Si realmente te interesa, preguntale a un climatólogo. Estoy seguro que te va a responder mucho mejor que yo.

      Lo mismo pasa con tu inocente pregunta de «¿cuánto influyó su destrucción en el enfriamiento y cuánto corresponde al los gases de efecto invernadero?». No sé, aunque sospecho que alguno de los papers que cité tiene la proporción expresa. Pero sea cual sea el porcentaje, como ya mostré citando múltiples estudios, es consistente con nuestro entendimiento actual del clima.

      Ahora, no respondiste a la pregunta principal de mi post (probablemente sea mi culpa por no resaltarla y meterla en la mitad): Para formar una opinión informada hay que mirar a toda la evidencia, ¿cómo proponés hacerlo?

    • #30690
      smaga
      Miembro

      Daneel Olivaw dijo:

      Smaga, lo siento pero no puedo seguir una discusión que se actualiza cada dos meses.

      Yo sí.

      Daneel Olivaw dijo:

      Especialmente si las repuestas son siempre la misma.

      Idem

      Daneel Olivaw dijo:

      No voy a repetirlo nuevamente pero ya te mostré varias instancias de estudios publicados en revistas con revisión por pares que muestran que los datos de las temperaturas estratosféricas NO son un problema serio para la teoría antropogénica del cambio climático y son consistentes con nuestro conocimiento actual.

      También, esos mismos datos son consistentes con la posición del SMN.

      Daneel Olivaw dijo:

      No sólo eso sino que los papers que vos ofreciste lo máximo que decían es que esos datos brindarían «limitada información» sobre el desarrollo del calentamiento global.

      No te olvides de agregar el que demuestra que la estratósfera no se está enfriando.

      Daneel Olivaw dijo:

      Que se sepa o no que causó el calentamiento en los años 20 es irrelevante, un genuino arenque rojo.

      No, si los modelos no pueden explicar calentamientos anteriores, sus resultados sobre este no serían confiables.

      Daneel Olivaw dijo:

      Es una mera pregunta retórica que trata de meter púa en vez de buscar verdaderas respuestas. Si realmente te interesa, preguntale a un climatólogo. Estoy seguro que te va a responder mucho mejor que yo.

      Si quisiera meter púa comentaría en lo de tamino. No contestás mi pregunta, si lo afirmás, lo tenés que probar.

      Daneel Olivaw dijo:

      Lo mismo pasa con tu inocente pregunta de «¿cuánto influyó su destrucción en el enfriamiento y cuánto corresponde al los gases de efecto invernadero?». No sé, aunque sospecho que alguno de los papers que cité tiene la proporción expresa. Pero sea cual sea el porcentaje, como ya mostré citando múltiples estudios, es consistente con nuestro entendimiento actual del clima.

      Si no sabés, no entiendo entonces por qué afirmás que nuestras emisiones son la principal causa del enfriamiento de la estratósfera.

      Daneel Olivaw dijo:

      Ahora, no respondiste a la pregunta principal de mi post (probablemente sea mi culpa por no resaltarla y meterla en la mitad): Para formar una opinión informada hay que mirar a toda la evidencia, ¿cómo proponés hacerlo?

      Mirando toda la evidencia. Espero que tu inquietud sobre las preguntas sin responder también te haga efecto.

    • #30691

      ¿Y vos miraste toda la evidencia?

    • #30692
      smaga
      Miembro

      Si.

    • #30693

      Jajajaja. Cuidado. Dicen que el escepticismo puede llevar a la arrogancia.

    • #30694
      smaga
      Miembro

      /sarc off.

    • #30695

      Quizás les interese esto:

      Quote:
      Lunes 26 de Marzo, 11hs, 2º Piso, DCAO, Aula 8

      COLOQUIO

      ¿Quién Controla el Cambio Climático en el Hemisferio Sur: Calentamiento Global o la reducción de Ozono?

      Dr. I. Orlanski

      Princeton University y Geophysical Fluid Dynamics Laboratory /NOAA- USA

      El Dr. Orlanski es Senior Scientist en Princeton University e investigador del GFDL (Geophysical Fluid Dynamics Laboratory /NOAA- USA). Se graduó en Física en la UBA y obtuvo su PhD en el MIT en el año 1967. Ha publicado más de 60 artículos en revistas internacionales sobre diversos aspectos de la dinámica y variabilidad de la atmósfera y el cambio climático. Ha recibido numerosos premios a lo largo de su carrera científica. Recientemente ha recibido el Premio Raices 2011 otorgado por el MINCyT por la ininterrumpida interacción científica con científicos argentinos desde hace más de 20 años.

      Resumen:

      La mayor característica del cambio climático en el Hemisferio Sur de las ultimas décadas del siglo XX es el incremento de la diferencia en la presión al nivel del mar entre latitudes medias y polares, comúnmente es conocido como el incremento de la fase positiva del modo anular del sur (índice SAM). La tendencia del índice positivo SAM se refleja en un desplazamiento hacia latitudes polares de los vientos del oeste en latitudes medias acompañando el storm-track y la expansión hacia el polo del borde sur de la celda de Hadley y la expansión hacia el polo de la zonas subtropicales de las regiones secas.

      Las causas de esas tendencias todavía son inexplicables. Hay un número considerable de artículos que sugieren que la reducción estratosférica de la concentración de ozono puede ser un contribuidor muy importante para explicar ese cambio. Sin embargo otros tantos estudios mostraron que el incremento de gases de invernadero solos, podrían también ocasionar ese tipo de tendencias. Al presente la importancia relativa de la reducción de ozono estratosférico versus el incremento de los gases de invernadero sigue siendo poco claro. Demás esta recalcar la importancia de esos cambios para entender el cambio climático en la región Sur-Americana. El seminario abordará la importancia de cada uno de estos mecanismos para comprender cual de los dos controla el cambio observado.

      Lugar: Aula 8, DCAO, 2º Piso

      Pabellón 2,

      Ciudad Universitaria.

      Los esperamos a todos!

    • #30696

      ¡Si! Nos sacan el aula de Meteorología General para dar esa charla :P. Tengo muchas ganas de ir, lástima que a esa hora tengo práctico de esa materia. Quizás me rateo y voy a la charla.

    • #30697
      Nube
      Participante

      Meteorología general… te espera un largo camino al cielo!!! Buena suerte!!!!

      Si vas al coloquio contá las novedades!

    • #30698

      ¡Gracias! :D

    • #30699
      smaga
      Miembro

      Sé que es tarde, pero de casualidad encontré esto:

      On the linkage between tropospheric and Polar Stratospheric clouds in the Arctic as observed by space–borne lidar

      Abstract. The type of Polar stratospheric clouds (PSCs) as well as their temporal and spatial extent are important for the occurrence of heterogeneous reactions in the polar stratosphere. The formation of PSCs depends strongly on temperature. However, the mechanisms of the formation of solid PSCs are still poorly understood. Recent satellite studies of Antarctic PSCs have shown that their formation can be associated with deep-tropospheric clouds which have the ability to cool the lower stratosphere radiatively and/or adiabatically. In the present study, lidar measurements aboard the Cloud-Aerosol Lidar and Infrared Pathfinder Satellite Observation (CALIPSO) satellite were used to investigate whether the formation of Arctic PSCs can be associated with deep-tropospheric clouds as well. Deep-tropospheric cloud systems have a vertical extent of more than 6.5 km with a cloud top height above 7 km altitude. PSCs observed by CALIPSO during the Arctic winter 2007/2008 were classified according to their type (STS, NAT, or ice) and to the kind of underlying tropospheric clouds. Our analysis reveals that 172 out of 211 observed PSCs occurred in connection with tropospheric clouds. 72% of these 172 observed PSCs occurred above deep-tropospheric clouds. We also find that the type of PSC seems to be connected to the characteristics of the underlying tropospheric cloud system. During the Arctic winter 2007/2008 PSCs consisting of ice were mainly observed in connection with deep-tropospheric cloud systems while no ice PSC was detected above cirrus. Furthermore, we find no correlation between the occurrence of PSCs and the top temperature of tropospheric clouds. Thus, our findings suggest that Arctic PSC formation is connected to adiabatice cooling, i.e. dynamic effects rather than radiative cooling.

      http://www.atmos-chem-phys.net/12/3791/2012/acp-12-3791-2012.html

    • #30700
      Nube
      Participante

      Smaga, ¿y esto, qué creés que significa?

    • #30701
      smaga
      Miembro

      Como estábamos debatiendo sobre el enfriamiento de la estratósfera, que la hipótesis del CGA atribuye al calentamiento de la tropósfera, en este estudio se sugiere que ese enfriamiento podría ser un enfriamiento adiabático en lugar del enfriamiento radiativo, por lo tanto habría un error bastante importante el los modelos.

    • #30702
      Nube
      Participante

      No, el paper no trata eso. El paper habla de que las nubes estratosféricas polares (NEP) podrían deberse a un enfriamiento adiabático y no a un enfriamiento radiativo, lo que corrobora un estudio de 2001, o sea que no es algo nuevo. No se dice nada sobre el enfriamiento de la estratosfera.

      Un punto interesante de las conclusiones de este paper es que, al estar las NEP relacionadas con las nubes de tormentas troposféricas, un corrimiento hacia el polo del storm track (por el cambio climático) podría aumentar las NEP y por lo tanto la destrucción de ozono.

    • #30703
      smaga
      Miembro

      La formación de las NEP está directamente relacionada con la temperatura de la misma estratósfera, como lo dice en el abstract.

      El del 2001, Teitelbaum, H., Moustaoui, M., and Fromm, M.: Exploring polar

      stratospheric cloud and ozone minihole formation: The primary

      importance of synoptic-scale flow perturbations dice entre otras cosas: «The adiabatic cooling of the air masses creates the conditions for PSC formation»

      http://www.agu.org/pubs/crossref/2001/2000JD000065.shtml

    • #30704
      Nube
      Participante

      Está claro que la formación de NEP está relacionada con la temperatura. Incluso hay distintos tipos de NEP dependiendo de la temperatura. Y, por lo que dicen estos papers, no solo importa la temperatura sino que también influyen mecanismos dinámicos.

      Supongo que te queda claro que estos papers no hablan para nada de los mecanismos de enfriamiento de la estratosfera misma y por lo tanto no se infiere de aquí ningún tipo de errores en los modelos.

    • #30705
      smaga
      Miembro

      Si bien influyen mecanismos dinamicos, la temperatura es el principal. Y no, no me queda claro ya que dicen que no hay correlacion entre la aparicion de las NEP y la temperatura de la parte superior de las nubes troposfericas (que si deberia existir segun la hipotesis radiativa).

    • #30706

      Jajajaja. De todos los papers justo elegís dos que no dicen nada sobre el cambio climático.

    • #30707
      smaga
      Miembro

      Fundamentos?

    • #30708

      Ya están escritos arriba y explicados por alguien que sabe más que yo :).

    • #30709
      smaga
      Miembro

      Ok, esperemos su fundamento a mi respuesta…..

    • #30710
      Nube
      Participante

      Smaga, última oportunidad de tener una discusión productiva.

      ¿Qué buscás cuando leés un paper? Yo, al menos en este, veo lo siguiente:

      1- En la introducción: los antecedentes del tema y las novedades. Si está bien escrito, te podés enterar rápidamente de muchas cosas. En el paper que estamos discutiendo hay un descripción aceptable de antecedentes sobre el tema concreto de que son las NEP y su observación.

      2 – En el cuerpo del trabajo: qué es lo que hicieron. Aquí, miraron los datos del Calipso, un lídar satelital (un lidar es como un radar que usa láser en lugar de sonido) para ver si las NEP estaban relacionadas o no con nubes tropsféricas. Nada más (y nada menos) que esto.

      3 – En la discusión y conclusiones: qué interpreta el que estudió el tema (y es el que sabe). En este paper, que confirmaron lo que ya habían visto otros (quizás en mediciones menos prolongadas, o en otros lugares). Y el comentario que ya puse antes, de que quizás el corrimiento del storm track con el cambio climático pueda ser peligroso para el ozono del ártico.

      Lo de la falta de correlación entre temperatura de tope de nubes y NEPs, se refiere también a distintas hipótesis sobre la formación de las NEPs.

      O sea, trato de entender a que se refiere el paper y no trato de buscar alguna frase para justificar lo que yo quiera.

      Y repito mi pregunta anterior: ¿te queda claro que este paper no habla para nada de los mecanismos de enfriamiento de la estratosfera misma y por lo tanto no se infiere de aquí ningún tipo de errores en los modelos? Si no es así, espero que expliques claramente lo que entendés del tema y no tires solamente frases aisladas.

      Para algún lector ocasional: abro un nuevo hilo explicando que son las NEP

      Las nubes del más allá

    • #30711
      smaga
      Miembro

      Nube dijo:

      Lo de la falta de correlación entre temperatura de tope de nubes y NEPs, se refiere también a distintas hipótesis sobre la formación de las NEPs.

      Ok, me expresé mal, debería haber escrito «…Como estábamos debatiendo sobre el enfriamiento de la estratósfera, que la hipótesis del CGA atribuye al calentamiento de la tropósfera, en este estudio se sugiere que el enfriamiento que precede a la formación de las NEP podría ser un enfriamiento adiabático en lugar del enfriamiento radiativo…»

      Lo que todavía no me queda claro es la falta de correlación que creo que debería haber según la hipótesis radiativa: más calor en la tropósfera, más frío en la estratósfera, por lo tanto condiciones para la formación de NEP.

      Las amenazas son para nada productivas.

    • #30712
      Nube
      Participante

      Smaga, seguís sin entender (o sin querer entender) que son cosas a distintas escalas temporales y espaciales.

      Yo puedo decir:

      -Hoy no tengo plata porque no tengo mi billetera ¿la perdí, me la robaron, me la olvidé en casa?

      o también:

      – No tengo plata para comprarme una casa (que dependerá de mis ingresos y gastos a lo largo de años)

      En los dos casos hablo de plata, pero encontrar mi billetera abajo de la mesa esta noche en casa, no me sirvirá para solucionar el problema de la casa. A no ser que tuviera U$S 100.000 en la billetera, pero entonces no diría tan tranquilamente «hoy no tengo mi billetera».

      Para llegar de la discusión sobre el mecanismo de formación de NEP a la tendencia de enfriamiento de la estratosfera a largo a plazo hay un camino muuuuuyyyyyy largo. Aunque en los dos casos se hable de radiación/enfriamiento/dinámica/ etc.

    • #30713
      smaga
      Miembro

      Nube dijo:

      Para llegar de la discusión sobre el mecanismo de formación de NEP a la tendencia de enfriamiento de la estratosfera a largo a plazo hay un camino muuuuuyyyyyy largo. Aunque en los dos casos se hable de radiación/enfriamiento/dinámica/ etc.

      Ahora comprendo mejor, gracias.

Mostrando 234 respuestas a los debates
  • Debes iniciar sesión para responder a este tema.